On-line: гостей 7. Всего: 7 [подробнее..]
АвторСообщение
RedStar
постоянный участник




Пост N: 2452
Зарегистрирован: 02.08.16
Откуда: Смоленск
Рейтинг: 1
ссылка на сообщение  Отправлено: 14.10.19 16:45. Заголовок: Что такое Ri?


Прошу высказать полное обоснование необходимости расчетов Ri.
Сколько сталкиваюсь, столько прихожу к мнению, что знать Ri нет необходимости.

Не надо иметь много колес, лучше иметь много спиц, удерживающих одно колесо. © Спасибо: 0 
Профиль
Ответов - 299 , стр: 1 2 3 4 5 6 7 8 9 10 11 12 13 14 15 All [только новые]


Пермяк
moderator




Пост N: 6323
Зарегистрирован: 15.01.10
Откуда: Россия
Рейтинг: 24
ссылка на сообщение  Отправлено: 15.10.19 14:34. Заголовок: Попробую-таки в очер..


Попробую (в очередной раз) пояснить, что такое Ri, попроще.

RedStar, нагрузка, включенная в ламповый каскад, "видит" лампу, как источник переменного напряжения (или источник переменного тока), имеющий именно такое внутреннее сопротивление, как Ri данной лампы.

http://hiend.borda.ru/?1-13-20-00000091-000-0-0-1427557566

RedStar, опробуйте хотя бы осмыслить, не торопитесь комментировать и выкладывать свои открытия.

Спасибо: 0 
Профиль
Кузьмич



Пост N: 1390
Зарегистрирован: 21.11.15
Рейтинг: 8
ссылка на сообщение  Отправлено: 15.10.19 16:03. Заголовок: r9o-11 пишет: так а..


RedStar, лампа для нагрузки является генератором переменного напряжения. Если вместо лампы поставить генератор с теми же выходными параметрами напряжения и тока, то получишь то же самое.


Пермяк пишет:
 цитата:
Попробуйте хотя бы осмыслить,

Неоднократно замечал, что он путается, скажем так, с сопротивлением лампы по постоянному току, т.е. Ua/Iа.
Да ещё хочет поймать этих блох в разной точке нагрузочной. Забывая или не понимая того, что для определения Ri в рабочей точке, напряжение на сетке должно быть постоянно, конста́нта.

Спасибо: 0 
Профиль
RedStar
постоянный участник




Пост N: 2464
Зарегистрирован: 02.08.16
Откуда: Смоленск
Рейтинг: 1
ссылка на сообщение  Отправлено: 15.10.19 19:06. Заголовок: Кузьмич пишет: Ri п..


Кузьмич пишет:
 цитата:
Ri при расчётах каскадов УНЧ принимается в рабочей точке, а дальнейшее его изменение в пределах нагрузочной линии, как правило не учитывается.

Если учитывается в РТ, а далее нет, почему? Потому ли, что оно будет разным и в моменты времени и при разных напряжений?
Да и не про РТ идет разговор, где каждое значение будет константой относительно других.
При подсчете эквивалентного сопротивления, вообще ничего не получится, так как как раз не учитывается выше приведенный фактор при разных значениях по нагрузочной.

Генератор - устройство, производящее какие-либо продукты, вырабатывающее электроэнергию или преобразующее один вид энергии в другой.
Ключевое слово - вырабатывающие. Но лампа преобразует постоянное напряжение в переменное под действием внешнего переменного напряжения.

Пермяк пишет:
 цитата:
НАГРУЗКА "видит" лампу, как генератор переменного напряжения (или генератор переменного тока), который имеет внутреннее сопротивление, равное Ri лампы.

Нагрузка какая? Про нее потом.

Не надо иметь много колес, лучше иметь много спиц, удерживающих одно колесо. © Спасибо: 0 
Профиль
Stan Marsh





Пост N: 1879
Зарегистрирован: 20.08.13
Откуда: Россия, Москва
Рейтинг: 2
ссылка на сообщение  Отправлено: 15.10.19 19:23. Заголовок: Краснозвёздыч, Вы за..


Краснозвёздыч, Вы бы засунули свой гонор туда, где ему место, и честно признались бы в том, что просто не понимаете. Глядишь, и дело бы пошло. Так нет! Пытаетесь сохранить лицо. А со стороны видна рожа, без проблесков интеллекта, с тупой гордыней и без желания учиться. И лично у меня нет никакого желания Вам помогать.

Спасибо: 0 
Профиль
RedStar
постоянный участник




Пост N: 2465
Зарегистрирован: 02.08.16
Откуда: Смоленск
Рейтинг: 1
ссылка на сообщение  Отправлено: 15.10.19 19:42. Заголовок: Гордыня скорее у вас..


Гордыня скорее у вас всех прет от великого знания книжных истин.
Что то не понимаю, спрашиваю. Только ответы всегда отрицательный у вас на мои вопросы.
Верно, сохранить "свое лицо" пытаюсь, а ваши "рожи" пропитаны брюзжанием.
Учиться можно и нужно. Только без Хороших учителей вряд ли получится у кого либо.

Не надо иметь много колес, лучше иметь много спиц, удерживающих одно колесо. © Спасибо: 0 
Профиль
Кузьмич



Пост N: 1392
Зарегистрирован: 21.11.15
Рейтинг: 8
ссылка на сообщение  Отправлено: 15.10.19 19:47. Заголовок: RedStar пишет: Гене..


RedStar пишет:
 цитата:
Генератор - устройство, производящее какие-либо продукты, вырабатывающее электроэнергию или преобразующее один вид энергии в другой.

Толя, в очередной раз удивляюсь, какой ты глупый...
Это общее понятие генератора. Превращение механической или тепловой... энергии в электрическую. А ты слышал когда-нибудь про мост Ви́на? Кажется так звали немецкого физика.
Который, скажем так, изобрёл электронный генератор синусоидальных колебаний. Сам мост состоит из пассивных элементов. А если его включить в цепь положительной обратной связи усилителя, выполнить условие баланса фаз и амплитуд, то усилитель превратится в генератор
гармонических колебаний.

RedStar пишет:
 цитата:
Нагрузка какая? Про нее потом.

Нет, дорогой. Лампа без нагрузки - это кусок стекла, металла и карболита. Ничего ты с неё кроме тепла не получишь.
Поэтому - только и только с нагрузкой. Оную выбираешь сам, исходя из Ri в рабочей точке, которую сам же и определишь....
 цитата:
Если учитывается в РТ, а далее нет, почему?

Потому что, решив собрать схему на лампе, ты и только ты выбираешь для неё режим, т.е. РТ с которой она начнёт работать.
И все её диф. параметры берутся или рассчитываются, или измеряются исходя из выбранной ЭТОЙ РТ.
Исходя из этих параметров рассчитывается эквивалентное сопротивлений (оно нужно больше для НЧ), задаётся Альфа и т.д, и т.п. ...
Всё остальное о чём ты думаешь - никакого значения не имеет.
Я не зря привёл аналогию с сетевой розеткой 220 вольт... "Бьёт" всегда 220, не зависимо от того, что замкнул собой в тот момент, когда было 25 вольт, или 57, или больше.
Ты задаёшь вопросы, типа:
- Почему апельсины круглые?
- Почему в дырках ничего нет?
Не забивай голову посторонними вещами.

Спасибо: 0 
Профиль
RedStar
постоянный участник




Пост N: 2466
Зарегистрирован: 02.08.16
Откуда: Смоленск
Рейтинг: 1
ссылка на сообщение  Отправлено: 18.10.19 16:33. Заголовок: Не приписывайте мне,..


Не приписывайте мне, что я не знаю как высчитывается Ri.
Хочу уточнить одну деталь, почему должно увеличиться (или уменьшиться) анодное напряжение, при константе Мю, в дин. режиме, с выбранной высокоомной нагрузкой?

Не надо иметь много колес, лучше иметь много спиц, удерживающих одно колесо. © Спасибо: 0 
Профиль
Плюмбум





Пост N: 267
Зарегистрирован: 14.12.16
Рейтинг: 2
ссылка на сообщение  Отправлено: 19.10.19 00:18. Заголовок: Кто-нибудь понял, о ..


Кто-нибудь понял, о чём спрашивает RedStar ?

Спасибо: 0 
Профиль
Пермяк
moderator




Пост N: 6333
Зарегистрирован: 15.01.10
Откуда: Россия
Рейтинг: 24
ссылка на сообщение  Отправлено: 19.10.19 00:43. Заголовок: Мне кажется, его воп..


Мне кажется, его вопрос можно перефразировать так: почему коэфф. усиления каскада (Ку) всегда меньше, чем Мю (µ) лампы?
Анатолий, я угадал?

Спасибо: 0 
Профиль
Кузьмич



Пост N: 1395
Зарегистрирован: 21.11.15
Рейтинг: 8
ссылка на сообщение  Отправлено: 19.10.19 03:06. Заголовок: Пермяк пишет: Анато..


Пермяк пишет:
 цитата:
Анатолий, я угадал?

А я пыаюсь понимать по другому. Ведь
RedStar пишет:
 цитата:
с выбранной высокоомной нагрузкой?

Например взята лампа 6Н2П или 6Н9С, или 6Г2... Где сопротивление анодной нагрузки высокоомное и может быть 150, 200, 270 и даже более кОм. Почему при такой огромной нагрузке напряжение на аноде должно изменяться под воздействием входного сигнала (т.е. при изменении сеточного напряжения)???? Кажется так надо понимать его вопрос.
Только при чём там Мю константа, не понимаю...

Спасибо: 0 
Профиль
RedStar
постоянный участник




Пост N: 2468
Зарегистрирован: 02.08.16
Откуда: Смоленск
Рейтинг: 1
ссылка на сообщение  Отправлено: 19.10.19 04:19. Заголовок: Пермяк пишет: я уга..


Пермяк пишет:
 цитата:
я угадал?

Нет.
Кузьмич пишет:
 цитата:
Почему при такой огромной нагрузке........

Нет.

Почему должно изменится анодное напряжение дельта Ua при нагрузке Ra?


Не надо иметь много колес, лучше иметь много спиц, удерживающих одно колесо. © Спасибо: 0 
Профиль
Кузьмич



Пост N: 1396
Зарегистрирован: 21.11.15
Рейтинг: 8
ссылка на сообщение  Отправлено: 19.10.19 12:20. Заголовок: r9o-11 пишет: Почем..


RedStar пишет:
 цитата:
Почему должно изменится анодное напряжение дельта Ua при нагрузке Ra?

А дальше читаем и осмысливаем, потом запоминаем учебник. Например вот этот:



Т.е. не почему должно измениться, а на сколько надо изменить анодное напряжение, что бы анодный ток изменился на 1 ма, в
рабочей точке, которую выбрал самодельщик, т.е. ты.
И при чём тут Ra?.

RedStar пишет:
 цитата:
Не приписывайте мне, что я не знаю как высчитывается Ri.

Дело значительно хуже, ты не знаешь что такое Ri.

Спасибо: 0 
Профиль
Пермяк
moderator




Пост N: 6334
Зарегистрирован: 15.01.10
Откуда: Россия
Рейтинг: 24
ссылка на сообщение  Отправлено: 19.10.19 12:54. Заголовок: На рисунке Редстара ..


На рисунке Редстара изображён характеристический треугольник для раб.точки красного цвета.
Поскольку гипотенуза этого треугольника - касательная к раб, точке, то такой треугольник предназначен для определения Ri в данной точке:
∆Ua/∆Ia=Ri
(напряжение делённое на ток - это ведь сопротивление, не так ли?)

Если рассматривается трансформаторный каскад, то при изменении нагрузки изменится угол наклона нагрузочной при той же рабочей точке А, а величина Ri не изменится:



ЗЫ. Однако, о каком Мю ранее спрашивал Редстар, не понятно.

ЗЗЫ. Ох, Куззмич опять опередил, экий торопыга!

Спасибо: 0 
Профиль
RedStar
постоянный участник




Пост N: 2469
Зарегистрирован: 02.08.16
Откуда: Смоленск
Рейтинг: 1
ссылка на сообщение  Отправлено: 19.10.19 13:11. Заголовок: Все не поняли вопрос..


Все не поняли вопроса.

Кузьмич пишет:
 цитата:
не почему должно измениться, а на сколько надо изменить анодное напряжение,

Зачем изменять напряжение? Что бы узнать Ri, это-то понятно.

Но Почему должно изменится анодное напряжение в конкретной схеме (режиме)???
В РТ установлен режим "А". Каким образом изменится Ua0 при изменении нагрузки?

Пермяк пишет:
 цитата:
при изменении нагрузки изменится угол наклона нагрузочной при той же рабочей точке А, а величина Ri не изменится

Само собой.
Вы все так говорите, будто я не знаю.

Не надо иметь много колес, лучше иметь много спиц, удерживающих одно колесо. © Спасибо: 0 
Профиль
Кузьмич



Пост N: 1397
Зарегистрирован: 21.11.15
Рейтинг: 8
ссылка на сообщение  Отправлено: 19.10.19 13:35. Заголовок: RedStar пишет: Заче..


RedStar пишет:
 цитата:
Зачем изменять напряжение? (что бы узнать Ri, это то понятно)

Чудеса! Тогда не понятно, что тебе не понятно.
RedStar пишет:
 цитата:
но Почему должно изменится анодное напряжение в конкретной схеме (режиме)???

Да не должно ОНО меняться! Ты сам себе ответил на вопрос:
RedStar пишет:
 цитата:
(чтобы узнать Ri, ...)



Спасибо: 0 
Профиль
RedStar
постоянный участник




Пост N: 2471
Зарегистрирован: 02.08.16
Откуда: Смоленск
Рейтинг: 1
ссылка на сообщение  Отправлено: 19.10.19 13:41. Заголовок: Кузьмич пишет: Да н..


Кузьмич пишет:
 цитата:
Да не должно ОНО меняться!

Тогда зачем считать Ri?
Если оно дает неточный расчет нагрузки.

Не надо иметь много колес, лучше иметь много спиц, удерживающих одно колесо. © Спасибо: 0 
Профиль
Пермяк
moderator




Пост N: 6336
Зарегистрирован: 15.01.10
Откуда: Россия
Рейтинг: 24
ссылка на сообщение  Отправлено: 19.10.19 13:59. Заголовок: RedStar пишет: Тогда..


RedStar пишет:
 цитата:
Тогда зачем считать Ri?
Если оно дает неточный расчет нагрузки.

Ri - это параметр ЛАМПЫ.
А нагрузку мы не рассчитываем, а ВЫБИРАЕМ, задаём, например для триода - в пределах (3...5...7)Ri, и более.
Желаем побольше мощность - выбираем 2Ri, но Кг будет относительно большим, желаем меньшее Кг - выбираем (3...7)Ri, но мощность на выходе получим меньше.
Какая тут может быть точность? Всё зависит от наших предпочтений.

Спасибо: 0 
Профиль
RedStar
постоянный участник




Пост N: 2472
Зарегистрирован: 02.08.16
Откуда: Смоленск
Рейтинг: 1
ссылка на сообщение  Отправлено: 19.10.19 14:15. Заголовок: Кузьмич пишет: Что ..


Кузьмич пишет:
 цитата:
Что значит не точный?

Такой вот не точный:
Пермяк пишет:
 цитата:
нагрузку мы ВЫБИРАЕМ, например для триода - в пределах (3...5...7)Ri, и более.

Кузьмич пишет:
 цитата:
И какая точность тебе нужна?

Расчетная, до 5%. Выбирая Ra от Ri, ее не получить никогда.

Не надо иметь много колес, лучше иметь много спиц, удерживающих одно колесо. © Спасибо: 0 
Профиль
Кузьмич



Пост N: 1399
Зарегистрирован: 21.11.15
Рейтинг: 8
ссылка на сообщение  Отправлено: 19.10.19 14:17. Заголовок: Можно и не считать R..


Можно и не считать Ri, а взять его из даташита, с условием, что РТ будет такая же, как указано в даташите.

RedStar пишет:
 цитата:
Расчетная, до 5%. Но с Ri ее не получить никогда.

Получишь и даже меньше.
Для этого тоже есть вариант, снимаем самостоятельно ВАХи конкретной лампы и считаем Ri в РТ.
А лучше пользоваться программами. Это удобно, быстро и точно. Сайчас множество ВАХов различных ламп, снятыми людьми для облегчения.

П.С. Когда года подходят к 70... остро понимаешь, как мало мы живём. И как много потеряно ценного времени на всякую хрень.

Спасибо: 0 
Профиль
Пермяк
moderator




Пост N: 6337
Зарегистрирован: 15.01.10
Откуда: Россия
Рейтинг: 24
ссылка на сообщение  Отправлено: 19.10.19 14:20. Заголовок: RedStar , прочти ещё..


RedStar Повторюсь: если мы изменяем нагрузку по своей прикидке, зачем тебе точность 5% ?

Спасибо: 0 
Профиль
RedStar
постоянный участник




Пост N: 2473
Зарегистрирован: 02.08.16
Откуда: Смоленск
Рейтинг: 1
ссылка на сообщение  Отправлено: 19.10.19 14:34. Заголовок: Кузьмич пишет: Полу..


Кузьмич пишет:
 цитата:
Получишь и даже меньше.
Для этого тоже есть вариант, снимаем самостоятельно ВАХи конкретной лампы и считаем Ri в РТ.

Самостоятельно можно снимать, но зачем? Есть готовые ВАХи.
Как же можно меньше %? Приведите пример.

Пермяк пишет:
 цитата:
Если мы изменяем нагрузку по своей прикидке...

Я не изменяю нагрузку. Ее рассчитываю сразу.

П.С.
Пермяк пишет:
 цитата:
А при увеличении Ra напряжение Ua0 приходится задавать бОльшим (при этом часто - и ток Ia0 снижать) чтобы вых. мощность не снижалась, или снижалась не так сильно.

Так это справедливо для выходного каскада, и то не обязательно, если потеря мощности не важна.
А для входного не обязательно это условие, если не превышать мощность анода.

Не надо иметь много колес, лучше иметь много спиц, удерживающих одно колесо. © Спасибо: 0 
Профиль
Пермяк
moderator




Пост N: 6339
Зарегистрирован: 15.01.10
Откуда: Россия
Рейтинг: 24
ссылка на сообщение  Отправлено: 19.10.19 14:52. Заголовок: RedStar пишет: Самос..


RedStar пишет:
 цитата:
Я не изменяю нагрузку. Ее рассчитываю сразу.

Не получится "сразу". Всегда приходится делать несколько заходов.
И полученный результат всё равно не обеспечить с такой точностью, т.к. ещё не имеешь точного расчёта ТВЗ, который считают уже после расчёта каскада.
ЗЫ. А как у тебя с расчётом ТВЗ? Неужели научился? С учётом твоих КО и ЭО ?

RedStar пишет:
 цитата:
Самостоятельно можно снимать, но зачем? Есть готовые ВАХи.

Готовые? В даташите? Читаем даташит:


Посчитайте процент разброса.

Спасибо: 0 
Профиль
RedStar
постоянный участник




Пост N: 2474
Зарегистрирован: 02.08.16
Откуда: Смоленск
Рейтинг: 1
ссылка на сообщение  Отправлено: 19.10.19 14:53. Заголовок: Для входных, драйвер..


Для входных, драйверных, каскадов рассчитывается сразу, без Ri.

Пермяк пишет:
 цитата:
ещё не имеешь точного расчёта ТВЗ, который считают уже после расчёта каскада.

А если все вместе считается?

ТВЗ вообще по другому рассчитываю. Про ЭО долго думал. Нашел принцип, похож на тот, что ранее делал ветку об ЭС, но еще не довел к основанию.

П.С. Про даташит.
Мне же не надо Ri, я про график ВАХ.

Не надо иметь много колес, лучше иметь много спиц, удерживающих одно колесо. © Спасибо: 0 
Профиль
Пермяк
moderator




Пост N: 6340
Зарегистрирован: 15.01.10
Откуда: Россия
Рейтинг: 24
ссылка на сообщение  Отправлено: 19.10.19 15:02. Заголовок: RedStar пишет: Мне ж..


RedStar пишет:
 цитата:
Мне же не надо Ri, я про график ВАХ.

Если Ri имеет разброс, то и графики ВАХ - такой же. Ri ведь снимается именно с ВАХ.
Припомните Ваше недоумение по поводу несоответствия в разных даташитах ВАХ ГУ-50 в пентоде...

Спасибо: 0 
Профиль
RedStar
постоянный участник




Пост N: 2475
Зарегистрирован: 02.08.16
Откуда: Смоленск
Рейтинг: 1
ссылка на сообщение  Отправлено: 19.10.19 15:09. Заголовок: Есть график. Нужны т..


Есть график. Нужны только его ветви и оси, по которым находится нагрузка и прочее, не прибегая к вычислению и даташитного Ri.
Да без этой Ri все считается.
Пусть имеется разброс. А смещение для чего? Им же можно полуоси нагрузки выставить относительно РТ.

Пермяк пишет:
 цитата:
Припомните Ваше недоумение по поводу несоответствия в разных даташитах ВАХ ГУ-50 в пентоде...

Было дело. Теперь это можно обойти.

Не надо иметь много колес, лучше иметь много спиц, удерживающих одно колесо. © Спасибо: 0 
Профиль
Кузьмич



Пост N: 1400
Зарегистрирован: 21.11.15
Рейтинг: 8
ссылка на сообщение  Отправлено: 19.10.19 15:10. Заголовок: RedStar пишет: Как ..


RedStar пишет:
 цитата:
Как же можно меньше %? Приведите пример.

Толя, не понимаю, тупишь??? Я же объяснил.
Ну хорошо, снимаешь ВАХи, своей конкретной лампы
Например 6С3П, при различных напряжениях анода, те, которые ты желаешь применить.




И вычисляешь Ri. Буржуйские приборы обозначают это Rp.



RedStar пишет:
 цитата:
без этой Ri все считается.

Это у кустарщиков всё так считается. А ты на техническом форуме.
И к примеру, твоя "транзисторная", но суперская акустика (8 Ом), не может нормально воспроизводить с усилителем, вых сопротивление которого больше 1 Ом.
И как ты будешь рассчитывать вых. сопр. усилителя без Ri??? Расскажи. Как кустарщик???

Спасибо: 0 
Профиль
RedStar
постоянный участник




Пост N: 2476
Зарегистрирован: 02.08.16
Откуда: Смоленск
Рейтинг: 1
ссылка на сообщение  Отправлено: 19.10.19 15:18. Заголовок: Кузьмич, зачем мне R..


Кузьмич, зачем мне Ri?
А дальше расчет вычисления Ra, ...?
Из десятков 6с3п и 6с4п ни одна не подходит под ВАХ Клауса. Под "наши" - идеально.
РТ для них Ua=140, Uсм=-2,0. Ra=30к

Кузьмич пишет:
 цитата:
А ты на техническом форуме.

Именно. Разве технически нельзя применить другое вычисление?
 цитата:
И как ты будешь рассчитывать вых. сопр. усилителя без Ri??? Расскажи. Как кустарщик???

Почему выходное сопротивление считается от разницы нагрузок? Ктр меняется, - меняется переменный ток анода.
Ri остается неизменным, а Ra изменяется.
(Выше, у Леонида, пост N: 6334)

Не надо иметь много колес, лучше иметь много спиц, удерживающих одно колесо. © Спасибо: 0 
Профиль
Кузьмич



Пост N: 1401
Зарегистрирован: 21.11.15
Рейтинг: 8
ссылка на сообщение  Отправлено: 19.10.19 15:41. Заголовок: RedStar пишет: заче..


RedStar пишет:
 цитата:
зачем мне Ri?

Чего ты тогда пристаёшь к народу???
Вот за это высказывание его надобно и близко не подпускать к форуму.

Спасибо: 0 
Профиль
RedStar
постоянный участник




Пост N: 2480
Зарегистрирован: 02.08.16
Откуда: Смоленск
Рейтинг: 1
ссылка на сообщение  Отправлено: 19.10.19 15:48. Заголовок: Кузьмич пишет: Чего..


Кузьмич пишет:
 цитата:
Чего ты тогда пристаёшь к народу???

Нужен принцип его применения. Зачем и для чего, расчетами.
В сравнении познается разница методик.

П.С. Кузьмич, не надо категорично высказываться. Нравится вам это или нет, - не дает право делать такие выводы.

Не надо иметь много колес, лучше иметь много спиц, удерживающих одно колесо. © Спасибо: 0 
Профиль
r9o-11





Пост N: 572
Зарегистрирован: 28.01.15
Откуда: Искитим, НСО
Рейтинг: 3
ссылка на сообщение  Отправлено: 19.10.19 16:09. Заголовок: А Вы не читали описа..


А Вы читали описание к программе Tube TransCalc? Здесь лежит
Там есть формулы, применяемые в расчётах и по ним можно посмотреть что к чему привязывается и как зависит - разве это не принцип применения?

Андрей Спасибо: 0 
Профиль
Кузьмич



Пост N: 1402
Зарегистрирован: 21.11.15
Рейтинг: 8
ссылка на сообщение  Отправлено: 19.10.19 16:10. Заголовок: RedStar пишет: Нуже..


RedStar пишет:
 цитата:
Нужен принцип его применения. Зачем и для чего

Всё в учебнике, который тебе не под силу.
Остальное - всё твои выдумки = чушь!

Спасибо: 0 
Профиль
RedStar
постоянный участник




Пост N: 2481
Зарегистрирован: 02.08.16
Откуда: Смоленск
Рейтинг: 1
ссылка на сообщение  Отправлено: 19.10.19 16:16. Заголовок: r9o-11, спасибо. Куз..


r9o-11, спасибо.

Кузьмич пишет:
 цитата:
Остальное - всё твои выдумки

Это как посмотреть.
Всего лишь задаю вопросы и нужны ответы. Я не предлагаю вам это как истину.
Сравнение для меня важно.

Не надо иметь много колес, лучше иметь много спиц, удерживающих одно колесо. © Спасибо: 0 
Профиль
rv10



Пост N: 285
Зарегистрирован: 23.09.19
Рейтинг: -1
ссылка на сообщение  Отправлено: 20.10.19 15:32. Заголовок: RedStar пишет: Ri о..


RedStar пишет:
 цитата:
Ra изменяется, а Ri остается неизменным.

И поэтому Ri знать не нужно?

Turn on, tune in, drop out Спасибо: 0 
Профиль
RedStar
постоянный участник




Пост N: 2489
Зарегистрирован: 02.08.16
Откуда: Смоленск
Рейтинг: 1
ссылка на сообщение  Отправлено: 20.10.19 16:23. Заголовок: rv10 пишет: Полное ..


rv10, так объясните популярно, зачем нужно Ri. Никто не отвечает по сути, полным объемом.

П.С. Есть хоть какая книга, где существует практические работы с теоретическими расчетами объяснения Ri?
Сказал же - я сравниваю разные методики!

Не надо иметь много колес, лучше иметь много спиц, удерживающих одно колесо. © Спасибо: 0 
Профиль
r9o-11





Пост N: 577
Зарегистрирован: 28.01.15
Откуда: Искитим, НСО
Рейтинг: 3
ссылка на сообщение  Отправлено: 20.10.19 16:46. Заголовок: "Проектирование ..


"Проектирование схем на компьютере" Васильченко и Наседкина читали?

Андрей Спасибо: 0 
Профиль
Aleph





Пост N: 599
Зарегистрирован: 03.06.15
Откуда: Макеевка
Рейтинг: 7
ссылка на сообщение  Отправлено: 20.10.19 16:53. Заголовок: Теперь то я понимаю ..


Не понимаю что тут во внутреннем сопротивлении сложного. Допустим мы даже ВАХов лампы не знаем. Просто подали напряжение на неё (при постоянном смещении) и получили определенный ток. По закону Ома мы узнаем сопротивление лампы в этой точке. При других режимах будет и другое сопротивление. Если бы сопротивление не менялось с режимом, лампа бы являлась обычным резистором.
Готовые ВАХи нам нужны уже для того, чтобы выбрать такую нагрузку чтоб усиление лампы на положительной и отрицательной полуволне было максимально одинаковым.
Как люди при одном размере одежды всё же отличаются, так и лампы, хоть и сделанные одинаково, немного отличаются по параметрам. Мы пользуемся в жизни стандартными размерами одежды, так и с лампами пользуемся усредненными ВАХами подразумевая что они не идеально описывают работу именно нашей лампы.
Что тут может быть такого мудрёного, чтоб создавать отдельную тему и не спать ночами?

С ув. Сергей Спасибо: 0 
Профиль
r9o-11





Пост N: 578
Зарегистрирован: 28.01.15
Откуда: Искитим, НСО
Рейтинг: 3
ссылка на сообщение  Отправлено: 20.10.19 17:04. Заголовок: Может быть, вечная б..


Может быть, вечная борьба с компромиссами не даёт спать по ночам?
Ведь выбрать такую нагрузку чтоб усиление лампы на положительной и отрицательной полуволне было максимально одинаковым нужно при некоторых условиях, которые часто связаны между собой и нередко противоречат друг другу.

Андрей Спасибо: 0 
Профиль
Пермяк
moderator




Пост N: 6349
Зарегистрирован: 15.01.10
Откуда: Россия
Рейтинг: 24
ссылка на сообщение  Отправлено: 20.10.19 17:04. Заголовок: Aleph пишет: подали ..


Aleph пишет:
 цитата:
подали напряжение на неё (при постоянном смещении) и получили определенный ток. По закону Ома мы узнаем сопротивление лампы в этой точке.

При этом мы получите значение Ro (Эр нулевое) - сопротивление лампы по ПОСТОЯННОМУ току (для практических расчётов ненужное) а не Ri - внутреннее сопротивление ПЕРЕМЕННОМУ току.

forensic anthropology working conditions

Спасибо: 0 
Профиль
RedStar
постоянный участник




Пост N: 2491
Зарегистрирован: 02.08.16
Откуда: Смоленск
Рейтинг: 1
ссылка на сообщение  Отправлено: 20.10.19 17:22. Заголовок: r9o-11 пишет: выбра..


Aleph, не совсем понял, что имеете в виду.
Не зная ВАХ лампы, можно применить даташит, в котором указаны максимальные значения. Исходя из них, делаем поправку на уменьшение.
Собираю макет, где рассчитываю своим методом и общепринятым методом. Знаете, общепринятым, получается полная "лажа".
Подсчитайте по всему диапазону от 2 до 10 Ri (для триодов, и 0,1-0,5 для пентодов). Что получится перебирая такое количество неизвестных?

r9o-11 пишет:
 цитата:
выбрать такую нагрузку чтоб усиление лампы на положительной и отрицательной полуволне было максимально одинаковым

Дело скорее не в нагрузке, ее влияние только на 60% примерно. Больше влияет напряжение смещения, которое как раз определяет положение РТ для равнозначности полуволн.
Нагрузка дает оптимум переменного напряжения и переменного тока. Отсюда, - гармонические искажения.
А есть ли где расчет влияния смещения к изменению других величин, как то напряжения и Тока с применением конкретных формул?

Пермяк пишет:
 цитата:
сопротивление лампы по ПОСТОЯННОМУ току (для практических расчётов ненужное)

Как это не нужное? Привел же доказательство, как с его помощью можно точно рассчитать питание и ожидаемый коэффициент усиления.

Не надо иметь много колес, лучше иметь много спиц, удерживающих одно колесо. © Спасибо: 0 
Профиль
Пермяк
moderator




Пост N: 6350
Зарегистрирован: 15.01.10
Откуда: Россия
Рейтинг: 24
ссылка на сообщение  Отправлено: 20.10.19 17:31. Заголовок: RedStar пишет: Приве..


RedStar пишет:
 цитата:
Как это не нужное?

А так. Ua0 и Ia0 - задаём, и пользуемся их величинами, а зачем нам делить Ua0/Ia0, чтобы получить Ro? Куда потом это Ro приткнуть, где использовать?
 цитата:
Привел же доказательство, как с его помощью можно точно рассчитать питание и ожидаемый коэффициент усиления.

Прошу прощения, но НИЧЕГО ВЫ НЕ ПРИВЕЛИ.

Спасибо: 0 
Профиль
r9o-11





Пост N: 579
Зарегистрирован: 28.01.15
Откуда: Искитим, НСО
Рейтинг: 3
ссылка на сообщение  Отправлено: 20.10.19 17:31. Заголовок: RedStar пишет: Нагр..


RedStar пишет:
 цитата:
Нагрузка дает оптимум переменного напряжения и переменного тока.

Что такое "оптимум"? Что важнее - КПД усилителя, его КНИ и ИМД, Кдемпфирования нагрузки, рабочая полоса или какой-либо другой параметр?

Андрей Спасибо: 0 
Профиль
RedStar
постоянный участник




Пост N: 2494
Зарегистрирован: 02.08.16
Откуда: Смоленск
Рейтинг: 1
ссылка на сообщение  Отправлено: 20.10.19 17:41. Заголовок: Пермяк пишет: Прошу..


Пермяк пишет:
 цитата:
Прошу прощения, но НИЧЕГО ВЫ НЕ ПРИВЕЛИ.

При всем уважении, было мной приведено в удаленной ветке "Беседы с ...".
r9o-11 пишет:
 цитата:
Что такое "оптимум"?

Это наиболее благоприятный режим исходя из выбранной РТ. Который рассчитан верно, а не длительным подбором.
Если режим некорректен, то пессиум покажет отклонение от режима.
 цитата:
Что важнее -...

Когда настроено правильно и отобраны исключительные параметры, то и КПД будет высоким.
Как то выше написал, что нужна совокупность расчета. Она определяет сразу два каскада - выходной лампы и трансформатора.

Не надо иметь много колес, лучше иметь много спиц, удерживающих одно колесо. © Спасибо: 0 
Профиль
r9o-11





Пост N: 580
Зарегистрирован: 28.01.15
Откуда: Искитим, НСО
Рейтинг: 3
ссылка на сообщение  Отправлено: 20.10.19 17:43. Заголовок: Дык, наверное, ответ..


Пермяк, так, наверное, ответа и нет - это же всё компромиссы, переходящие во вкусовщину...

Андрей Спасибо: 0 
Профиль
Aleph





Пост N: 600
Зарегистрирован: 03.06.15
Откуда: Макеевка
Рейтинг: 7
ссылка на сообщение  Отправлено: 20.10.19 17:44. Заголовок: r9o-11 пишет: Ведь ..


r9o-11 пишет:
 цитата:
Ведь выбрать такую нагрузку чтоб усиление лампы на положительной и отрицательной полуволне было максимально одинаковым нужно при некоторых условиях, которые часто связаны между собой и нередко противоречат друг другу.

А ещё полное сопротивление АС прыгает с частотой в несколько раз и вся это идеально выверенная нагрузка оборачивается пшиком. Поэтому я перешёл на двухтакты. Там как ни прыгает нагрузка всё равно суммарно выходит симметрично при подобранных лампах.

to Пермяк (на пост 6349) да, Леонид, тут я напутал с сопротивлениями...

С ув. Сергей Спасибо: 0 
Профиль
r9o-11





Пост N: 581
Зарегистрирован: 28.01.15
Откуда: Искитим, НСО
Рейтинг: 3
ссылка на сообщение  Отправлено: 20.10.19 17:48. Заголовок: Так я про это и гово..


Так я и про это тоже говорю - сможет ли усилитель работать с разными нагрузками или максимально хорошо только с определёнными?..

Андрей Спасибо: 0 
Профиль
RedStar
постоянный участник




Пост N: 2495
Зарегистрирован: 02.08.16
Откуда: Смоленск
Рейтинг: 1
ссылка на сообщение  Отправлено: 20.10.19 17:50. Заголовок: Aleph пишет: А ещё ..


Aleph пишет:
 цитата:
А ещё полное сопротивление АС прыгает с частотой в несколько раз и вся это идеально выверенная нагрузка оборачивается пшиком.

Абсолютно согласен. Нет такой методики, в которой это учитывается.
Я только начинаю это учитывать, во избежание казусов от нагрузки.

r9o-11 пишет:
 цитата:
наверное, ответа и нет - это же всё компромиссы, переходящие во вкусовщину...

Кому как. Я не вижу практического взаимодействия между Ri и Ra.

Не надо иметь много колес, лучше иметь много спиц, удерживающих одно колесо. © Спасибо: 0 
Профиль
Пермяк
moderator




Пост N: 6352
Зарегистрирован: 15.01.10
Откуда: Россия
Рейтинг: 24
ссылка на сообщение  Отправлено: 20.10.19 17:55. Заголовок: RedStar пишет: было..


RedStar пишет:
 цитата:
было мной приведено в удаленной ветке "Беседы ..." с

Ещё раз повторю: ничего Вы там не привели, одни бессвязные и лишённые смысла выкладки. Без пояснений, что делаете и для чего.

Спасибо: 0 
Профиль
RedStar
постоянный участник




Пост N: 2496
Зарегистрирован: 02.08.16
Откуда: Смоленск
Рейтинг: 1
ссылка на сообщение  Отправлено: 20.10.19 18:04. Заголовок: Ea = (Ro + Ra) x Io ..


Ea = (Ro + Ra) x Io
Крутизна будет от требуемой выходной амплитуды напряжения и тока. Из формулы видно как что на что влияет.
Мне все заново писАть? Увольте.

Не надо иметь много колес, лучше иметь много спиц, удерживающих одно колесо. © Спасибо: 0 
Профиль
Пермяк
moderator




Пост N: 6353
Зарегистрирован: 15.01.10
Откуда: Россия
Рейтинг: 24
ссылка на сообщение  Отправлено: 20.10.19 18:12. Заголовок: Ну галиматья же, в ч..



 цитата:
Ea = (Ro + Ra) x Io

Ну галиматья же, в чистом виде, прости Господи...

Спасибо: 0 
Профиль
RedStar
постоянный участник




Пост N: 2499
Зарегистрирован: 02.08.16
Откуда: Смоленск
Рейтинг: 1
ссылка на сообщение  Отправлено: 20.10.19 18:23. Заголовок: Пермяк пишет: Ну га..


Пермяк пишет:

 цитата:
Ну галиматья же, в чистом виде, прости Господи...


Обоснуйте доказательством противоположного. Вам и всем приводил расчетом по векторной математике.
Жду.

rv10, от вас требую вашего обстоятельного объяснения с согласием на пост Aleph
Вы читали мой ответ, прежде чем его повторить?

Не надо иметь много колес, лучше иметь много спиц, удерживающих одно колесо. © Спасибо: 0 
Профиль
Пермяк
moderator




Пост N: 6354
Зарегистрирован: 15.01.10
Откуда: Россия
Рейтинг: 24
ссылка на сообщение  Отправлено: 20.10.19 18:30. Заголовок: RedStar пишет: Обосн..


RedStar пишет:
 цитата:
Обоснуйте доказательством противоположного. Вам приводил расчетом по векторной математике.

Обосновывать должны Вы, поскольку пытаетесь доказать что-то своё. Причём с пояснениями, зачем делаете то-то и то-то.
А мне обосновывать незачем, всё, что нужно было, я обосновал Вам 4 года назад, когда Вы были под другим никнэймом.

Спасибо: 0 
Профиль
RedStar
постоянный участник




Пост N: 2500
Зарегистрирован: 02.08.16
Откуда: Смоленск
Рейтинг: 1
ссылка на сообщение  Отправлено: 20.10.19 18:40. Заголовок: Тот Ник заблокирован..


Тот Ник заблокирован. Ваш форум начал читать еще в 2005 году.
Мне вновь привести?:
KP=PL
KP=AC на оси "х"
PL=CB на оси "х"
KP=EF на оси "у"
PL=FD на оси "у"
Точка P делит отрезок KL пополам.


возьмем из первого графика:
H = (P + KL) x F
По второму рисунку выводится так:
x'/y'=z'
x/y=z
e = (x'/y')+(x/y)*y = (z'+z)*y
или:
Ea = (Ro + Ra) x Io



Или скажете, что здесь математика не верна? Здесь отрицаются какие либо законы?
П.С. Теперь жду вашего обоснования противоположного ответа с применением Ri.
П.С.2. Или Декарт не прав?

Не надо иметь много колес, лучше иметь много спиц, удерживающих одно колесо. © Спасибо: 0 
Профиль
Пермяк
moderator




Пост N: 6355
Зарегистрирован: 15.01.10
Откуда: Россия
Рейтинг: 24
ссылка на сообщение  Отправлено: 20.10.19 18:49. Заголовок: Вы опять за старое? ..


Вы опять за старое? Где пояснения? Каждая, подчёркиваю: КАЖДАЯ строка с матем.зависимостями, приведёнными Вами, должна сопровождаться пояснениями: для того-то и того-то берём такую-то формулу, взятую оттуда-то.
Что, Вас не учили этому?

А прежде всего надо пояснить коллективу, что за дисциплина "Векторная алгеба", и для чего предназначена.

Так что, Вам надо писАть статью, на форуме всё это изложить. Да, трудно...

Спасибо: 0 
Профиль
RedStar
постоянный участник




Пост N: 2501
Зарегистрирован: 02.08.16
Откуда: Смоленск
Рейтинг: 1
ссылка на сообщение  Отправлено: 20.10.19 18:52. Заголовок: Хорошо. Вот пояснени..


Хорошо. Вот пояснение.
По ординате:


По абсциссе:


Не надо иметь много колес, лучше иметь много спиц, удерживающих одно колесо. © Спасибо: 0 
Профиль
RedStar
постоянный участник




Пост N: 2502
Зарегистрирован: 02.08.16
Откуда: Смоленск
Рейтинг: 1
ссылка на сообщение  Отправлено: 20.10.19 18:53. Заголовок: Пермяк пишет: что з..


Пермяк пишет:

 цитата:
прежде всего надо пояснить коллективу, что за дисциплина "Векторная алгеба", и для чего предназначена.

http://kvm.gubkin.ru/vector.pdf

 цитата:
Вам надо писАть статью, на форуме всё это изложить

Согласен с вами, надо написать. Без плотно ухоженного состояния текста, вряд ли поймут.
Только не могу найти того, кто поможет грамотно составить изложение.

Не надо иметь много колес, лучше иметь много спиц, удерживающих одно колесо. © Спасибо: 0 
Профиль
Пермяк
moderator




Пост N: 6356
Зарегистрирован: 15.01.10
Откуда: Россия
Рейтинг: 24
ссылка на сообщение  Отправлено: 20.10.19 19:07. Заголовок: rv10 пишет: rv10 пи..


rv10 пишет:
 цитата:
нет никакой разницы между переменым и постоянным сигналом при усилении его лампой

Uо и Iо - не сигнал, который требуется усилить, а установленный исходный режим ДО ПОЯВЛЕНИЯ управляющего сигнала, рабочий режим.
У Вас плохо с пониманием даташитов? Я ведь именно из даташита выложил скан (пост 6349).

Спасибо: 0 
Профиль
rv10



Пост N: 295
Зарегистрирован: 23.09.19
Рейтинг: -1
ссылка на сообщение  Отправлено: 20.10.19 19:08. Заголовок: RedStar Короче варит..


RedStar пишет:
 цитата:
Постоянный ток не имеет частоту. Вы о чем вообще

конечно не имеет но её имеет меандр который и состоит из отрезков постоянного тока. представьте периодичность импульсов час, представили полка импульса длительностью час.
 цитата:
Только не могу найти того, кто поможет грамотно составить изло

что и требовалось доказать.

Пермяк, я понимаю о чем Вы, но я рассматриваю лампу и как усилитель постоянного тока, а это есть не что иное как установленный режим, или я ошибаюсь?

Turn on, tune in, drop out Спасибо: 0 
Профиль
RedStar
постоянный участник




Пост N: 2506
Зарегистрирован: 02.08.16
Откуда: Смоленск
Рейтинг: 1
ссылка на сообщение  Отправлено: 20.10.19 19:13. Заголовок: Послушайте, rv10, ес..


Послушайте, rv10, если многого до вас не доходит, зачем пытаетесь казаться многозадачным без практических работ?
У меня все основывается на практике.
Попробуйте опровергнуть!? У вас есть практические работы?

Сейчас физики-математики не в состоянии переварить такие данные в отсутствии знаний в этой области. И не только в этой, а и в общем.
Какое обучение сейчас, такие и люди.

rv10 пишет:
 цитата:
имеет меандр который и состоит из отрезков постоянного тока.

А где вы видели музыкальный сигнал (гармонический) с такой периодичностью?
Это больше к вычислительной технике подходит, где Ri требуется. Ха.

Не надо иметь много колес, лучше иметь много спиц, удерживающих одно колесо. © Спасибо: 0 
Профиль
rv10



Пост N: 297
Зарегистрирован: 23.09.19
Рейтинг: -1
ссылка на сообщение  Отправлено: 20.10.19 19:16. Заголовок: RedStar Примените св..


RedStar Примените свою теорию для УПТ (усилитель постоянного тока).

Turn on, tune in, drop out Спасибо: 0 
Профиль
RedStar
постоянный участник




Пост N: 2507
Зарегистрирован: 02.08.16
Откуда: Смоленск
Рейтинг: 1
ссылка на сообщение  Отправлено: 20.10.19 19:20. Заголовок: rv10 пишет: Примени..


rv10 пишет:
 цитата:
Примените свою теорию для УПТ (усилитель постоянного тока).

Надо подумать. И вы хотите сказать, что я не знаю Ri?
Прежде, чем писать подобное, определитесь, что имею в виду. Постоянные напряжения или переменные.

Не надо иметь много колес, лучше иметь много спиц, удерживающих одно колесо. © Спасибо: 0 
Профиль
Пермяк
moderator




Пост N: 6357
Зарегистрирован: 15.01.10
Откуда: Россия
Рейтинг: 24
ссылка на сообщение  Отправлено: 20.10.19 19:20. Заголовок: Ошибаетесь. Посудите..


rv10, ошибаетесь.
Посудите сами: если входной сигнал неизменный, тогда зачем его усиливать? Другое дело - сигнал постоянного напряжения, который может изменяться, но - мееедленно...
Тогда рабочая, исходная точка всегда одна, а медленно изменяющийся сигнал изменяется относительно этой точки, которая неизменна, это точка отсчёта, опорная точка, и в каскаде УПТ даже жёстко застабилизирована.

Спасибо: 0 
Профиль
rv10



Пост N: 301
Зарегистрирован: 23.09.19
Рейтинг: -1
ссылка на сообщение  Отправлено: 20.10.19 20:59. Заголовок: Пермяк пишет: rv10,..


Пермяк пишет:
 цитата:
rv10, ошибаетесь.
Посудите сами: если входной сигнал неизменный, тогда зачем его усиливать? Другое дело - сигнал постоянного напряжения, который может изменяться, но - мееедленно.

Вы уклоняетесь от признания факта что усилитель постоянного тока или напряжения то есть УПТ, это усилитель именно постоянного тока или напряжения. допустим на входе имеем 10 вольт или 10 миллиампер, а на выходе имеем 100 вольт или 100 миллиампер. Всё. Никаких медленно изменяющихся сигналов нет.

Плюмбум пишет:
 цитата:
Ну,дела! Оказывается,Равен не знает, что такое рабочая точка? Не знает, что для усиления обеих волн переменного тока лампа должна быть наполовину открыта?

Ошибаетесь, я знаю это, это вещь очевидная. Но разговор не об этом а о усилении постоянного напряжения, то есть неизменного сигнала: на входе 1 вольт на выходе 10, на входе 2 вольта на выходе соответственно 20.

Однотактный усилитель постоянного тока:click here, click here

Turn on, tune in, drop out Спасибо: 0 
Профиль
RedStar
постоянный участник




Пост N: 2514
Зарегистрирован: 02.08.16
Откуда: Смоленск
Рейтинг: 1
ссылка на сообщение  Отправлено: 20.10.19 22:16. Заголовок: rv10 пишет: Любой с..


rv10 пишет:
 цитата:
Любой сколь угодно медленно меняющийся сигнал есть ток переменный.

По сути, верно. Только надо смотреть, что является переменным, а что постоянным сигналом на входе.

Однако, вы все, несомненно, понимаете о чем пытаюсь толковать.
Только математика расставит все по местам.
Так что придется искать помощь в написании статьи.

Не надо иметь много колес, лучше иметь много спиц, удерживающих одно колесо. © Спасибо: 0 
Профиль
Ученик



Пост N: 872
Зарегистрирован: 06.04.13
Рейтинг: 1
ссылка на сообщение  Отправлено: 21.10.19 04:31. Заголовок: Странное желание - п..


RedStar пишет:
 цитата:
придется искать помощь в написании статьи.

Странное желание - писать статьи о вещах, которые не понимаешь.

Учиться никогда не поздно, и никому не вредно. Спасибо: 0 
Профиль
Кузьмич



Пост N: 1404
Зарегистрирован: 21.11.15
Рейтинг: 8
ссылка на сообщение  Отправлено: 21.10.19 13:56. Заголовок: RedStar пишет: Одна..


RedStar пишет:
 цитата:
Однако, вы все, несомненно, понимаете о чем пытаюсь толковать.

Ну да, мы все тут экстрасенсы...
 цитата:
в написании статьи.

Пепец! С ума сойти!

Спасибо: 0 
Профиль
RedStar
постоянный участник




Пост N: 2518
Зарегистрирован: 02.08.16
Откуда: Смоленск
Рейтинг: 1
ссылка на сообщение  Отправлено: 21.10.19 14:12. Заголовок: Кузьмич пишет: Пепе..


Кузьмич пишет:
 цитата:
Пепец! С ума сойти!

Кто вам запрещает, сходите себе тихо.
Кто приведет доказательное опровержение моего расчета или его несостоятельность?

Не надо иметь много колес, лучше иметь много спиц, удерживающих одно колесо. © Спасибо: 0 
Профиль
RedStar
постоянный участник




Пост N: 2519
Зарегистрирован: 02.08.16
Откуда: Смоленск
Рейтинг: 1
ссылка на сообщение  Отправлено: 21.10.19 15:27. Заголовок: RedStar пишет: Кто ..


RedStar пишет:
 цитата:
Кто приведет доказательное опровержение моего расчета или его несостоятельность?

Это часть для расчета драйверного (входного) каскада. И это, отчасти, подходит для выходного каскада. Только есть дополнения объединяющее с расчетом трансформатора, которые не буду приводить.
Причина проста, - ваше нежелание просто выслушать. У вас же всё переходит на оскорбление и на книжный указ.
Сомневаюсь в ответе от "светил" форумчан.


Толком никто здесь не написал: что же такое Ri?
И почему нет точной формулы Ra из Ri?


Не надо иметь много колес, лучше иметь много спиц, удерживающих одно колесо. © Спасибо: 0 
Профиль
Ученик



Пост N: 873
Зарегистрирован: 06.04.13
Рейтинг: 1
ссылка на сообщение  Отправлено: 22.10.19 02:59. Заголовок: RedStar пишет: Кто п..


RedStar пишет:
 цитата:
Кто приведет доказательное опровержение моего расчета или его несостоятельность?

Предлагаешь опровергать это?



RedStar пишет:
 цитата:
И почему нет точной формулы Ra из Ri?

А ещё глупее вопрос сможешь придумать? Чтобы определить хотя бы, где твой край непонимания.

Учиться никогда не поздно, и никому не вредно. Спасибо: 1 
Профиль
Кузьмич



Пост N: 1413
Зарегистрирован: 21.11.15
Рейтинг: 8
ссылка на сообщение  Отправлено: 22.10.19 03:11. Заголовок: RedStar пишет: Заст..


RedStar пишет:
 цитата:
Застрелись, если не по твоему думаю (не научно).

Да по мне - думай как хочешь, здесь чушь не пиши...
 цитата:
И почему нет точной формулы Ra из Ri?

Формула есть. Нет точной - да. Ну а ты сам можешь подумать почему?
Ведь в учебнике всё расписано. Да и тебе неоднократно об этом говорили.
Хорошо, скажу ещё раз. Знаю наперёд, что придумаешь очередную чушь...
Потому, что мы сами выбираем, исходя из того, что нам нужно.
Для триодов: Нужна бОльшая мощность и плевать на искажения
Ra опт. = 2Ri Чёткая, конкретная формула! (8.26) Дедушка Цыкин, стр. 456.
Но при этом триод имеет значительные искажения. И если они не устраивают,
(а в SE УНЧ так и есть), то увеличивают коэффициент нагрузки, до получения приемлемой мощности и приемлемых искажений.
Для пентодов такой вариант не подходит. И Ra = (0,1 - 0,5)Ri для пентодов писАлось в научно - популярной литературе, для пионЭров, т.е для начинающих. В учебниках ничего подобного не найдёшь. Там по другому...

Спасибо: 0 
Профиль
Raven network



Пост N: 1
Зарегистрирован: 22.10.19
Рейтинг: 0
ссылка на сообщение  Отправлено: 22.10.19 05:07. Заголовок: Вот про RI доступно ..


Вот про Ri доступно (в динамическом режиме при переменных составляющих) и понятно,и соответствует тематике и названию ветки, а то развели демагогию.
RedStar, тебе - в первую очередь прочитать.

Спасибо: 0 
Профиль
RedStar
постоянный участник




Пост N: 2533
Зарегистрирован: 02.08.16
Откуда: Смоленск
Рейтинг: 1
ссылка на сообщение  Отправлено: 22.10.19 13:46. Заголовок: Подсчитаем по ВАХ: h..


Подсчитаем по ВАХ: http://hiend.borda.ru/?1-1-1571722521361-00000561-000-0-0#002
Какое внутреннее при смещении от -1,5 до -2,0?
Какая будет нагрузка по "Чёткой, конкретной формуле"? При условии 330 вольт питания.

П.С. Неужели нельзя понять, что я сравниваю. Считаю, собираю, налаживаю и проверяю совместимость.
Это некорректно и не научно по вашему?

Не надо иметь много колес, лучше иметь много спиц, удерживающих одно колесо. © Спасибо: 0 
Профиль
Кузьмич



Пост N: 1414
Зарегистрирован: 21.11.15
Рейтинг: 8
ссылка на сообщение  Отправлено: 22.10.19 14:55. Заголовок: RedStar пишет: Это ..


RedStar пишет:
 цитата:
Это некорректно и не научно по вашему?

Некорректно выставлять сюда вот такую чушь:



, и потом артачиться на замечания.
 цитата:
Какая будет нагрузка по "Чёткой, конкретной формуле"?

Опять ты за своё. Нет такой чёткой и конкретной формулы. И не будет.
Существующая формула предоставляет выбор разработчику в зависимости от желаемого им усиления и искажений.

Спасибо: 0 
Профиль
RedStar
постоянный участник




Пост N: 2534
Зарегистрирован: 02.08.16
Откуда: Смоленск
Рейтинг: 1
ссылка на сообщение  Отправлено: 22.10.19 15:29. Заголовок: Чушь, да. Коряво нап..


Чушь, да. Коряво написАл. Так поправьте, а не ссыпьте оскорблениями.
Сложно?

Не надо иметь много колес, лучше иметь много спиц, удерживающих одно колесо. © Спасибо: 0 
Профиль
Кузьмич



Пост N: 1415
Зарегистрирован: 21.11.15
Рейтинг: 8
ссылка на сообщение  Отправлено: 22.10.19 15:38. Заголовок: RedStar пишет: Слож..


RedStar пишет:
 цитата:
Сложно?

Не то слово. Потому что нормальному человеку невозможно понять, что ты там изобразил.
Нормальный человек по праву считает ЭТО чушью.
И это не оскорбление. Чушь - она и в Африке чушь.

Спасибо: 0 
Профиль
RedStar
постоянный участник




Пост N: 2535
Зарегистрирован: 02.08.16
Откуда: Смоленск
Рейтинг: 1
ссылка на сообщение  Отправлено: 22.10.19 15:39. Заголовок: Кузьмич, это ваша фр..


Кузьмич, это ваша фраза была про "Четкое...".
Кузьмич пишет:
 цитата:
Существующая формула предоставляет выбор разработчику в зависимости от желаемого усиления и искажений.

Этот выбор - долгий поиск. Так прошу помочь, как вы считаете. От и до.
Сопоставлю данные. Макет готов для эксперимента.

Не надо иметь много колес, лучше иметь много спиц, удерживающих одно колесо. © Спасибо: 0 
Профиль
Aleph





Пост N: 601
Зарегистрирован: 03.06.15
Откуда: Макеевка
Рейтинг: 7
ссылка на сообщение  Отправлено: 22.10.19 15:41. Заголовок: RedStar, а покажите ..


RedStar, а покажите нам с нуля как Вы рассчитываете каскад с резистивной нагрузкой. Какие величины Вы используете. От чего отталкиваетесь. Возьмём для примера 6ж5п в пентодном включении. Неужели для этого нужна векторная алгебра?
Мне допустим не важно что там написано в учебнике про нагрузку, я её выбираю с линейкой по ВАХам исходя из моего питания. А величина резистора сама по закону Ома выходит

С ув. Сергей Спасибо: 0 
Профиль
Кузьмич



Пост N: 1416
Зарегистрирован: 21.11.15
Рейтинг: 8
ссылка на сообщение  Отправлено: 22.10.19 16:24. Заголовок: RedStar пишет: Этот..


RedStar пишет:
 цитата:
Этот выбор - долгий поиск. Так прошу помочь, как вы считаете. От и до.

Не долгий. И он многократно короче, чем заниматься векторной алгеброй.

Вот тут

всё расписано доступным русским языком. Читай и запоминай.

Спасибо: 0 
Профиль
RedStar
постоянный участник




Пост N: 2536
Зарегистрирован: 02.08.16
Откуда: Смоленск
Рейтинг: 1
ссылка на сообщение  Отправлено: 22.10.19 16:48. Заголовок: Aleph пишет: покажи..


Aleph пишет:
 цитата:
покажите нам с нуля как Вы рассчитываете каскад с резистивной нагрузкой.

Охотно расскажу, если интересно вам, но позже.
Хотелось бы увидеть сначала расчет от Ri.

П.С. Глядя на ВАХ 6ж5п, с нее надо попробовать выжать 100 вольт выходной амплитуды. Было бы не плохо.

Не надо иметь много колес, лучше иметь много спиц, удерживающих одно колесо. © Спасибо: 0 
Профиль
Кузьмич



Пост N: 1418
Зарегистрирован: 21.11.15
Рейтинг: 8
ссылка на сообщение  Отправлено: 22.10.19 18:03. Заголовок: RedStar пишет: Хоте..


RedStar пишет:
 цитата:
Хотелось бы увидеть сначала расчет от Ri.

Нет такого расчёта от Ri. В природе не существует.
Есть расчёт УН, УМ (реостатные, трансформаторные, дроссельные...),
в которых обязательно фигурирует Ri.
Смотрим Войшвилло 1963 год "Усилители низкой частоты на электронных лампах"
Например со стр. 453 и далее...



Спасибо: 0 
Профиль
RedStar
постоянный участник




Пост N: 2541
Зарегистрирован: 02.08.16
Откуда: Смоленск
Рейтинг: 0
ссылка на сообщение  Отправлено: 24.10.19 00:45. Заголовок: Кузьмич пишет: Нет ..


Кузьмич пишет:
 цитата:
Нет такого расчёта от Ri. В природе не существует.

Так зачем же упираетесь, что из него получаете Ra?
В одном могу согласиться. Оно хорошо, когда выбираешь Статические параметры, но не динамические с переменными величинами.
Знаете, различные "допущения" касательно расчетов, приведенных в книгах, считаю не допустимыми.
И, как написано у Войшвилло: "...зависят от координат начальной точки". Это много значит! (штудирую этот параграф от и до).

П.С. Обещал изложить Алефу мой расчет, время идет, а ответа по Обязательному применению Ri с обоснованием так и не увидел.
Что же, пока подожду. Но обязательно покажу, если позволят.

Не надо иметь много колес, лучше иметь много спиц, удерживающих одно колесо. © Спасибо: 0 
Профиль
Кузьмич



Пост N: 1419
Зарегистрирован: 21.11.15
Рейтинг: 8
ссылка на сообщение  Отправлено: 24.10.19 03:03. Заголовок: RedStar пишет: Так ..


RedStar пишет:
 цитата:
Так зачем же упираетесь, что из него получаете Ra?

А из чего же??? Задаёмся коэффициентом нагрузки А (Альфа) и считаем Ra.
Ra = A * Ri
(предварительно должны выбрать РТ на ВАХ и определить Ri).
На основании полученного Ra, на ВАХах рисуем нагрузочную (динамическую) через РТ, по которой вычисляем мощность и нелинейные искажения. Это главное, от чего идёт расчёт УНЧ.
Не от Ri, а от желаемой мощности и допустимых нелинейных искажений. А ещё... КПД.
И если полученные данные удовлетворяют заданным (задуманным), то оставляем...
Если нет, меняем рабочую точку, уточняем Ri, и или меняем Альфу, уточняем Ra, рисуем, считаем...
Или выбираем другую лампу и всё заново. Это так называемый классический расчёт.

В радиолюбительстве бывает несколько иначе. Есть лампа, её хочу... Сколько даст мощности, столько даст.
А нелинейных чаще хотят получить поменьше.

Всё выше сказанное - применительно к триодам. При определении оптимального Ra для пентодов Ri обычно не учитывают. Там несколько иначе.

RedStar пишет:
 цитата:
как написано у Войшвилло: "...зависят от координат начальной точки". Это много значит! (штудирую этот параграф от и до) ...

Правильно. От координат РТ (U0 и I0) много чего зависит. И их надо выбрать правильно.
Подозреваю, ты собрался сюда впихнуть векторную алгебру:
 цитата:
"допущения" касательно расчетов, приведенных в книгах, считаю недопустимыми.

Это от твоего низкого образовательного уровня. Ты не понимаешь сути допущений.
 цитата:
а ответа по Обязательному применению Ri с обоснованием так и не увидел.

Ri необходимо для расчёта некоторых параметров усилителя (каскада), на стадии его проектирования.
Самодельщикам-кустарщикам этот параметр зачастую не нужен. Да они про него просто и не знают. Как и ты.

Спасибо: 0 
Профиль
Пермяк
moderator




Пост N: 6375
Зарегистрирован: 15.01.10
Откуда: Россия
Рейтинг: 24
ссылка на сообщение  Отправлено: 24.10.19 09:53. Заголовок: Кузьмич, судя по все..


Кузьмич, судя по всему, Редстар в этой теме пишет о резистивном каскаде. А мы ему рассказываем про трансформаторный...

Спасибо: 0 
Профиль
r9o-11





Пост N: 591
Зарегистрирован: 28.01.15
Откуда: Искитим, НСО
Рейтинг: 3
ссылка на сообщение  Отправлено: 24.10.19 11:08. Заголовок: Так разве основной з..


Так разве основной задачей резистивного каскада является передача мощности в следующий каскад? Это же совсем не схемотехника транзисторных ВЧ усилителей мощности с их "омными" входными сопротивлениями...
Зачем в предварительных каскадах альфу считать?

Андрей Спасибо: 0 
Профиль
Кузьмич



Пост N: 1421
Зарегистрирован: 21.11.15
Рейтинг: 8
ссылка на сообщение  Отправлено: 24.10.19 14:16. Заголовок: Пермяк пишет: судя ..


Пермяк пишет:
 цитата:
судя по всему, Редстар в этой теме пишет о резистивном каскаде.

Принцип определения нагрузки триода в резистивном каскаде, мало чем отличается.
И сводится к достижению максимального усиления сигнала и допустимых частотных искажений на ВЧ.
При вычислении Ra~ необходимо учитывать Rc следующего каскада, т.к. Ra и Rc соединены параллельно.
Учебники "рекомендуют" нам выбирать Ra для триода не более (5 - 10)Ri.
П.С.
Краснозвёзному, по незнанию, не нравится такое допущение. Ему надо, что бы было строго, например 3,75Ri...
Кажется так.
Расчёт нагрузки для пентодов производится несколько иначе...

Спасибо: 0 
Профиль
RedStar
постоянный участник




Пост N: 2544
Зарегистрирован: 02.08.16
Откуда: Смоленск
Рейтинг: 0
ссылка на сообщение  Отправлено: 24.10.19 18:44. Заголовок: Кузьмич пишет: При ..


Кузьмич пишет:
 цитата:
При вычислении Ra~ необходимо учитывать Rc следующего каскада, т.к. Ra и Rc соединены параллельно.

Само собой. Всегда при макетировании ставлю Rc следующего каскада.
Только вот неувязочка получается. Если они параллельно, то суммарное сопротивление должно быть много меньше. А этого нет.
Ra*Rc / Ra+Rc - верно? Увы, не получается так.
 цитата:
Ему надо, чтобы было строго, например 3,75Ri...

Да, так и надо. Может быть 2,857 или 3,2144? Когда точнее рассчитываю, меньше Кг%.
Для пентодов, верно, иначе. Но суть то остается.
Плохого учебники не пишут. Но все они ориентированы временем становления ламповой техники, где питание было кенотронное.
Все последующие, - лишь копия старых книг с незначительными дополнениями.

Не надо иметь много колес, лучше иметь много спиц, удерживающих одно колесо. © Спасибо: 0 
Профиль
Ученик



Пост N: 875
Зарегистрирован: 06.04.13
Рейтинг: 1
ссылка на сообщение  Отправлено: 24.10.19 19:22. Заголовок: RedStar пишет: Если ..


RedStar пишет:
 цитата:
Если они параллельно, то суммарное сопротивление должно быть много меньше.

Какое такое "суммарное" сопротивление?
Почему "много меньше"? У тебя Ra и Rc одного порядка, что ли?
 цитата:
Ra*Rc / Ra+Rc

А что ты так считаешь - то?
 цитата:
Да, так и надо.

Не занимайся ерундой. Никогда Ra не выбиралась и не будет выбираться в каком-то строгом соотношении к Ri.
Почему - тебе писали много раз, но ты же не читатель...
 цитата:
Когда точнее рассчитываю, меньше Кг%

Чего и как ты рассчитываешь - тайна за семью печатями. Оформить свои изыскания в приемлемую форму ты не в состоянии.
P.S. Какая может быть векторная алгебра? У тебя с арифметикой нелады.

Учиться никогда не поздно, и никому не вредно. Спасибо: 0 
Профиль
Кузьмич



Пост N: 1425
Зарегистрирован: 21.11.15
Рейтинг: 8
ссылка на сообщение  Отправлено: 24.10.19 19:43. Заголовок: Ученик пишет: Какое..


Ученик пишет:
 цитата:
Какое такое "суммарное" сопротивление?

Это смоленский диалект, понимать надо...

RedStar
 цитата:
меньше Кг%

Тогда бери 10 - 15, 20 Ri. Или ещё больше, коль только Кг% интересует.
Но не плачь, когда столкнёшься с другими трудностями.

Спасибо: 0 
Профиль
RedStar
постоянный участник




Пост N: 2545
Зарегистрирован: 02.08.16
Откуда: Смоленск
Рейтинг: 0
ссылка на сообщение  Отправлено: 25.10.19 17:58. Заголовок: Кузьмич пишет: Тогд..


Кузьмич пишет:
 цитата:
Тогда бери 10 - 15, 20 Ri.

Зачем? Там питание будет велико.

Тогда из каких критериев исходить при расчете?
От имеющегося питания или, все же от РТ, где потом питание делать?

Не надо иметь много колес, лучше иметь много спиц, удерживающих одно колесо. © Спасибо: 0 
Профиль
Кузьмич



Пост N: 1426
Зарегистрирован: 21.11.15
Рейтинг: 8
ссылка на сообщение  Отправлено: 25.10.19 19:06. Заголовок: RedStar пишет: Тогд..


RedStar пишет:
 цитата:
Тогда из каких критериев исходить при расчете?

Ну я же тебе всё дал в ссылках. Читай, запоминай, выписывай...



По поводу искажений. Во многих случаях примерно до 5% искажений ты можешь даже и не заметить, что они там есть.
Зависит от спектра искажений.

Спасибо: 0 
Профиль
aleks8845



Пост N: 611
Зарегистрирован: 07.03.16
Рейтинг: 1
ссылка на сообщение  Отправлено: 25.10.19 23:48. Заголовок: Кузьмич пишет: Зави..


Кузьмич пишет:
 цитата:
Зависит от спектра искажений.

Тут - в точку, спектр очень важен. Малейшее превышение 3й гармоники относительно 2й или наравне, иль вообще больше 2й - звук начинает «подвизгивать»...

Спасибо: 0 
Профиль
RedStar
постоянный участник




Пост N: 2546
Зарегистрирован: 02.08.16
Откуда: Смоленск
Рейтинг: 0
ссылка на сообщение  Отправлено: 26.10.19 00:01. Заголовок: Кузьмич пишет: Ну я..


Кузьмич пишет:
 цитата:
Ну я же тебе всё дал в ссылках.

Это для трансформаторного. Хотя для резистивного тоже подходит 2...10*Ri.
А как быть, если ограничено питание некой величиной и из нее надо строить нагрузочную?
Перебирать 2...10? Как-то напряжённо.
А в случае с трансформатором, - перематывать многократно?

Допустим, зная питание и выбрав на графике приемлемую РТ для резистивного каскада, как же можно точнее рассчитать со статическим данными на лампу?
Причем отдельно, триод и пентод.

Не надо иметь много колес, лучше иметь много спиц, удерживающих одно колесо. © Спасибо: 0 
Профиль
Aleph





Пост N: 602
Зарегистрирован: 03.06.15
Откуда: Макеевка
Рейтинг: 7
ссылка на сообщение  Отправлено: 26.10.19 00:01. Заголовок: RedStar пишет: Да, ..


RedStar пишет:
 цитата:
Да, так и надо. Может быть 2,857 или 3,2144? Когда точнее рассчитываю, меньше Кг%.
Для пентодов, верно, иначе. Но суть остаётся.

Толку в этих точных расчётах, если ВАХи - усредненные, и очень часто снятые собственноручно выглядят покривее справочных. Даже если снимете ВАХ для своей лампы, в течение службы она меняет свои параметры. Кроме того, параметры плавают вместе с напряжением смещенеия (или Вы все питания стабилизируете с точностью до трех знаков? А что со сменой отслужившей лампы? Под нее потом новый анодный резистор подбираете, тоже с точностью до 3 знаков?

Читал недавно, что есть такое заболевание, когда даже легкая непараллельность столовых приборов, или скажем на сантиметр короче галстук, выводят из себя до нервного срыва. Кажется, любовь к излишне точным цифрам тоже туда относится

С ув. Сергей Спасибо: 1 
Профиль
RedStar
постоянный участник




Пост N: 2547
Зарегистрирован: 02.08.16
Откуда: Смоленск
Рейтинг: 0
ссылка на сообщение  Отправлено: 26.10.19 00:11. Заголовок: Aleph пишет: Толку ..


Aleph пишет:
 цитата:
Толку в этих точных расчетах если ВАХи усредненные

Не скажите. Даже если ВАХ усредненнные значительно, то совсем небольшой подстройкой автоматическим смещением или фиксированным, всегда можно "выправить" нелинейность.
 цитата:
анодный резистор подбираете, тоже с точностью до 3 знаков?

Зачем же? По расчетным собираю иногда сборку. К примеру, надо 6245, ставлю 6200. Или надо 15300, - ставлю подбором тестером ближайшие к этому сопротивлению.
 цитата:
Кажется, любовь к излишне точным цифрам тоже туда относится

Эти цифры не лишние. Как правило - сразу удается получить требуемое.

Не надо иметь много колес, лучше иметь много спиц, удерживающих одно колесо. © Спасибо: 0 
Профиль
Пермяк
moderator




Пост N: 6378
Зарегистрирован: 15.01.10
Откуда: Россия
Рейтинг: 24
ссылка на сообщение  Отправлено: 26.10.19 00:42. Заголовок: RedStar Хм, ратуете ..


RedStar
Хм, ратуете за точность?
Какими бы точными ни были ВАХи, Вы ведь в своих схемах радикально их изменяете:
- в резистивном драйвере - применением питания экр. сетки с части анодной нагрузки (как бы УЛ);
- в выходном каскаде - применением КО и ЭО.
И ничего заранее вычислить невозможно. Даже с помощью векторной алгебры. Только утомительным подбором :)

Спасибо: 0 
Профиль
RedStar
постоянный участник




Пост N: 2548
Зарегистрирован: 02.08.16
Откуда: Смоленск
Рейтинг: 0
ссылка на сообщение  Отправлено: 26.10.19 01:59. Заголовок: Пермяк пишет: ратуе..


Пермяк пишет:
 цитата:
ратуете за точность?
... ничего заранее вычислить невозможно.

Верно, ратую. И вполне с этим справляюсь.
Все же можно. Что в резистивном, что в трансформаторном. А эта часть векторной алгебры как раз нужна была для основы.
Совсем близко добрался до вычисления ЭО. Немного додумать и сопоставить, только по времени долго придется работать с макетированием.

Не надо иметь много колес, лучше иметь много спиц, удерживающих одно колесо. © Спасибо: 0 
Профиль
Пермяк
moderator




Пост N: 6379
Зарегистрирован: 15.01.10
Откуда: Россия
Рейтинг: 24
ссылка на сообщение  Отправлено: 26.10.19 11:20. Заголовок: RedStar пишет: эта ..


RedStar пишет:
 цитата:
эта часть векторной алгебры как раз нужна была для основы.

Мало пояснена эта часть (кстати, которая?), надо подробнее пояснять, я Вам уже советовал...

Спасибо: 0 
Профиль
RedStar
постоянный участник




Пост N: 2550
Зарегистрирован: 02.08.16
Откуда: Смоленск
Рейтинг: 0
ссылка на сообщение  Отправлено: 26.10.19 19:02. Заголовок: Которая расписана, и..


Которая расписана, и довольно подробно: http://hiend.borda.ru/?1-22-1572087598365-00000544-000-80-0#041
Больше добавить нечего, и нет желания из-за неадекватности некоторых форумчан, так как оскорбления принимаю всерьез.
Больше рассказывать о своем методе не буду.
Варитесь дальше в прошлом.

Не надо иметь много колес, лучше иметь много спиц, удерживающих одно колесо. © Спасибо: 0 
Профиль
Ученик



Пост N: 879
Зарегистрирован: 06.04.13
Рейтинг: 2
ссылка на сообщение  Отправлено: 26.10.19 23:30. Заголовок: RedStar пишет: Котор..


RedStar пишет:
 цитата:
Которая расписана, и довольно подробно

А что там является целью вычислений? Напряжение анодного источника и сопротивление нагрузки?

Учиться никогда не поздно, и никому не вредно. Спасибо: 0 
Профиль
RedStar
постоянный участник




Пост N: 2553
Зарегистрирован: 02.08.16
Откуда: Смоленск
Рейтинг: 0
ссылка на сообщение  Отправлено: 27.10.19 00:08. Заголовок: Абстрагирование от R..


Абстрагирование от Ri.

Какая эквивалентная схема является последовательной (Ri+Ra), а какая параллельной? Резистивная или трансформаторная?

Не надо иметь много колес, лучше иметь много спиц, удерживающих одно колесо. © Спасибо: 0 
Профиль
Ученик



Пост N: 881
Зарегистрирован: 06.04.13
Рейтинг: 2
ссылка на сообщение  Отправлено: 27.10.19 00:20. Заголовок: RedStar пишет: Абстр..


RedStar пишет:
 цитата:
Абстрагирование от Ri.

Хм...оно и при обычном расчёте не является обязательной отправной точкой. Уж для резистивного - точно. Да и для трансформаторного его учёт не всегда обязателен.
Я вот вижу, что в начале расчёта тебе известны точки K, P и L. Откуда ты взял, или предполагаешь брать эти данные?
 цитата:
Какая эквивалентная схема является последовательной (Ri+Ra), а какая параллельной? Резистивная или трансформаторная?

Смотря что тебя интересует. Если выходное сопротивление каскада - параллельная. А если, например, влияние пульсаций анодного источника - последовательная.

Учиться никогда не поздно, и никому не вредно. Спасибо: 0 
Профиль
RedStar
постоянный участник




Пост N: 2554
Зарегистрирован: 02.08.16
Откуда: Смоленск
Рейтинг: 0
ссылка на сообщение  Отправлено: 27.10.19 01:02. Заголовок: Так зачем же "от..


Так зачем же "отправной точкой" используете Ri для вычислений, что резистивных, что тран-ных?
Включая Альфу и примерные значения от 2 до 10 (или 0,1-0,5) Ri.

Ученик пишет:
 цитата:
в начале расчёта тебе известны точки K, P и L. Откуда ты взял, или предполагаешь брать эти данные?

Известна только Рабочая точка, которую выбираем. Остальное зависит от требуемой выходной амплитуды.
 цитата:
Если выходное сопротивление каскада - параллельная. А если, например, влияние пульсаций анодного источника - последовательная.

Это как? В транс-ном - последовательная? Нет?

Не надо иметь много колес, лучше иметь много спиц, удерживающих одно колесо. © Спасибо: 0 
Профиль
Кузьмич



Пост N: 1427
Зарегистрирован: 21.11.15
Рейтинг: 8
ссылка на сообщение  Отправлено: 27.10.19 01:45. Заголовок: RedStar пишет: (ил..


RedStar пишет:
 цитата:
(или 0,1-0,5) Ri

А это что? И откуда взято???
 цитата:
Это как? В транс-ном - последовательная? Нет?

По переменному току верхний "конец" по схеме анодного резистора, или верхний "конец" обмотки трансформатора "сидит"
на общем проводе через конденсатор большой ёмкости БП, т.е. параллельно лампе, т.е. её внутреннему сопротивлению Ri.
А этот конденсатор большой ёмкости БП на НЧ( не говоря уже для более высоких частот) имеет сопротивление близкое к нулю.

Спасибо: 0 
Профиль
RedStar
постоянный участник




Пост N: 2555
Зарегистрирован: 02.08.16
Откуда: Смоленск
Рейтинг: 0
ссылка на сообщение  Отправлено: 27.10.19 02:00. Заголовок: Кузьмич пишет: Дожи..


Кузьмич пишет:
 цитата:
Дожили...

Покажите пример. (не ради забавы).
То, что:
Кузьмич пишет:
 цитата:
По переменному току верхний "конец" по схеме анодного резистора, или верхний "конец" обмотки трансформатора "сидит" на общем проводе через конденсатор большой ёмкости БП,

Естественно. А между транс-ром (нижний вывод тр-ра) и лампой (анодом) есть что либо? Считаете это параллельным?

Добавлю:
"...ток пойдет по цепи из последовательно соединенного га с параллельной ветвью RL и LP ."
http://www.vestnikara.spb.ru/vestn/n3/moir.htm

Не надо иметь много колес, лучше иметь много спиц, удерживающих одно колесо. © Спасибо: 0 
Профиль
Кузьмич



Пост N: 1428
Зарегистрирован: 21.11.15
Рейтинг: 8
ссылка на сообщение  Отправлено: 27.10.19 02:41. Заголовок: RedStar пишет: Пока..


RedStar пишет:
 цитата:
Покажите пример. (не ради забавы).

Цыкин, раздел 5,3 Трансформаторный каскад.
Стр. 132 и далее.
 цитата:
А между транс-ром (нижний вывод тр-ра) и лампой (анодом) есть что либо?

Есть! Твоя любимая амплитуда, которую надо втиснуть куда-то далее.
Ещё раз спрашиваю:
Откуда это взято: RedStar пишет:
 цитата:
(или 0,1-0,5) Ri.

???

Спасибо: 0 
Профиль
Ученик



Пост N: 883
Зарегистрирован: 06.04.13
Рейтинг: 2
ссылка на сообщение  Отправлено: 27.10.19 03:19. Заголовок: RedStar пишет: Так з..


RedStar пишет:
 цитата:
Так зачем же "отправной точкой" используете Ri для вычислений

Кто использует????
 цитата:
Известна только Рабочая точка, которую выбираем. Остальное зависит от требуемой выходной амплитуды.

Рабочая точка - это одна точка. У тебя - три. Объясни, откуда ты берёшь точки K и L.
 цитата:
Это как? В транс-ном - последовательная?

Хоть в каком. Выше написал: если рассматриваешь влияние пульсаций анодного источника, по простому - фон от этих пульсаций, то здесь рассматривается как обычный делитель: Rа включена с Ri последовательно.

Учиться никогда не поздно, и никому не вредно. Спасибо: 0 
Профиль
Пермяк
moderator




Пост N: 6381
Зарегистрирован: 15.01.10
Откуда: Россия
Рейтинг: 24
ссылка на сообщение  Отправлено: 27.10.19 15:44. Заголовок: 1. Ri драйверной лам..


В начале этой темы RedStar писал:
 цитата:
До сих пор не пойму реальное применение, хотя прекрасно знаю, что представляет из себя внутреннее.зачем нужно Ri. Никто не отвечает по сути, полным объемом.

1. Ri драйверной лампы нужно знать, чтобы вычислить выходное сопротивление драйвера Rвых.
Если каскад резистивный, то его выходное сопротивление равно:
Rвых=Ra||Ri
Если Rвых составляет 10% и более от величины гридлика выходной лампы, то разделительный конденсатор Ср рассчитывают с учётом Rвых драйвера.
При наличии в драйверном каскаде ООС (например, Rк не зашунтирован ёмкостью), то Ri.св лампы возрастёт:



, и его тоже надо узнать (вычислить), чтобы определить величину Ср.

Некто может возразить, что Ср легче подобрать, чем рассчитывать, но, во-первых, для этого нужен запас разных по ёмкости конденсаторов, а во-вторых, зачем тогда нужна векторная алгебра, если детали подбираем "методом тыка"?

2. Ri лампы в выходном каскаде также участвует в формировании выходного сопротивления:



Чем меньше Rвых, тем лучше коэффициент демпфирования подвижной системы динамика.

Вывод: Ri - важный параметр лампы, и его, так же, как как µ и S желательно знать.

Спасибо: 0 
Профиль
Aleph





Пост N: 603
Зарегистрирован: 03.06.15
Откуда: Макеевка
Рейтинг: 7
ссылка на сообщение  Отправлено: 27.10.19 17:52. Заголовок: RedStar пишет: Так ..


RedStar пишет:
 цитата:
Так зачем же такой "отправной точкой" используете Ri для вычислений, что резистивных, что трансформаторных? Включая Альфу и примерные значения от 2 до 10 (или 0,1-0,5)Ri.

Да никто из умеющих проводить расчёт не использует эти соотношения, они просто показывают, в каких примерно пределах может находиться значение Ri, для новичков, просто для сведения. Кто умеет - строит нагрузку по ВАХ.

С ув. Сергей Спасибо: 0 
Профиль
RedStar
постоянный участник




Пост N: 2556
Зарегистрирован: 02.08.16
Откуда: Смоленск
Рейтинг: 0
ссылка на сообщение  Отправлено: 27.10.19 20:08. Заголовок: Ученик пишет: Объяс..


Ученик пишет:
 цитата:
Объясни, откуда ты берёшь точки K и L.

Зачем? Вы же все считаете, что у меня всё идет вразрез физике и закону дедушки Ома.
Кроме РТ есть еще два значения, - питание - Ea, и требуемая выходная амплитуда. От них надо исходить.

Aleph пишет:
 цитата:
никто из умеющих проводить расчёт не использует эти соотношения, они просто показывают,...

Далее и без Ri можно обойтись, верно? Уже говорил, что даже по примерным ВАХ можно точно настроить каскад.
Ведь только формулами можно доказать выходное сопротивление. Или нет?

Не надо иметь много колес, лучше иметь много спиц, удерживающих одно колесо. © Спасибо: 0 
Профиль
aleks8845



Пост N: 612
Зарегистрирован: 07.03.16
Рейтинг: 1
ссылка на сообщение  Отправлено: 27.10.19 20:37. Заголовок: RedStar пишет: Дале..


RedStar пишет:
 цитата:
Далее и без Ri можно обойтись, верно?

Если можно так выразитmся, пусть меня поправят, при определении РТ пусть Ri не будет являться? самым главным расчетным параметром , но в дальнейшем ,оно (Ri), все равно, будет иметь значение. В резистивном каскаде будет влиять на выходное сопротивление каскада, а в трансформаторном.... К примеру мне нужен трансформаторный драйвер, я буду выбирать к МКТ низкоомную лампу, т.е. с бОлее низким Ri, и уж точно не поставлю ничего в пентодно, тетродном включении, где Ri бывает до сотни кОм...

Спасибо: 0 
Профиль
Пермяк
moderator




Пост N: 6383
Зарегистрирован: 15.01.10
Откуда: Россия
Рейтинг: 24
ссылка на сообщение  Отправлено: 27.10.19 22:13. Заголовок: RedStar пишет: Ведь ..


RedStar пишет:
 цитата:
и без Ri можно обойтись, верно?

Моим постом выше я показал, для чего нужно ЗНАТЬ величину Ri.

RedStar пишет:
 цитата:
Ведь только формулами можно доказать выходное сопротивление. Или нет?

Представьте , что нагрузка немного изменилась. например, Rдинамика уменьшилось на НЧ. Уменьшится и напряжение на динамике. И изменится оно тем сильнее, чем выше значение Rвых, которое зависит от Ri выходной лампы. Формулу я показал, поэтому вот эта Ваша Фраза свидетельствует, что Вы не не поняли:
 цитата:
Ведь только формулами можно доказать выходное сопротивление. Или нет?

Повторюсь: падение напряжения при снижении Rнагрузки показывает, что Ri - реальная физическая величина.

Спасибо: 0 
Профиль
Ученик



Пост N: 886
Зарегистрирован: 06.04.13
Рейтинг: 2
ссылка на сообщение  Отправлено: 27.10.19 22:31. Заголовок: RedStar пишет: Зачем..


RedStar пишет:
 цитата:
Зачем?

Затем. Ты ведёшь расчёт от этих точек. Так объясни, откуда ты их берёшь.
RedStar пишет:
 цитата:
Кроме РТ есть еще два значения, - питание - Ea

Ты не выдумывай на ходу, Еа ты получаешь далее из своего расчёта. А расчёт ведёшь от трёх точек. Про точку Р понятно, откуда взялись K и L?

Учиться никогда не поздно, и никому не вредно. Спасибо: 0 
Профиль
RedStar
постоянный участник




Пост N: 2557
Зарегистрирован: 02.08.16
Откуда: Смоленск
Рейтинг: 0
ссылка на сообщение  Отправлено: 27.10.19 23:31. Заголовок: А для случая с пенто..


А для случая с пентодом, у которого 30 кОм без МОС? Какое выходное сопротивление? Около 60-100 Ом?
Почему должно измениться Ri при изменении нагрузки? Изменяется ведь Ra при неизменном Ктр. Что участвует как Ra? (пункт 2 из поста Леонида).
W1 = Ra? Тогда должно быть последовательно.

Ученик пишет:
 цитата:
откуда взялись K и L?

Из графика, циркулем от РТ.

Не надо иметь много колес, лучше иметь много спиц, удерживающих одно колесо. © Спасибо: 0 
Профиль
Ученик



Пост N: 887
Зарегистрирован: 06.04.13
Рейтинг: 2
ссылка на сообщение  Отправлено: 28.10.19 00:00. Заголовок: RedStar пишет: Из гр..


RedStar пишет:
 цитата:
Из графика, циркулем от РТ.

1 Из какого "графика"? Дежурств по кухне?
2 Циркулем - это будет окружность.
3 Почему ты поставил точки в этих местах окружности - непонятно.
P.S. Ты сочинил какой-то расчёт. Что является целью расчёта, откуда и как берёшь исходные данные, почему считаешь, что так будет лучше (в сравнении с чем?)...никакой информации. Почему из тебя всё клещами надо вытаскивать?

Учиться никогда не поздно, и никому не вредно. Спасибо: 0 
Профиль
RedStar
постоянный участник




Пост N: 2558
Зарегистрирован: 02.08.16
Откуда: Смоленск
Рейтинг: 0
ссылка на сообщение  Отправлено: 28.10.19 00:24. Заголовок: 1. А разве не понятн..


1. А разве не понятно, что ВАХ.
2. У вас циркулем только окружность? Про измерения им расстояний никак не догадались?
Сказал ведь, что хватит про мой расчет. Ходите вокруг да около не понимая.

Не надо иметь много колес, лучше иметь много спиц, удерживающих одно колесо. © Спасибо: 0 
Профиль
Пермяк
moderator




Пост N: 6384
Зарегистрирован: 15.01.10
Откуда: Россия
Рейтинг: 24
ссылка на сообщение  Отправлено: 28.10.19 00:30. Заголовок: RedStar пишет: А для..


RedStar пишет:
 цитата:
А для случая с пентодом, у которого 30 кОм без МОС? Какое выходное сопротивление? Около 60-100 Ом?

Я Вам писал выше:
 цитата:
Rвых=Ra||Ri

, т.е. параллельному соединению внутреннего и анодного сопротивлений.
Умеете считать сопро двух параллельно соединённых?

Спасибо: 0 
Профиль
Ученик



Пост N: 888
Зарегистрирован: 06.04.13
Рейтинг: 2
ссылка на сообщение  Отправлено: 28.10.19 01:47. Заголовок: RedStar пишет: А раз..


RedStar пишет:
 цитата:
А разве не понятно, что ВАХ.

Так-так...
 цитата:
У вас циркулем только окружность?

Циркуль для того и придуман. А ты предлагаешь тыкать циркулем в экран монитора, или ВАХ в книге, а потом прикладывать его к линейке? Офигенная "точность", и удобство.
 цитата:
Сказал ведь, что хватит про мой расчет.

Нет, не хватит. В сухом остатке: тебя чем-то не устраивала точность графического метода. Поскрипев мозгами, ты взял те же самые графические ВАХ, но выбрал для отсчёта не крайние точки, а близлежащие к Рт. То, что точность от этого может только пострадать, в голову тебе не пришло. Ну ладно, взял и взял. Но потом проводишь какие-то идиотские громоздкие подсчёты...а просто считать с осей значения сложно? Вот же, на твоём графике Ua0 = 270 Iam = 13,5. Или же составить обычные пропорции, как в начальной школе учили - ума не хватило?
Полный капец...векторная алгебра.
Наткнулся:



Учиться никогда не поздно, и никому не вредно. Спасибо: 1 
Профиль
RedStar
постоянный участник




Пост N: 2560
Зарегистрирован: 02.08.16
Откуда: Смоленск
Рейтинг: 0
ссылка на сообщение  Отправлено: 28.10.19 03:28. Заголовок: Ученик пишет: Цирку..


Ученик пишет:
 цитата:
Циркуль для того и придуман.

О как.
Ци́ркуль — инструмент для черчения окружностей и дуг, также может быть использован для измерения расстояний, в частности, на картах.
Причем здесь монитор? Есть принтер, распечатать, а потом проводить измерения.
 цитата:
точность от этого может только пострадать

Больше страдает точность от 2-10Ri, или в пентоде 0.2-0.5Ri. (заодно ответил на вопрос Кузьмича).
 цитата:
Вот же, на твоём графике Ua0 = 270, Iam = 13,5.

Маленько ошиблись: Ea=270, Imax=13.5. А точки Ua0 = 190, Iam = 1,0
 цитата:
Наткнулся:

Ой, тоже мне, нашли столетней давности, взятое из одной хорошей книги. И что?


Не надо иметь много колес, лучше иметь много спиц, удерживающих одно колесо. © Спасибо: 0 
Профиль
Aleph





Пост N: 605
Зарегистрирован: 03.06.15
Откуда: Макеевка
Рейтинг: 7
ссылка на сообщение  Отправлено: 28.10.19 04:34. Заголовок: RedStar, Вы бы заявк..


RedStar, Вы бы заявку на нобелевку дали, что ли (или может на премию Дарвина?). Почти 100 лет все считают как в учебнике, а тут Вы новый метод открыли. Да ещё и всех тайн методики не пишете, только обзываете всех неучами.
Вы рискните на Вегалабе написать. Там быстро бан влепят за "тайные знания, недоступные простым физикам".
Пермяк же написал доступно, хоть глотай и переваривай:
 цитата:
Представьте , что нагрузка немного изменилась, например, Rдинамика уменьшилось на НЧ. Уменьшится и напряжение на динамике. И изменится оно тем сильнее, чем выше значение Rвых, которое зависит от Ri выходной лампы.
Повторюсь: падение напряжения при снижении Rнагрузки показывает, что Ri - реальная физическая величина.



С ув. Сергей Спасибо: 0 
Профиль
Ученик



Пост N: 889
Зарегистрирован: 06.04.13
Рейтинг: 2
ссылка на сообщение  Отправлено: 28.10.19 05:50. Заголовок: RedStar пишет: такж..


RedStar пишет:
 цитата:
Больше страдает точность от 2-10Ri, или в пентоде 0.2-0.5Ri.

Вам уже объясняли, и не раз, что в большинстве случаев никто не считал и не считает нагрузку строго от Ri. Типа нагрузка д.б. = 4,15Ri. Иногда задаются альфой не меньше какой-то определённой величины. А какой конкретно - выбирает разработчик, хочет мощность побольше - одно, хочет высокий коэффициент демпфирования - другое. Для пентодов в звуковых цепях Ri вообще не учитывается. Сколько раз повторять?
 цитата:
Маленько ошиблись: Ea=270, Imax=13.5.

Видишь, ты понял о чём речь, ибо я не поленился, и пояснил про точки пересечения нагрузочной прямой с осями. Что же мешает тебе (уж не меньше года, наверное) сосредоточиться, и вместо обрывков-ребусов оформить свои мысли в нормальную форму?
Ну так что там про точность "твоего метода"?
 цитата:
Ой, тоже мне, нашли столетней давности, взятое из одной хорошей книги.

Да из какой книги? Такая чушь только в твоей голове родиться могла.

Aleph пишет:
 цитата:
а тут Вы новый метод открыли.

Да какой там новый метод. Вбил себе в голову, что нужна какая-то суперточность в этих вопросах... вот и пыжится. Сколько раз писали, что всё это усреднённое, лампы отличаются, да и нет существенных изменений от вариаций +/- 5, а часто и 10% - хоть кол на голове теши. Это называется просто - навязчивая мысль.

Учиться никогда не поздно, и никому не вредно. Спасибо: 0 
Профиль
Пермяк
moderator




Пост N: 6385
Зарегистрирован: 15.01.10
Откуда: Россия
Рейтинг: 24
ссылка на сообщение  Отправлено: 28.10.19 13:45. Заголовок: RedStar пишет: Ведь ..


RedStar пишет:
 цитата:
Ведь только формулами можно доказать выходное сопротивление. Или нет?

Выходное сопротивление можно измерить. Причём, разными методами.
Например, для трансформаторного каскада, "метод двух нагрузок":



Rвых резистивного каскада можно измерить таким же методом.
Нагрузкой резистивного каскада является сеточный резистор Rg последующего каскада, для драйвера - выходного.
Подаём на вход каскада сигнал такой величины, чтобы переменка на Rg стала равной, скажем, U1=5В. Затем, параллельно Rg подключаем резистор R2 такой величины, чтобы напряжение на нём уменьшилось, скажем, до U2=4В.
Формула для вычисления Rвых будет такой:


В формуле R1 - это Rg.

Спасибо: 1 
Профиль
RedStar
постоянный участник




Пост N: 2561
Зарегистрирован: 02.08.16
Откуда: Смоленск
Рейтинг: 0
ссылка на сообщение  Отправлено: 28.10.19 15:03. Заголовок: Ученик пишет: в бол..


Ученик пишет:
 цитата:
в большинстве случаев никто не считал и не считает нагрузку строго от Ri.

Почему тогда строго считают Rвых от Ri? (не считая метода двух нагрузок)

Пермяк пишет:
 цитата:
Выходное сопротивление можно измерить.


Спасибо за повтор. А вопрос выше.

Не надо иметь много колес, лучше иметь много спиц, удерживающих одно колесо. © Спасибо: 0 
Профиль
Пермяк
moderator




Пост N: 6387
Зарегистрирован: 15.01.10
Откуда: Россия
Рейтинг: 24
ссылка на сообщение  Отправлено: 28.10.19 15:14. Заголовок: RedStar пишет: Почем..


RedStar пишет:
 цитата:
Почему тогда строго считают Rвых от Ri? (не считая метода двух нагрузок)

Методом двух нагрузок - ИЗМЕРЯЮТ.
А с учётом Ri - ВЫЧИСЛЯЮТ. На стадии проектирования.

Кстати, Ri лампы ведь тоже можно измерить. Умеете? Пробовали?

Спасибо: 0 
Профиль
Shef



Пост N: 229
Зарегистрирован: 21.02.19
Откуда: Vancouver
Рейтинг: -1
ссылка на сообщение  Отправлено: 28.10.19 15:21. Заголовок: RedStar, я всегда за..


RedStar, я всегда за справедливость. Если вы несёте очевидную пургу - я вам так же об этом укажу.

However, на ваших видео я увидел увлечённость кружкА энтузиастов, чего Ученик не представил. Это ахринительмо весомый аргумент, и меняет всё. ПостИте ваши видео как в дальнейшем идут дела, мы внимаемъ ))
Я вам советовал заглушить комнату, в противоположной стороне от акустики, вы сделал?

Спасибо: 0 
Профиль
RedStar
постоянный участник




Пост N: 2562
Зарегистрирован: 02.08.16
Откуда: Смоленск
Рейтинг: 0
ссылка на сообщение  Отправлено: 28.10.19 16:05. Заголовок: Пермяк пишет: А с у..


Пермяк пишет:
 цитата:
А с учётом Ri - ВЫЧИСЛЯЮТ. На стадии проектирования.

Хорошо. Вычисление выходного сопротивления от нагрузки, каким образом Ri изменяет?
Вроде должны уменьшится ток и напряжение одновременно, или наоборот, увеличиться ток с напряжением.

Shef пишет:
 цитата:
Если вы несёте очевидную пургу - я вам так же об этом укажу.

Я не против.
 цитата:
Я вам советовал заглушить комнату,

Комната не моя, одного из любителей. Так ему больше нравится. Только мой фотик сильно искажает при записи видео.
Продолжение будет. Летний период приостановил все работы. Сейчас начинается новый виток экспериментов и прослушек.

Плюмбум,там усваивать, в примере, особого труда нет.
https://drive.google.com/file/d/1Ug-7P87ybPSugJS3z0e7Mvnk1me06oHB/view?usp=sharing

Для Ученика: циркулем проще найти равнозначность отрезков, чем линейкой. Попробуйте на досуге. Только не на мониторе и книге, а на отдельно распечатанном листе.

Не надо иметь много колес, лучше иметь много спиц, удерживающих одно колесо. © Спасибо: 0 
Профиль
RedStar
постоянный участник




Пост N: 2564
Зарегистрирован: 02.08.16
Откуда: Смоленск
Рейтинг: 0
ссылка на сообщение  Отправлено: 28.10.19 18:08. Заголовок: Вот что интересно. К..


Ученик пишет:
 цитата:
Что же мешает тебе (уж не меньше года, наверное) сосредоточиться, и вместо обрывков-ребусов оформить свои мысли в нормальную форму?

Нет учителей, готовых подробно растолковать, и нет полного технического образования.
Вот и вся проблема.
Есть любители, нет теоретиков, в моем окружении. А практика только постепенно самостоятельности учит. И на это требуется больше времени.
Вот пример, Леонид, Пермяк. Ему обязан многим. Толково пишет и в тему, без "заковыристо-оскорбительных" слов. Вам бы всем научиться его выдержке.
Который раз прошу, кто же может общаться со мной вне форума? Увы, нет желающих. Только "обос...ь" можете.
Жаль только то, что не все сразу понимается. Хотя есть реальные отличия от теории с практикой. Есть прямые нестыковки. Вот за них и цепляюсь.
Всему нужны доказательства.

Вот что интересно.
Ктр трансформатора неизменно, вроде, так как не изменяется соотношение витков. Значит это не Ктр, а "n"? (как называетя?)
При снижении сопротивления нагрузки или повышении, соотношение нагрузки к приведенному изменяется, - это Ктр.
Индуктивное сопротивление меняется. Значит ток первички становится больше-меньше но с одновременным уменьшением-повышением напряжения.
Причем тут Ri?
Извините, ели опять "белиберду" написал.

Не надо иметь много колес, лучше иметь много спиц, удерживающих одно колесо. © Спасибо: 0 
Профиль
Пермяк
moderator




Пост N: 6388
Зарегистрирован: 15.01.10
Откуда: Россия
Рейтинг: 24
ссылка на сообщение  Отправлено: 28.10.19 18:41. Заголовок: RedStar пишет: Вычис..


RedStar пишет:
 цитата:
Вычисление выходного сопротивления от нагрузки, каким образом Ri изменяет?

Вычисление, как процесс, само по себе ничего не меняет. Оно просто даёт результат.
Может быть, Вы хотели спросить как-то иначе?
 цитата:
циркулем проще найти равнозначность отрезков

Не равнозначность, а равенство.

Спасибо: 0 
Профиль
XMR



Не зарегистрирован
Зарегистрирован: 01.01.70
ссылка на сообщение  Отправлено: 28.10.19 19:32. Заголовок: Пермяк пишет: RedSt..


Пермяк пишет:
 цитата:
RedStar пишет:
цитата:
Ведь только формулами можно доказать выходное сопротивление. Или нет?

Выходное сопротивление можно измерить. Причём, разными методами.
Например, для трансформаторного каскада, "метод двух нагрузок":



Rвых резистивного каскада можно измерить таким же методом.
Нагрузкой резистивного каскада является сеточный резистор Rg последующего каскада, для драйвера - выходного.
Подаём на вход каскада сигнал такой величины, чтобы переменка на Rg стала равной, скажем, U1=5В. Затем, параллельно Rg подключаем резистор R2 такой величины, чтобы напряжение на нём уменьшилось, скажем, до U2=4В.
Формула для вычисления Rвых будет такой:


В формуле R1 - это Rg.

А этот метод подойдет для измерения выходного сопротивления пентодов и каскодов?

Спасибо: 0 
Пермяк
moderator




Пост N: 6389
Зарегистрирован: 15.01.10
Откуда: Россия
Рейтинг: 24
ссылка на сообщение  Отправлено: 28.10.19 20:58. Заголовок: XMR пишет: А этот ме..


XMR пишет:
 цитата:
А этот метод подойдет для измерения выходного сопротивления пентодов и каскодов?

Подойдёт для любых каскадов.

Спасибо: 0 
Профиль
aleks8845



Пост N: 613
Зарегистрирован: 07.03.16
Рейтинг: 1
ссылка на сообщение  Отправлено: 28.10.19 21:52. Заголовок: RedStar пишет: Ктр ..


RedStar пишет:
 цитата:
Ктр трансформатора неизменно, вроде, так как не изменяется соотношение витков.
При снижении сопротивления нагрузки или повышении, соотношение нагрузки к приведенному изменяется, - это Ктр.
Индуктивное сопротивление меняется.
Причем тут Ri?

Может не все удалось понять, но к выше сказанному такие мысли пришли:
Возьмем простой макет:
-Лампа , в аноде резистор или Твз ( у резистора R пост. току неизменно, у тр-ра тоже );
- На выход такого каскада подключим нагрузку в виде пост. резистора ( не АС, чтоб не менялся импенданс);
- Подадим Ea и переменный сигнал на управляющую сетку лампы с опр-й частотой;
Если у лампы нет Ri, то на выходе каскада не будет сигнала....

Спасибо: 1 
Профиль
Ученик



Пост N: 890
Зарегистрирован: 06.04.13
Рейтинг: 2
ссылка на сообщение  Отправлено: 29.10.19 03:25. Заголовок: Shef, на приведённых..


RedStar пишет:
 цитата:
циркулем проще найти равнозначность отрезков, чем линейкой.

Что значит "найти", отложить? На чём отложить, на этой линии U-I ?
http://hiend.borda.ru/?1-22-1573443393804-00000544-000-60-0#039
Откуда эта линия взялась, возникла из воздуха? Это линия нагрузки, или ещё что-то? Что эти отрезки означают? Почему они равные, это по ВАХ так получилось (что сомнительно) или ты так решил? Если ты так решил - почему, есть какие-то основания, или просто почесал в затылке? Зачем откладывать два отрезка? Точки U и I, соответствующие второй точке, легко находятся простой пропорцией.

Вот ты наверное думаешь, это я специально придираюсь...

RedStar пишет:
 цитата:
Нет учителей, готовых подробно растолковать, и нет полного технического образования.

Так ведь даже не о технической стороне... Как-то не получается у тебя начать с самого начала, с простого изложения:
вот расчёт, (чего рассчитываешь, какие цели у расчёта), данные для расчёта (откуда, каким образом взяты или получены).
Вот ты пишешь: "а разве непонятно, что ВАХ". А где эти ВАХи-то на твоём рисунке? Какой лампы? Почему не на них нарисовал? - непонятно, что мешало.

P.S. Упорядочить мысли очень хорошо помогает написание инструкций (считаю, что умение составить хорошую инструкцию - это искусство). Представь, что тебе надо на бумаге изложить для нормального не тупого человека инструкцию по пользованию... пусть микроволновкой. Допустим, он никогда о них не слышал, и не видел.

Учиться никогда не поздно, и никому не вредно. Спасибо: 0 
Профиль
Кузьмич



Пост N: 1430
Зарегистрирован: 21.11.15
Рейтинг: 8
ссылка на сообщение  Отправлено: 29.10.19 11:37. Заголовок: RedStar пишет: Нет ..


RedStar пишет:
 цитата:
Нет учителей, готовых подробно растолковать, и нет полного технического образования.
Вот и вся проблема.

Тебе здесь уже по пятнадцатомц разу растолковывают каждый пункт. Так ты всё отвергаешь и придумываешь свою какую-то хрень,
типа векторной алгебры. Это равнозначно пятому колесу в телеге.
RedStar пишет:
 цитата:
XMR пишет:
цитата:
Мне кажется что ты не можешь доходчиво донести суть своей методики
Это вернее.

Какая ещё своя методика??? Методика выработана почти 100 лет назад.
Всё продумано и изложена понятным, доступным языком.
Если соображалка плохо работает, тупо выписать для себя что и как считать.

Спасибо: 1 
Профиль
RedStar
постоянный участник




Пост N: 2567
Зарегистрирован: 02.08.16
Откуда: Смоленск
Рейтинг: 0
ссылка на сообщение  Отправлено: 29.10.19 18:28. Заголовок: Кто нарисует график ..


Кто нарисует график изменения внутреннего Ri при переменной составляющей анод-катод и индуктивной нагрузке?

Не надо иметь много колес, лучше иметь много спиц, удерживающих одно колесо. © Спасибо: 0 
Профиль
Кузьмич



Пост N: 1432
Зарегистрирован: 21.11.15
Рейтинг: 8
ссылка на сообщение  Отправлено: 29.10.19 20:05. Заголовок: RedStar пишет: Кто ..


RedStar пишет:
 цитата:
Кто нарисует график изменения внутреннего Ri при переменной составляющей анод-катод и индуктивной нагрузке?

Во-первых, снова написал бессвязную, несуразную чушь.
Во-вторых, никто не будет, т.к. этот график никому и нигде не нужен, по причине того, что какой-либо полезной информации он не несёт.
Т.е. тратить время ради твоей забавы-прихоти вряд ли кто намерен.
И в-третьих, различные радиолампы имеют своё внутреннее сопротивление, изменяющееся при изменении РТ.
Степень изменения так же различна. Ты собрался всё в одну зависимость определить?
И в-четвёртых, если тебе это так нужно и у тебя есть много свободного времени, то проделай это самостоятельно...

Спасибо: 0 
Профиль
RedStar
постоянный участник




Пост N: 2568
Зарегистрирован: 02.08.16
Откуда: Смоленск
Рейтинг: 0
ссылка на сообщение  Отправлено: 29.10.19 20:17. Заголовок: Кузьмич пишет: этот..


Кузьмич пишет:
 цитата:
этот график никому и нигде не нужен, по причине того, что какой-либо полезной информации он не несёт.

Если он информации не несет, нет такого графика, - зачем вам Ri? Мне тем более он не нужен.
 цитата:
различные радиолампы имеют своё внутреннее сопротивление, изменяющееся при измерении РТ.

Но если РТ неизменна? А она неизменна.

Не надо иметь много колес, лучше иметь много спиц, удерживающих одно колесо. © Спасибо: 0 
Профиль
Кузьмич



Пост N: 1433
Зарегистрирован: 21.11.15
Рейтинг: 8
ссылка на сообщение  Отправлено: 29.10.19 20:47. Заголовок: RedStar пишет: Если..


RedStar пишет:
 цитата:
Если он информации не несет, нет такого графика, - зачем вам Ri?

Это твой график не несёт информации. А Ri в РТ нужна.
Больше, ни в какой другой точке нагрузочной (динамической) она не нужна. Ну меняется она немного, в одну сторону в плюс, в другую в минус.
Это ничего не значит, как напруга в розетке 220, от ноля до 310...
- Для того, что бы определиться с Альфой и последующим расчётом нагрузки, и источника питания;
- что бы рассчитать вых. сопр. каскада и Rg следующего каскада.
Ты ведь разделительный конденсатор и Rg следующего каскада берёшь какой под рукой и лепишь в схему.
А их, по уму, рассчитывать надо.
- Да и вообще по Ri определяю, пойдёт ли мне эта лампа в каскаде или всё таки взять другую, с Ri поменьше.
RedStar пишет:
 цитата:
Мне тем более не нужен он.

А нахрена тогда задаёшь вопрос:
Кто нарисует график изменения внутреннего Ri при переменной составляющей анод-катод и индуктивной нагрузке?
???????

Спасибо: 0 
Профиль
RedStar
постоянный участник




Пост N: 2569
Зарегистрирован: 02.08.16
Откуда: Смоленск
Рейтинг: 0
ссылка на сообщение  Отправлено: 29.10.19 21:02. Заголовок: Кузьмич пишет: А Ri..


Кузьмич пишет:
 цитата:
А Ri в РТ нужна.

Чтобы узнать, насколько изменятся ток с напряжением? С чего они там изменятся?
Как верно пишете, что при разных значениях смещения - Ri разное.
 цитата:
А нахрена тлгда задаёшь вопрос:

Когда увижу абсолютную зависимость, тогда перестану задавать вопрос. Так повторю его:
Кто нарисует график изменения внутреннего Ri при переменной составляющей анод-катод и индуктивной нагрузке?

Не надо иметь много колес, лучше иметь много спиц, удерживающих одно колесо. © Спасибо: 0 
Профиль
RedStar
постоянный участник




Пост N: 2570
Зарегистрирован: 02.08.16
Откуда: Смоленск
Рейтинг: 0
ссылка на сообщение  Отправлено: 29.10.19 21:18. Заголовок: П.С. Почему же у мен..


П.С. Почему же у меня, при экспериментах и сборке, не возникает вопроса расчетов через Ri и Альфу?

Почему достигаю минимальные искажения всего тракта не более 0,5% при максимальном выходной мощности?Почему все напряжения соответствуют моим расчетам? Без тщательного подбора элементов и намотки трансформатора за один раз.

Вот пример работы с циркулем. Где ожидается большая мощность с минимум искажений?
Оба режима мною оттестированы.



Не надо иметь много колес, лучше иметь много спиц, удерживающих одно колесо. © Спасибо: 0 
Профиль
Кузьмич



Пост N: 1434
Зарегистрирован: 21.11.15
Рейтинг: 8
ссылка на сообщение  Отправлено: 29.10.19 21:37. Заголовок: RedStar пишет: Что ..


RedStar пишет:
 цитата:
Что бы узнать на сколько изменится ток с напряжением? С чего они там изменятся?

Нет. Тебе несколько раз объясняли для чего.
Давай так:
Вот тебе клиент заказал для своей дубовой резиновой акустики 8 Ом усилитель, но что бы коэффициент демпфирования был не менее 6 (шести).
Иначе его акустика брыкается и плохо звучит.
Про мощность ... хватит и 2-х ватт. На 10 ватт обижаться не станет.
Как ты будешь выбирать выходную лампу и угождать клиенту с коэфициентом демпфирования???
Можешь рассказать??? Своими словами.
 цитата:
Почему же у меня, при экспериментах и сборке, не возникает вопроса расчетов через Ri и Альфу?

Очень просто, ты про них не знаешь. Как про суслика..., ты его не видишь, а он там есть.
Плюс ещё то, что лампа, в отличие от транзистора, очень терпеливый компонент. И способна терпеть многие огрехи паяльщика.
Ну, и это говорит о низком уровне знаний паяльщика.
 цитата:
Почему достигаю минимальные искажения всего тракта не более 0,5% при максимальном выходной мощности?

Вот на этом рисунке (с циркулем) не вооружённым глазом видно, что 0,5% искажений и близко нет. Гораздо больше.
И если твои измерения показывают 0,5% по указанному рисунку и в указанных режимах - они далеко не точные.

Да, с введением ОС искажения уменьшаются и можно получить 0,5%. Но качество звука от этого не всегда выигрывает.
Только тебе это неведомо.

Спасибо: 0 
Профиль
омельян





Пост N: 665
Зарегистрирован: 07.03.16
Откуда: Украина, Полтава
Рейтинг: 0
ссылка на сообщение  Отправлено: 30.10.19 01:29. Заголовок: RedStar пишет: Оба ..


RedStar пишет:
 цитата:
Оба режима

Я бы выбрал: Ua0 = 210 В; Ia0 = 80 мА. Рабочая точка должна быть красивой. Тогда и искажения минимальны.

Спасибо: 0 
Профиль
Ученик



Пост N: 891
Зарегистрирован: 06.04.13
Рейтинг: 2
ссылка на сообщение  Отправлено: 30.10.19 02:03. Заголовок: Кузьмич пишет: Кто н..


RedStar пишет:
 цитата:
при переменной составляющей анод-катод и индуктивной нагрузке?

Переменной - какой величины? При чисто индуктивной нагрузке - зачем тебе это в звуковом усилителе?

И да, Кузьмич прав, никаких 0,5% без ОС нет и не будет. Не вешай лапшу.

Учиться никогда не поздно, и никому не вредно. Спасибо: 0 
Профиль
Кузьмич



Пост N: 1437
Зарегистрирован: 21.11.15
Рейтинг: 8
ссылка на сообщение  Отправлено: 30.10.19 03:04. Заголовок: Ученик пишет: Во вс..


омельян пишет:
 цитата:
Я бы выбрал: Ua0 = 210 В; Ia0 = 80 мА

.Я бы 250 вольт взял. И ток ма 70...
25 вольт смещение, ламп для драйвера огромный выбор.

Спасибо: 0 
Профиль
Пермяк
moderator




Пост N: 6391
Зарегистрирован: 15.01.10
Откуда: Россия
Рейтинг: 24
ссылка на сообщение  Отправлено: 30.10.19 12:19. Заголовок: Смотрю на рисунок Re..


Смотрю на рисунок RedStar'a, и - круги перед глазами... :)

Для тех, кто пока не в курсе.
Чтобы оценить коэфф. гармоник Кг хотя бы по 2-й гармонике, нужно иметь на ВАХ, на нагрузочной прямой, минимум три точки:
- точку покоя при Ua0, Ia0, Ug0.
- точку на пересечении нагрузочной прямой с кривой Ug=0,
- точку на пересечении нагрузочной прямой с кривой Ug=2Ug0.
(см."Красный" Цыкин, стр.75.)

У RedStar'a на рисунке третьей точки нет, так что никакой оценки Кг по его графику быть не может.

Анатолий, триоды и псевдотриоды имеют такое свойство: линии ВАХ неэквидистантны, т.е расстояние между кривыми на графике тем меньше, чем правее расположена кривая ВАХ. А ты откладываешь два одинаковых радиуса влево и вправо от точки покоя... Не может правая часть нагрузочной быть равной по длине левой, правая всегда короче левой.

png hosting

Спасибо: 1 
Профиль
Кузьмич



Пост N: 1441
Зарегистрирован: 21.11.15
Рейтинг: 8
ссылка на сообщение  Отправлено: 31.10.19 01:52. Заголовок: Пермяк пишет: Да.....


Пермяк пишет:
 цитата:
Не может правая часть нагрузочной быть равной по длине левой, правая всегда короче левой.

Отсюда и искажения. И никак не 0,5%, а поболе.:

Леонид, Вам респект!!!
Вот решил проверить с помощью psud2, и что получилось:



Как в аптеке! С точностью до сотых. А краснозвёзному с какой точностью надо?
Внутренее в рабочей точке 860 Ом.

Спасибо: 1 
Профиль
Ученик



Пост N: 894
Зарегистрирован: 06.04.13
Рейтинг: 2
ссылка на сообщение  Отправлено: 31.10.19 03:25. Заголовок: Кузьмич пишет: Как в..


Кузьмич пишет:
 цитата:
Как в аптеке!

А как же иначе, если формулы одни и те же, причём правильные?
Редстар как-то распинался, что Tubecurve - ужасная программа. Ну, до его "методики" конечно, не дотягивает...

Учиться никогда не поздно, и никому не вредно. Спасибо: 0 
Профиль
Кузьмич



Пост N: 1442
Зарегистрирован: 21.11.15
Рейтинг: 8
ссылка на сообщение  Отправлено: 31.10.19 03:34. Заголовок: Ученик пишет: Редст..


Ученик пишет:
 цитата:
Редстар как-то распинался, что Tubecurve - ужасная программа.

Имею основания полагать, что он не может справиться с программой.

Спасибо: 1 
Профиль
Shef



Пост N: 240
Зарегистрирован: 21.02.19
Откуда: Vancouver
Рейтинг: 0
ссылка на сообщение  Отправлено: 31.10.19 04:56. Заголовок: RedStar, распечатайт..


RedStar, распечатайте на стену, рамка уже есть



Кстати говоря, совсем недавно видел топик на DIYA, обсуждалась рабочая точка кажется 88 в пентоде, выбранная с заходом в область "клюшки", так, чтобы эквидистантность более-менее выполнялась.

Спасибо: 0 
Профиль
Пермяк
moderator




Пост N: 6394
Зарегистрирован: 15.01.10
Откуда: Россия
Рейтинг: 25
ссылка на сообщение  Отправлено: 31.10.19 12:28. Заголовок: Немного отвлеклись о..


Shef пишет:
 цитата:
видел топик на DIYA, обсуждалась рабочая точка 88 в пентоде, так, чтобы эквидистантность более-менее выполнялась.

Для пентода это возможно.

RedStar пишет в топе:
 цитата:
Прошу высказать полное обоснование расчетов Ri

Анатолий, забыл тебе упомянуть.
Есть топология с параллельной ООС, применённая О.Чернышёвым в его услителе "Покемон". В такой схеме Ri драйверной лампы входит в цепь обратной связи, и не зная Ri, невозможно провести расчёт всей схемы.



И ещё, и это важно: значение Ri необходимо знать при вычмслении требуемой индуктивности первички при расчёте трансформаторного каскада:
http://ulfdiysound.ucoz.ru/publ/avtorskie_stranicy/stranica_leonida_permjaka/chto_takoe_soprotivlenie_ehkvivalentnogo_generatora_pri_raschjote_l1/8-1-0-21
то есть:



Спасибо: 0 
Профиль
RedStar
постоянный участник




Пост N: 2573
Зарегистрирован: 02.08.16
Откуда: Смоленск
Рейтинг: 0
ссылка на сообщение  Отправлено: 01.11.19 01:00. Заголовок: Aleph пишет: Сопрот..


Леонид, всё, что пишете, мне известно. Еще раз спасибо за напоминание.

Вот есть Ri - красным цветом, Ra - синим.
Какая между ними взаимосвязь? (Почему внутреннее не может быть Ro?)



Не надо иметь много колес, лучше иметь много спиц, удерживающих одно колесо. © Спасибо: 0 
Профиль
Кузьмич



Пост N: 1445
Зарегистрирован: 21.11.15
Рейтинг: 8
ссылка на сообщение  Отправлено: 01.11.19 01:10. Заголовок: RedStar пишет: Вот ..


RedStar пишет:
 цитата:
Вот есть Ri - красным цветом

Это - не Ri !!! Это отрезок... Ну, можно назвать линией.
Может хватит придумывать-то?
 цитата:
Почему внутреннее не может быть Ro?

А Ro - это что у тебя? Подозреваю, что это сопротивление лампы постоянному току в рабочей точке.
Численно Ri может оказаться равным Ro. Только это редкий частный случай. Не всегда и не везде и не для всех ламп.
 цитата:
Какая между ними взаимосвязь?

Взамоствязь между ними - коэффициент нагрузки Альфа, который ты выбрал, задумал... (сам об этом не зная).
Но это справедливо только для триода.

Спасибо: 0 
Профиль
RedStar
постоянный участник




Пост N: 2574
Зарегистрирован: 02.08.16
Откуда: Смоленск
Рейтинг: 0
ссылка на сообщение  Отправлено: 01.11.19 01:14. Заголовок: Кузьмич пишет: Это ..


Кузьмич пишет:
 цитата:
Это - не Ri

А это что у вас на графике? (такая черная линия):
http://hiend.borda.ru/?1-22-1572542031509-00000544-000-180-0#080.001.001.001
Кузьмич пишет:
 цитата:
А Ro - это что за новая выдумка? Это что???

Сопротивление постоянному току в РТ.

Не надо иметь много колес, лучше иметь много спиц, удерживающих одно колесо. © Спасибо: 0 
Профиль
Кузьмич



Пост N: 1446
Зарегистрирован: 21.11.15
Рейтинг: 8
ссылка на сообщение  Отправлено: 01.11.19 01:29. Заголовок: RedStar пишет: А эт..


RedStar пишет:
 цитата:
А это что у вас на графике? ( черная линия):

Это отрезок (линия) касательная к ветви ВАХ в рабочей точке.

Спасибо: 0 
Профиль
RedStar
постоянный участник




Пост N: 2575
Зарегистрирован: 02.08.16
Откуда: Смоленск
Рейтинг: 0
ссылка на сообщение  Отправлено: 01.11.19 01:36. Заголовок: А разве это не одно ..


И разве это не одно и то же?



Не надо иметь много колес, лучше иметь много спиц, удерживающих одно колесо. © Спасибо: 0 
Профиль
Кузьмич



Пост N: 1447
Зарегистрирован: 21.11.15
Рейтинг: 8
ссылка на сообщение  Отправлено: 01.11.19 01:40. Заголовок: RedStar пишет: А ра..


RedStar пишет:
 цитата:
А разве это не одно и тоже?

А как может быть линия и сопротивление быть одним и тем же????? Ну сам-то подумай?
Ты вроде русский человек, не папуас какой, приехавший недавно в Россию и не знающий русского языка...
Это вспомогательная линия, для вычисления внутреннего сопротивления лампы.

Кстати, при работе с программой эту линию не надо рисовать. Она в программе есть. Только её надо "сделать" что бы она была касательной.
Тогда программа в верхнем левом углу высвечивает Ri. У басурман оно обозначается

Спасибо: 0 
Профиль
RedStar
постоянный участник




Пост N: 2576
Зарегистрирован: 02.08.16
Откуда: Смоленск
Рейтинг: 0
ссылка на сообщение  Отправлено: 01.11.19 01:46. Заголовок: Кузьмич пишет: Это ..


Кузьмич пишет:
 цитата:
Это вспомогательная линия, для вычисления внутреннего сопротивления лампы.

Ну да, а я про что? Как они взаимосвязаны, Ri и Ra?

Не надо иметь много колес, лучше иметь много спиц, удерживающих одно колесо. © Спасибо: 0 
Профиль
Кузьмич



Пост N: 1448
Зарегистрирован: 21.11.15
Рейтинг: 8
ссылка на сообщение  Отправлено: 01.11.19 01:49. Заголовок: RedStar пишет: Как..


RedStar пишет:
 цитата:
Как они взаимосвязаны, Ri и Ra?

Так я же написал, ты читаешь или нет?
Кузьмич пишет:
 цитата:
Взамосвязь между ними - коэффициент нагрузки Альфа, который ты выбрал, задумал... (сам об этом не зная).
Но это справедливо только для триода.

Ra =α*Ri

Спасибо: 0 
Профиль
RedStar
постоянный участник




Пост N: 2577
Зарегистрирован: 02.08.16
Откуда: Смоленск
Рейтинг: 0
ссылка на сообщение  Отправлено: 01.11.19 02:00. Заголовок: Альфу тоже выбирать ..


Альфу тоже выбирать примерно надо. Не вижу связи.
Значит коэффициент нагрузки можете использовать, а мне коэффициенты изменения анодных напряжения и тока от входного напряжения, - нельзя?
Только не говорите, что такого не писАл.

Не надо иметь много колес, лучше иметь много спиц, удерживающих одно колесо. © Спасибо: 0 
Профиль
Кузьмич



Пост N: 1449
Зарегистрирован: 21.11.15
Рейтинг: 8
ссылка на сообщение  Отправлено: 01.11.19 02:12. Заголовок: RedStar пишет: Знач..


RedStar пишет:
 цитата:
Значит коэффициент нагрузки можете использовать

А почему нет? Использую всегда. Особенно при расчёте выходного каскада.
 цитата:
а мне коэффициенты изменения анодных напряжения и тока от входного напряжения, - нельзя?

А это что за такие коэффициенты??? Откуда взято???

Спасибо: 0 
Профиль
Пермяк
moderator




Пост N: 6395
Зарегистрирован: 15.01.10
Откуда: Россия
Рейтинг: 25
ссылка на сообщение  Отправлено: 01.11.19 05:43. Заголовок: RedStar пишет:Вот ес..


RedStar пишет:
 цитата:
Вот есть Ri - красным цветом, Ra - синим.
Какая между ними взаимосвязь?

Для триода - вот такая:

free picture upload

Хотим получить больше мощность - выбираем "альфу"≈2. Но искажения при такой нагрузке будут больше. Увеличивая "альфу" можем получить меньшие искажения, но и мощность будет пониже.
Всё зависит от наших целей. Никакой другой ЗАВИСИМОСТИ нет.

Этот график - теоретический.
А вот - для конкретной лампы (её Ri - около 800 Ом):



ЗЫ. Могу и про пентод показать зависимости, только надо ли?

ЗЗЫ. И ещё, как художник - художнику: поправочка. Почувствуйте разницу.https://postimg.cc/18SXZhtN

Спасибо: 0 
Профиль
RedStar
постоянный участник




Пост N: 2578
Зарегистрирован: 02.08.16
Откуда: Смоленск
Рейтинг: 0
ссылка на сообщение  Отправлено: 02.11.19 02:30. Заголовок: Пермяк пишет: Этот ..


Пермяк пишет:
 цитата:
Этот график - теоретический.

Само собой теоретический. К практике он имеет максимум 60% достоверности.
Но не об этом речь.

Вот график. Сомневающийся Кузьмич не может понять, что вспомогательная линия является внутренним лампы, так как она определяет изменение напряжения и тока анода при изменении сеточного напряжения.
По синим стрелкам переменное напряжение - индуктивное сопротивление нагрузки анода (без изменения частоты).
Красным - показывает влияние анодного напряжения на анодный ток при постоянном напряжении управляющей сетки.

Кузьмич, объясните, почему ваша "вспомогательная" линия имеет угол φ, которая составляет именно Ri?

Скажите, как все же эти две величины влияют друг на друга?



Не надо иметь много колес, лучше иметь много спиц, удерживающих одно колесо. © Спасибо: 0 
Профиль
Кузьмич



Пост N: 1456
Зарегистрирован: 21.11.15
Рейтинг: 8
ссылка на сообщение  Отправлено: 02.11.19 03:18. Заголовок: RedStar пишет: Само..


RedStar пишет:
 цитата:
Само собой теоретический. К практике он имеет максимум 60% достоверности.

Теоретический - значит показать как надо делать, что бы легко и просто было понять суть.

Если на реальный ВАХах, практика на 60% расходится с теорией - то с практикой у этого мастера беда.
 цитата:
Сомневающийся Кузьмич

Кузьмич ни чуть не сомневается. Он полностью уверен что,
RedStar пишет:
 цитата:
что вспомогательная линия является внутренним лампы, так как она определяет изменение напряжения и тока анода при изменении сеточного напряжения.

Никакая линия не является внутренним сопротивлением лампы.
И линия не может определять изменение напряжения и тока.
 цитата:
По синим стрелкам переменное напряжение - индуктивное сопротивление нагрузки анода (без изменения частоты).

А это на каком языке? Или слОгане?
Причём тут угол φ ? Я разве о нём что-то говорил?
Хотя могу сказать, сравнивая этот угол φ на ВАХах разных ламп, можно определить у какой лампы Ri больше, а у какой меньше.
Не более... Хотя опытный мастер и без угла это заметит.
Пока я не пойму что ты хочешь, буду отправлять тебя учиться.
Вот читай здесь про внутреннее. Да и про остальные диф. параметры полезно будет.

Спасибо: 0 
Профиль
RedStar
постоянный участник




Пост N: 2579
Зарегистрирован: 02.08.16
Откуда: Смоленск
Рейтинг: 0
ссылка на сообщение  Отправлено: 02.11.19 03:28. Заголовок: Кузьмич пишет: Теор..


Кузьмич пишет:
 цитата:
Теоретический - значит показать как надо делать, чтобы легко и просто было понять суть.

Как надо? А если НЕ как надо?
 цитата:
Если на реальный ВАХах, практика на 60% расходится с теорией - то с практикой у этого мастера беда.

Значит выкиньте эту теорию, займитесь РЕАЛЬНОЙ практикой.
 цитата:
Никакая линия не является внутренним сопротивлением лампы.

Да ладно, Вы уверены? Почитайте сами книги, в которых сказано про углы. Извинения приму, не сомневайтесь.
 цитата:
Причём тут угол φ ? Я разве о нём что-то говорил?

Вы же утверждаете:
Кузьмич пишет:
 цитата:
Это - не Ri !!! Это отрезок... Ну, можно назвать линией.

У вас линия не имеющая ничего с Ri?

Не надо иметь много колес, лучше иметь много спиц, удерживающих одно колесо. © Спасибо: 0 
Профиль
Ученик



Пост N: 898
Зарегистрирован: 06.04.13
Рейтинг: 2
ссылка на сообщение  Отправлено: 02.11.19 03:42. Заголовок: RedStar пишет: вспом..


RedStar пишет:
 цитата:
вспомогательная линия является внутренним лампы, так как она определяет изменение напряжения и тока анода при изменении сеточного напряжения.

Выделенное не имеет НИКАКОГО отношения к внутреннему сопротивлению усилительного прибора.
 цитата:
почему ваша "вспомогательная" линия имеет угол φ, которая составляет именно Ri?

Белиберда, что языковая, что техническая. Угол, которая составляет...
1 Прямая линия не имеет углов.
2 Две непараллельные линии в одной плоскости по определению образуют между собой какой-то угол.
3 Этот угол можно найти только после определения Ri, если оно нам заранее неизвестно. Т.е., проведя касательную (гипотенузу прямоугольного треугольника), мы легко находим катеты. Поэтому угол это бесполезен, лишняя, никому ненужная величина.
Всё это учили в школе...

RedStar пишет:
 цитата:
А если НЕ как надо?

Ты уж на пупе извертелся, пытаясь доказать, что "НЕ как надо" - лучше. Ничего путного выдавить из себя не можешь, оттого никого и не убедишь.
 цитата:
Значит выкиньте эту теорию

Потому что ты не способен её понять? Выкинь свой апломб... Выложенный Пермяком "теоретический" график, обрати внимание! - не для лампы конкретного типа. Это показательный график общих тенденций.
 цитата:
Даже не знаете внутреннего при постоянке

Нет такого параметра. Ri - динамическая характеристика.

Учиться никогда не поздно, и никому не вредно. Спасибо: 0 
Профиль
RedStar
постоянный участник




Пост N: 2580
Зарегистрирован: 02.08.16
Откуда: Смоленск
Рейтинг: 0
ссылка на сообщение  Отправлено: 02.11.19 04:17. Заголовок: Кузьмич пишет: Леон..


Кузьмич пишет:
 цитата:
Вы, видимо, вскользь смотрели примерный график, раз не поняли его сути.

Ученик пишет: [quote]1 Прямая линия не имеет углов.

А разве говорил, что именно "прямая линия"?
Вроде на графике указано, что угол имеет гипотенузу Ri относительно Ea (по оси абсцисс). А как же угол Ra относительно оси абсцисс? Или его тоже проигнорируете?

Не надо иметь много колес, лучше иметь много спиц, удерживающих одно колесо. © Спасибо: 0 
Профиль
Ученик



Пост N: 901
Зарегистрирован: 06.04.13
Рейтинг: 2
ссылка на сообщение  Отправлено: 02.11.19 05:12. Заголовок: RedStar пишет: А раз..


RedStar пишет:
 цитата:
А разве говорил, что именно "прямая линия"?

Ты свою писанину прочитать не можешь?
 цитата:
почему ваша "вспомогательная" линия имеет угол φ, которая составляет именно Ri?

Двух слов связать не можешь, ещё и споришь.
 цитата:
Этаким "болваном" хотите меня выставить?

Сам себя выставляешь. Не ищи проблемы на стороне, они - в тебе. Вот очередное подтверждение:
 цитата:
на графике указано, что угол имеет гипотенузу

Впечатление, что твоё образование закончилось на трёх классах.
 цитата:
А как же угол Ra относительно оси абсцисс?

А зачем знать величину этого угла?
Провели нагрузочную линию. Получился какой-то угол между этой линией и осью абсцисс. И? Куда его приткнёшь?

Учиться никогда не поздно, и никому не вредно. Спасибо: 1 
Профиль
Пермяк
moderator




Пост N: 6396
Зарегистрирован: 15.01.10
Откуда: Россия
Рейтинг: 25
ссылка на сообщение  Отправлено: 02.11.19 10:32. Заголовок: Пишу дляRedStar'..


Пишу для RedStar'а.

1.
В своём сообщении № 6395 про первый график я указал, что он -"теоретический" (то есть, указывает общую тенденцию зависимости от величины "альфа"). И это несказанно обрадовало тебя, и ты написал:
 цитата:
К практике он имеет максимум 60% достоверности.

. И при этом ты сделал вид, что совсем не заметил там второй график - практический, для конкретной лампы при конкретных условиях. Нехорошо.
Разумеется, Войшвилло не лично сам строил эти зависимости, а поручил кому-то из многих своих лаборантов и ассистентов. Не знаю также, этот график построен по результатам многочисленных измерений или просто получен в результате таких же многочисленных построений "на бумаге". Но то, что ты пишешь про какие-то 60% достоверности - то это очень неважно тебя характеризует. Такая погрешность ни в электронике, ни вообще в промышленности - недопустима. Твоя личная практика - не в счёт.
А допустимый разброс Ri этой лампы я показал тебе в даташите: http://hiend.borda.ru/?1-22-1572670000025-00000544-000-40-0#022.001.001.002.001.001.001.001.001.002

И ещё. Ты обозначаешь угол буквой "фи". Разве ты не знаешь, что в электротехнике издревле и по сей день угол "фи" - это угол сдвига фаз между током и напряжением, а "косинус "фи" - важнейший показатель эффективности силовых (и не только) электроустройств.

2.
Разумеется, Ri - это не линия, не гипотенуза, и не угол.
На рис. АВ-гипотенуза, АС и ВС - катеты.



Ri=ΔUa/ΔIa=АС/BC
То есть, графически Ri - это отношение длины катета АС ,прилежащего к углу α (альфа), к длине противолежащего катета ВС.

В математике указанное соотношение катетов называется КОТАНГЕНС угла α, т.е.:
Ri=ctgα

3. Про R0.
R0 не является параметрои лампы. Это всего лишь отношение напряжения и тока покоя анода, выбранных нами из своих соображений
R0=Ua0/Iа0.
А поэтому оно связано с работой в динамическом режиме весьма косвенно.
Оно просто показывает место точки покоя на графике.
И не надо его привязывать ни к внутреннему, ни к нагрузочному , вообще ни к чему. Это всего лишь начальная точка построения на графике. И не входит ни в какую формулу при расчёте.

Спасибо: 0 
Профиль
Плюмбум





Пост N: 277
Зарегистрирован: 14.12.16
Рейтинг: 2
ссылка на сообщение  Отправлено: 02.11.19 15:24. Заголовок: Друзья, а я разгадал..


Друзья, а я разгадал великую тайну Редстара!

Выбираем Ua0 и РТ. Строим мрямоугольник слева внизу. Проводим в нём диагональ.
Копируем полученную фигуру, переворачиваем её снизу вверх, и помещаем наверху.
Эту же фигуру переворачиваем слева направо и помещаем внизу справа.
Получили именно то, о чём РедСтар нам втолковывал, а мы не смогли дотумкать.



Вот оно! И никакой векторной алгебры не надо, мечта РедСтара булет воплощена!
и Ro=Rа (по углу наклона) !!!
и Ra поделена строго пополам точкой РТ! Красота!

Аплодисменты!
Редстарыч, Нобелевку - пополам!

Спасибо: 0 
Профиль
XMR



Не зарегистрирован
Зарегистрирован: 01.01.70
ссылка на сообщение  Отправлено: 02.11.19 18:42. Заголовок: Круто!!!..


Круто!!!

Спасибо: 0 
Пермяк
moderator




Пост N: 6398
Зарегистрирован: 15.01.10
Откуда: Россия
Рейтинг: 25
ссылка на сообщение  Отправлено: 02.11.19 20:01. Заголовок: XMR Посчитал с приме..


Посчитал с применением "метода Плюмбума-РедСтара".



Спасибо: 0 
Профиль
RedStar
постоянный участник




Пост N: 2581
Зарегистрирован: 02.08.16
Откуда: Смоленск
Рейтинг: 0
ссылка на сообщение  Отправлено: 02.11.19 23:01. Заголовок: Плюмбум пишет: я ра..


Плюмбум пишет:
 цитата:
я разгадал великую тайну

Если бы так было просто. Ну а как, если Ra другое?
Хотя подумали верно. С этого я начинал. Продолжить желаете?

Пермяк пишет:
 цитата:
Разумеется, Ri - это не линия, не гипотенуза, и не угол.

Немного добавил к рисунку, не возражаете?
Очевидно, что треугольник O, OP, F (Ro) вписан в треугольник O, D, E (Ra). Поэтому расчеты более корректны, чем расчеты с отстоящим отдельно треугольником А, В, С (Ri).
Дело не в треугольниках, а в их векторах.



Пермяк пишет:
 цитата:
Посчитал с применением "метода Плюмбума-РедСтара".

Без моего имени Никнейма. Спасибо.

Не надо иметь много колес, лучше иметь много спиц, удерживающих одно колесо. © Спасибо: 0 
Профиль
Ученик



Пост N: 902
Зарегистрирован: 06.04.13
Рейтинг: 2
ссылка на сообщение  Отправлено: 02.11.19 23:44. Заголовок: RedStar пишет: Очеви..


RedStar пишет:
 цитата:
Очевидно, что треугольник O, OP, F (Ro) вписан в треугольник O, D, E (Ra).

Треугольник вписан в треугольник?
 цитата:
Поэтому расчеты более корректны, чем расчеты с отстоящим отдельно треугольником А, В, С (Ri).

Для твоих расчётов Теорема Пифагора видоизменяется???
 цитата:
Не приписывайте мне того, чего не говорю.

Тогда потрудись объяснить, каким образом у тебя получается так, что для треугольника O, OP, F расчёты якобы точней, чем для треугольника А, В, С (которых, к слову, на твоём чертёжике два).
 цитата:
Просто не хотите понять метод

Просто ты вообще ничего объяснить не можешь. Что у тебя с изложением мыслей - просто кошмар. Ты до сих пор не смог объяснить, что же ты вычисляешь в своём "МЕТОДЕ".
Ты можешь собраться, и написать: я определяю (вычисляю) это, это, и это? Осилишь?

Учиться никогда не поздно, и никому не вредно. Спасибо: 0 
Профиль
Пермяк
moderator




Пост N: 6399
Зарегистрирован: 15.01.10
Откуда: Россия
Рейтинг: 25
ссылка на сообщение  Отправлено: 03.11.19 05:56. Заголовок: Ну, ладно. Не может ..


Жаль, что RedStar упорно не желает объяснить своих действий, попробую обратиться с наводящими вопросами.

Анатолий, я немного изменил твой очередной рисунок. Для удобства.
Треугольник АВС сделал побольше, для удобства рассмотрения. Вправе это сделать потому, что угол α не изменился, а значит, не изменилось и Ri, т.к.
Ri=ctg(α)



Ты пишешь:
 цитата:
Очевидно, что треугольник O, Oр, F вписан в треугольник O, D, E

Ну, допустим, "вписан" , но почему из этого следует, что:
 цитата:
Поэтому расчеты более корректны, чем расчеты с отстоящим отдельно треугольником А, В, С (Ri).

???
Мы, однако, не видели здесь таких расчётов, увы. И эта странная фраза:
 цитата:
Дело не в треугольниках, а в их векторах.

О каких векторах речь? У треугольника есть три стороны и три угла. Если это прямоугольный треугольник, то его стороны называют:катеты и гипотенуза.
Ну, можно, конечно, составить треугольник из векторов, в электротехнике это делают часто, но у тебя-то нет на рисунках никаких векторов.
И ты ещё сердишься...

Спасибо: 1 
Профиль
Плюмбум





Пост N: 278
Зарегистрирован: 14.12.16
Рейтинг: 2
ссылка на сообщение  Отправлено: 03.11.19 13:11. Заголовок: Вот что пришло в гол..


Вот что пришло в голову.
1. Редстар настойчиво повторяет, что Ri ему не нужно. Убираю с картинки характеристический треугольник.
2. На его рисунке между линиями, обозначенными как Ro и Ra - ПРЯМОЙ УГОЛ!
"Палочки должны быть попиньдикулярны!" (с)



Мне кажется, что на картинке высвечивается что-то до боли знакомое... Уж не подобие ли прямоугольных треугольников ?

Спасибо: 0 
Профиль
RedStar
постоянный участник




Пост N: 2583
Зарегистрирован: 02.08.16
Откуда: Смоленск
Рейтинг: 0
ссылка на сообщение  Отправлено: 03.11.19 16:53. Заголовок: Пермяк пишет: И ты ..


Пермяк пишет:
 цитата:
И ты ещё сердишься...

Сержусь на оскорбительные выпадки. Да и на себя, за "корявый" технический.

Ученик пишет:
 цитата:
Ты можешь собраться, и написать: я определяю (вычисляю) это, это, и это? Осилишь?

Насколько смогу.

Пермяк пишет:
 цитата:
но почему из этого следует, что:

Ключевая точка - Ор.
O, Op, F с вершиной Ор делит треугольник ODE на O, Op, E. Катет O, Op - Ro. Катет Op, E - Ra.
Известно только O, F. Добавляем точку на оси ординат "I". Получаем прямоугольник O, I, Op, F. Из него вычисляем периметр О, Ор (гипотенузу).
F/I=Op (Ro)
Достраиваем противоположный прямоугольник F, Op, I1, E. Так как точка E неизвестна, то неизвестна вершина E.
Заладим по оси абсцисс необходимую величину, примерно = 20 (по графикам
http://hiend.borda.ru/?1-22-1572769330015-00000544-000-60-0#039)Выбранная величина соответствует точке J. Прямая I=I1.
FJ/I=DE (Ra) полному отрезку или ее части, стремящейся к Е (не важно).
(Ro+Ra)*I=E
Если уже задано (выбрано) значение Е конструктивными особенностями, то выбирая наиболее подходящую РТ будет намного проще.
(F/I)+(E/I)*I=E
По оси ординат (анодному току) сложнее расчет. Главное - хотя бы примерно определить конечные точки амплитуды.
Об этом позже.



Пока что так, как мог.

Плюмбум пишет:
 цитата:
Уж не подобие ли прямоугольных треугольников ?

Нет. Ro не обязательно должно делить угол по 90 градусов отрезка DE.

Не надо иметь много колес, лучше иметь много спиц, удерживающих одно колесо. © Спасибо: 0 
Профиль
r9o-11





Пост N: 606
Зарегистрирован: 28.01.15
Откуда: Искитим, НСО
Рейтинг: 3
ссылка на сообщение  Отправлено: 03.11.19 17:51. Заголовок: А откуда известна то..


А откуда известна точка I1, если E неизвестна? Для чего нужна I1?
И что такое "20"? По ссылке это число не упоминается. Или это амплитуда напряжения?
Ну и, наверное, "...вычисляем пАрАметр О, Ор (гипотенузу)...", потому как у линии не может быть периметра. Если параметр, то наверное, это длина в каких-то единицах?

Андрей Спасибо: 0 
Профиль
RedStar
постоянный участник




Пост N: 2585
Зарегистрирован: 02.08.16
Откуда: Смоленск
Рейтинг: 0
ссылка на сообщение  Отправлено: 03.11.19 18:04. Заголовок: r9o-11 пишет: А отк..


r9o-11 пишет:
 цитата:
А откуда известна точка I1

Она - на вертикали, проведённой из точки Е. А точка Е - на пересечении линии Ra c осью абсцисс.
 цитата:
если E неизвестна

Найти Е можно определившись Ор и требуемой длины FJ (выбранной выходной амплитуды напряжения) по этой же оси абсцисс.
Соответственно, катет FE - будет значением падения напряжения.
И, как известно:
FE/I=Ra
 цитата:
потому как у линии не может быть периметра...

Не Линия, а периметр прямоугольника.

Не надо иметь много колес, лучше иметь много спиц, удерживающих одно колесо. © Спасибо: 0 
Профиль
RedStar
постоянный участник




Пост N: 2586
Зарегистрирован: 02.08.16
Откуда: Смоленск
Рейтинг: 0
ссылка на сообщение  Отправлено: 03.11.19 18:04. Заголовок: r9o-11 пишет: А отк..


r9o-11 пишет:
 цитата:
А откуда известна точка I1

Она известна по всей оси абсцисс. Так как это прямая линия I - Iбесконечность

 цитата:
если E неизвестна

Читайте внимательно. Найти Е можно определившись Ор и требуемой длины FJ (выбранной выходной амплитуды напряжения) по этой же оси абсцисс.
Соответственно, катет F, E - будет значением падения напряжения.
И, как известно:
FE/I=Ra
 цитата:
потому как у линии не может быть периметра...

Не Линия, а периметр прямоугольника.

Не надо иметь много колес, лучше иметь много спиц, удерживающих одно колесо. © Спасибо: 0 
Профиль
r9o-11





Пост N: 607
Зарегистрирован: 28.01.15
Откуда: Искитим, НСО
Рейтинг: 3
ссылка на сообщение  Отправлено: 03.11.19 18:08. Заголовок: Так я, вроде читаю: ..


Так я, вроде читаю:RedStar пишет:
 цитата:
Достраиваем противоположный прямоугольник F, Op, I1, E. Так как точка E неизвестна, то неизвестна вершина E.

Где логика последовательности принятия решений?
Как Вы всё любите запутывать... Да читая ответы ничего понятнее не становится... Ну как так-то?

Андрей Спасибо: 0 
Профиль
RedStar
постоянный участник




Пост N: 2587
Зарегистрирован: 02.08.16
Откуда: Смоленск
Рейтинг: 0
ссылка на сообщение  Отправлено: 03.11.19 18:23. Заголовок: r9o-11 пишет: Где л..


r9o-11 пишет:
 цитата:
Где логика последовательности принятия решений?

Определение периметра Op, E из прямоугольника F, Op, I1, E. Которая является Ra.
Сумма двух векторов O, Op и Op, E помноженная на "ключевую" вершину Ор даст координату точки Е. (что то коряво написал...)
Здесь не хватает точек по оси ординат. Если перенести данные на ось ординат с оси абсцисс, но применяя значения оси ординат, то получим результат по оси ординат.
То есть величины для анодного тока, которые мы сами определяем, как и величины анодного напряжения по оси абсцисс.
 цитата:
Как Вы всё любите запутывать...

Не запутываю. Это математика... Не все знаю про вектора (еще только учусь). Думаю, что теперь принцип понятен.
Можно далее приступать для обобщения с применением коэффициентами изменения тока анода и напряжения анода при изменении сеточного напряжения.
Может тогда станет более понятно. Хотя и так явно видно основу методики.

Так как же влияет Ri на Ra? (не надо сюда добавлять Альфу, о ней можно тоже поспорить отдельно).

Не надо иметь много колес, лучше иметь много спиц, удерживающих одно колесо. © Спасибо: 0 
Профиль
r9o-11





Пост N: 608
Зарегистрирован: 28.01.15
Откуда: Искитим, НСО
Рейтинг: 3
ссылка на сообщение  Отправлено: 03.11.19 18:35. Заголовок: Извините, я основу м..


Извините, я основу методики не увидел и принципа не понял, так как не понял по каким правилам происходит сложение и перемножение значений...
Погорячился спрашивать. Зайду ещё раз через полгода-год...

Андрей Спасибо: 0 
Профиль
RedStar
постоянный участник




Пост N: 2588
Зарегистрирован: 02.08.16
Откуда: Смоленск
Рейтинг: 0
ссылка на сообщение  Отправлено: 03.11.19 18:57. Заголовок: r9o-11 пишет: я осн..


r9o-11 пишет:
 цитата:
я основу методики не увидел и принципа не понял

Это ваше право. Основа по ссылке выше на этой странице. Принцип изложил, хоть и своими словами.
Это только начало анализа методики, применительно к резистивному каскаду
 цитата:
Зайду ещё раз через полгода-год...

Добро пожаловать потом.

Не надо иметь много колес, лучше иметь много спиц, удерживающих одно колесо. © Спасибо: 0 
Профиль
Пермяк
moderator




Пост N: 6402
Зарегистрирован: 15.01.10
Откуда: Россия
Рейтинг: 25
ссылка на сообщение  Отправлено: 03.11.19 19:14. Заголовок: RedStar пишет: Это т..


RedStar пишет:
 цитата:
Это только начало анализа методики, применительно к резистивному каскаду .



Хорошо начинаете, Анатолий.
Но продолжать не надо. Хватит с Вас...

Завязывайте.

Спасибо: 0 
Профиль
RedStar
постоянный участник




Пост N: 2590
Зарегистрирован: 02.08.16
Откуда: Смоленск
Рейтинг: 0
ссылка на сообщение  Отправлено: 03.11.19 19:34. Заголовок: Хорошо. Не раз говор..


Хорошо. Не раз говорил, что не поймете.
А воз и ныне там. В прошлом, в болоте.

П.С. Еще. Почему пентод в резистивном каскаде, не зашунтированным конденсатором катодного резистора, не приобретает свойства триода (по ВАХ)?
Как в выходном каскаде с Местной ОС.

Не надо иметь много колес, лучше иметь много спиц, удерживающих одно колесо. © Спасибо: 0 
Профиль
Пермяк
moderator




Пост N: 6404
Зарегистрирован: 15.01.10
Откуда: Россия
Рейтинг: 25
ссылка на сообщение  Отправлено: 03.11.19 19:44. Заголовок: У пентода очень высо..


У пентода очень высокое Ri (десятки - до сотни кОм. Резистор в катоде образует ПОСЛЕДОВАТЕЛЬНУЮ ООС ПО ТОКУ. Такая ООС увеличивает и без того высокое внутреннее пентода. Конденсатор в катоде, устранив эту ООС, только сохраняет исходные пентодные свойства.

Спасибо: 0 
Профиль
RedStar
постоянный участник




Пост N: 2591
Зарегистрирован: 02.08.16
Откуда: Смоленск
Рейтинг: 0
ссылка на сообщение  Отправлено: 03.11.19 19:56. Заголовок: Пермяк пишет: Такая..


Пермяк пишет:

 цитата:
Такая ООС увеличивает и без того высокое внутреннее пентода.


Тоже самое и в выходном каскаде с обмоткой КО, где ее сопротивление меняется от частоты.
Пентод остается пентодом. И никакие изменения, по ВАХ в триодные, не происходит.
Только не надо вводить Ri сюда. Это статический параметр лампы. Как неоднократно говорили на форуме, что РТ, по постоянке, не изменяется. С чего оно должно "плавать"?
Из-за нестабильности питающего напряжения, коими тогда являлись кенотронами?
Ri возможно только при повышении напряжения анода с повышением тока анода, или наоборот.
Что не может соответствовать по нагрузочной Ra.
Скажите, что это бред. И на этом закончим.

Не надо иметь много колес, лучше иметь много спиц, удерживающих одно колесо. © Спасибо: 0 
Профиль
Пермяк
moderator




Пост N: 6405
Зарегистрирован: 15.01.10
Откуда: Россия
Рейтинг: 25
ссылка на сообщение  Отправлено: 03.11.19 20:04. Заголовок: RedStar пишет: То же..


RedStar пишет:
 цитата:
То же самое и в выходном каскаде с обмоткой КО, где ее сопротивление меняется от частоты.

КО - это последовательная ООС ПО НАПРЯЖЕНИЮ, такая ООС снижвет внутренне лампы. Влияние Rакт КО здесь намного меньше (в десятки раз меньше, чем ООС с КО.

RedStar пишет:
 цитата:
Ri возможно только при повышении напряжения анода с повышением тока анода, или наоборот.

Сам-то подумал, что такое написал?

Спасибо: 0 
Профиль
RedStar
постоянный участник




Пост N: 2592
Зарегистрирован: 02.08.16
Откуда: Смоленск
Рейтинг: 0
ссылка на сообщение  Отправлено: 03.11.19 20:18. Заголовок: Пермяк пишет: КО - ..


Пермяк пишет:
 цитата:
КО - это последовательная ООС ПО НАПРЯЖЕНИЮ, такая ООС снижвет внутренне лампы.

Допустим, КО имеет индуктивное к сопротивление, не активное. Они последовательно соединены Ri+Zk.
Как же снижается внутреннее у выходного каскада, а у резистивного, - нет?

Не надо иметь много колес, лучше иметь много спиц, удерживающих одно колесо. © Спасибо: 0 
Профиль
Пермяк
moderator




Пост N: 6406
Зарегистрирован: 15.01.10
Откуда: Россия
Рейтинг: 25
ссылка на сообщение  Отправлено: 03.11.19 20:58. Заголовок: КО -часть первички, ..


КО -часть первички, перенесённая в катод. ЧЕРЕЗ неё идёт полный анодный ток. Этот ток создаёт на катоде переменное напряжение, которое прикладывается К СЕТКЕ.
ООС по напряжению понижает Ri лампы.
Индуктивное сопротивление ВСЕЙ первички должно быть достаточно велико, чтобы исключить заметное снижение отдачи на НЧ.
КО включена в катод. Со стороны катода лампа имеет очень небольшое входное сопротивление, которое шунтирует катодную часть первички по ПЕРЕМЕНКЕ, и индуктивность КО не играет рояли.

А в резистивном каскаде, как я и написАл - ПОСЛЕДОВАТЕЛЬНАЯ ООС ПО ТОКУ. Научитесь же, наконец, отличать, в чём разница видов ООС и их действие.
Трудно с Вами.

Спасибо: 0 
Профиль
XMR



Пост N: 12
Зарегистрирован: 02.11.19
Рейтинг: 0
ссылка на сообщение  Отправлено: 03.11.19 21:44. Заголовок: RedStar пишет: Еще...


RedStar пишет:
 цитата:
Еще. Почему пентод в резистивном каскаде, не зашунтированным конденсатором катодного резистора, не приобретает свойства триода (по ВАХ)?
Как в выходном каскаде с Местной ОС.

Потому что это ОС по току , он еще пентоднее становится.

Спасибо: 0 
Профиль
XMR



Пост N: 13
Зарегистрирован: 02.11.19
Рейтинг: 0
ссылка на сообщение  Отправлено: 03.11.19 21:49. Заголовок: Оооооо , дочитал до ..


Оооооо , дочитал до конца, так у Редстара полное непонимание как работает ОС и КО про которую он пишет, да какая там уж методика, плакать надо.

Спасибо: 0 
Профиль
RedStar
постоянный участник




Пост N: 2594
Зарегистрирован: 02.08.16
Откуда: Смоленск
Рейтинг: 0
ссылка на сообщение  Отправлено: 03.11.19 22:12. Заголовок: Пермяк пишет: КО -ч..


Пермяк пишет:
 цитата:
КО -часть первички, перенесённая в катод...

Так. А в резистивном не течет полный анодный ток?

Не надо иметь много колес, лучше иметь много спиц, удерживающих одно колесо. © Спасибо: 0 
Профиль
XMR



Пост N: 15
Зарегистрирован: 02.11.19
Рейтинг: 0
ссылка на сообщение  Отправлено: 03.11.19 22:38. Заголовок: Редстар неужели ты н..


Редстар неужели ты не знал что резистор в катоде - это ос по току, а ко - по напряжению? я вообще как-то в шоке теперь...

Вот если в катод поставить дроссель вместо резистора, то будет ос по току на индуктивности при переменном сигнале не увеличив RI при этом, а когда обмотка КО находится на одном железе с анодной и по ней течет анодный ток, то это ОС по напряжению и только по нему.

Спасибо: 0 
Профиль
RedStar
постоянный участник




Пост N: 2595
Зарегистрирован: 02.08.16
Откуда: Смоленск
Рейтинг: 0
ссылка на сообщение  Отправлено: 03.11.19 22:59. Заголовок: XMR пишет: резистор..


XMR пишет:
 цитата:
резистор это ос по току а ко по напряжению

В любом случае, через КО и катодный резистор течет один ток. Разве не так?
Почему лампа имеет ток покоя не связанный с Ri, а связанный с P - (мощность рассеяния анода), где как раз присутствует Ro?

Не надо иметь много колес, лучше иметь много спиц, удерживающих одно колесо. © Спасибо: 0 
Профиль
XMR



Пост N: 17
Зарегистрирован: 02.11.19
Рейтинг: 0
ссылка на сообщение  Отправлено: 03.11.19 23:13. Заголовок: RedStar пишет: В лю..


RedStar пишет:
 цитата:
В любом случае, через КО и катодный резистор течет один ток. Разве не так

По переменному току части анодной и катодной обмотки включены паралельно и находясь на одном железе находятся в тесной индуктивной связи, ну то есть они включены ПАРАЛЕЛЬНО.
А что бы включить ПОСЛЕДОВАТЕЛЬНО по переменному току нужно в катод как я выше сказал поставить отдельный независимый дроссель. на отдельном железе.

Спасибо: 0 
Профиль
volli





Пост N: 791
Зарегистрирован: 16.05.10
Откуда: Псков
Рейтинг: 2
ссылка на сообщение  Отправлено: 03.11.19 23:29. Заголовок: XMR пишет: Это же н..


RedStar пишет:
 цитата:
Это же насколько надо быть уверенным в непоколебимости основ прошлого века?

Появились сомнения на счёт "закона дедушки Ома"?

Говори что думаешь. Думай, что говоришь! Спасибо: 0 
Профиль
RedStar
постоянный участник




Пост N: 2596
Зарегистрирован: 02.08.16
Откуда: Смоленск
Рейтинг: 0
ссылка на сообщение  Отправлено: 03.11.19 23:31. Заголовок: XMR пишет: по перем..


XMR пишет:
 цитата:
по переменному току части анодной и катодной обмотки включены параллельно

Кто нарисует эквивалентную схему этого?
Не могу понять, почему два последовательных сопротивления Ri+Zk являются параллельными?
В резистивном отсутствует связь между ними. В трансформаторном - индуктивная связь, но по току такая же.
По напряжению? Как считать, Анод-катод, - это одно. Анод-земля, - другое. В первом случае амплитуды складываются, во втором - вычитаются.

П.С. Чувствую, не понимаете меня. Что же, пора остановиться, совсем.
Будьте добры друг-другу!
Спасибо всем.

Не надо иметь много колес, лучше иметь много спиц, удерживающих одно колесо. © Спасибо: 0 
Профиль
volli





Пост N: 792
Зарегистрирован: 16.05.10
Откуда: Псков
Рейтинг: 2
ссылка на сообщение  Отправлено: 03.11.19 23:45. Заголовок: RedStar пишет: Допу..


RedStar пишет:
 цитата:
Допустим, КО имеет индуктивное к сопротивление, не активное.


 цитата:
В любом случае, через КО и катодный резистор течет один ток. Разве не так?


 цитата:
Не могу понять, почему два последовательных сопротивления Ri+Zk являются параллельными?

Не может ли Анатолий пояснить, какой ток в каждом из случаев он имеет в виду?
Нет ли путаницы в понятиях разности реакции различных нагрузок на переменный и постоянный ток?
Не отсюда ли весь "геморой"?

Говори что думаешь. Думай, что говоришь! Спасибо: 0 
Профиль
XMR



Пост N: 18
Зарегистрирован: 02.11.19
Рейтинг: 0
ссылка на сообщение  Отправлено: 03.11.19 23:51. Заголовок: Ред стар ты реально ..


Редстар ты реально глуп, без обид. глуп не тот кто не понимает а тот кто не хочет понимать.

Спасибо: 0 
Профиль
XMR



Пост N: 19
Зарегистрирован: 02.11.19
Рейтинг: 0
ссылка на сообщение  Отправлено: 03.11.19 23:53. Заголовок: Как ты не можешь пон..


Как ты не можешь понять что обмотки ко и анодная на одном железе, а лампа имеет RI которое ты не признаёшь и вот именно из-за этого RI обмотки включены паралельно и между ними есть индуктивная связь по типу трансформатора.

Спасибо: 0 
Профиль
RedStar
постоянный участник




Пост N: 2598
Зарегистрирован: 02.08.16
Откуда: Смоленск
Рейтинг: 0
ссылка на сообщение  Отправлено: 04.11.19 00:28. Заголовок: XMR пишет: вот имен..


XMR пишет:
 цитата:
вот именно из-за этого RI обмотки включены паралЛельно

Как они параллельны? Нарисуйте график, эквивалентную схему. Подробно может кто объяснить?
Не надо мне говорить о выходном сопротивлении и Альфе. Все притянуто формулами.
Что это за Ri, и почему она нужна?
Ну не вижу связи, совсем.

Не надо иметь много колес, лучше иметь много спиц, удерживающих одно колесо. © Спасибо: 1 
Профиль
Ученик



Пост N: 906
Зарегистрирован: 06.04.13
Рейтинг: 2
ссылка на сообщение  Отправлено: 04.11.19 01:03. Заголовок: XMR пишет: Редстар т..


XMR пишет:
 цитата:
Редстар ты реально глуп, без обид. глуп не тот кто не понимает а тот кто не хочет понимать.

Ну вот, убедились, что я не придираюсь к редстару? Он в очередной раз не смог пояснить, что же он рассчитывает - токи, напряжения, нагрузку, Ri, какие-то зависимости, или что-то другое, применимое к электротехнике. Не-а, треугольники, прямоугольники, периметры, "вектор" треугольника...

Три!!!! года РедСтар своими глупостями полощет мозги всему форуму. Сдвигов не вижу, упрям, необучаем, не понимает простых вещей, не способен изложить свои мысли. Ещё и посылает разом всех. Кто как хочет, я отправляю его в полный игнор.

Учиться никогда не поздно, и никому не вредно. Спасибо: 0 
Профиль
XMR



Пост N: 21
Зарегистрирован: 02.11.19
Рейтинг: 0
ссылка на сообщение  Отправлено: 04.11.19 01:25. Заголовок: Ученик, да я если че..


Ученик, да я если честно поражён Вашему и других Коллег терпению.

Спасибо: 0 
Профиль
RedStar
постоянный участник




Пост N: 2599
Зарегистрирован: 02.08.16
Откуда: Смоленск
Рейтинг: 0
ссылка на сообщение  Отправлено: 04.11.19 01:26. Заголовок: Ученик пишет: что ж..


Ученик пишет:
 цитата:
что же он рассчитывает

Ученик, все вы поняли. Только вам на это наплевать, честно.
Но если вам не видно что и как рассчитывается, не надо говорить за всех.
Остальные молчат. Кто же попрет против всезнающих и "уважаемых гуру"?

Можете сносить тему. ...с большущей колокольни...
Говорил же, стоит ли продолжать? Вынудили.

Самые протоптанные пути - в никуда.
Счастливого пути всем!

Не надо иметь много колес, лучше иметь много спиц, удерживающих одно колесо. © Спасибо: 0 
Профиль
aleks8845



Пост N: 614
Зарегистрирован: 07.03.16
Рейтинг: 1
ссылка на сообщение  Отправлено: 04.11.19 02:20. Заголовок: RedStar пишет: И ни..


RedStar пишет:
 цитата:
И никакие изменения, по ВАХ в триодные, не происходит.
Только не надо вводить Ri сюда.

Я так и не смог понять, речь идет об Ri лампы как о ненужном параметре при расчетах или вообще , как не нужный параметр?

Спасибо: 0 
Профиль
Aleph





Пост N: 620
Зарегистрирован: 03.06.15
Откуда: Макеевка
Рейтинг: 7
ссылка на сообщение  Отправлено: 04.11.19 03:43. Заголовок: aleks8845 пишет: Я ..


aleks8845 пишет:
 цитата:
Я так и не смог понять, речь идет об Ri лампы как о ненужном параметре при расчетах или вообще , как не нужный параметр?

Для меня даже дико не понимать, что лампа имеет внутреннее. Это же основа. Меняя потенциал на сетке мы заставляем лампу менять своё сопротивление. А как иначе сигнал образуется?
Как вообще можно быть электрическим прибором и не иметь сопротивления? Ни нулевого, ни отрицательного, никакого?
Банальный пример - низкоомный триод 6с41с и первичка ТВЗ. Ток покоя 120 мА, питание 220 Вольт. Внутренне лампы, в которое RS похоже не верит, в этом режиме около 220 Ом. Почему мы для триода пытаемся сделать сопротивление обмотки как можно ниже? Подумаешь упадёт пара вольт на омическом провода, не смертельно. Вот только почему-то по переменке образуется делитель и лишь часть усиленного сигнала идёт в нагрузку. Одним резистором является пересчитанная на ТВЗ нагрузка, ну а вторым может быть только внутреннее лампы. С высоким внутренним пентода это не так явно видно, но тоже имеет место.

Я предпочитаю доказательства вроде приведенных выше. Не надо практически никаких цифр и графиков. Приложили так сказать действие и смотрим что получится. В нашем случае сигнал себя повёл будто в схеме по переменке есть делитель. Смотрим какие в цепи есть сопротивления, если видим только одно, значит ещё какой-то элемент цепи ведёт себя как сопротивление. В цепи у нас лампа, нагрузка и очень низкоомный источник питания, почти КЗ. Думаю это доказывает существование внутреннего сопротивления лампы.
Ну а уже зная что параметр существует можно снимать его график самому, или верить готовому.

С ув. Сергей Спасибо: 0 
Профиль
RedStar
постоянный участник




Пост N: 2600
Зарегистрирован: 02.08.16
Откуда: Смоленск
Рейтинг: 0
ссылка на сообщение  Отправлено: 04.11.19 03:57. Заголовок: Aleph пишет: Как во..


Aleph пишет:
 цитата:
Как вообще можно быть электрическим прибором и не иметь сопротивления? Ни нулевого, ни отрицательного, никакого?

Я отрицаю только наличие Ri.
 цитата:
Меняя потенциал на сетке мы заставляем лампу менять своё сопротивление.

Какое сопротивление? Под нагрузкой или в процессе составления ВАХ? С постоянными напряжениями сетки или с переменными?

Приводили график, где видно, что такое Ri, Ra, Ro.
Кто в нем видит несоответствие?
Достало уже все.

Не надо иметь много колес, лучше иметь много спиц, удерживающих одно колесо. © Спасибо: 0 
Профиль
XMR



Пост N: 25
Зарегистрирован: 02.11.19
Рейтинг: 0
ссылка на сообщение  Отправлено: 04.11.19 07:15. Заголовок: RedStar пишет: Толь..


RedStar пишет:
 цитата:
отрицаю только наличие Ri

Как можно отрицать Ri, оно существует независимо от твоего желания.

Спасибо: 0 
Профиль
Пермяк
moderator




Пост N: 6409
Зарегистрирован: 15.01.10
Откуда: Россия
Рейтинг: 25
ссылка на сообщение  Отправлено: 04.11.19 14:37. Заголовок: XMR пишет: По переме..


XMR пишет:
 цитата:
По переменному току части анодной и катодной обмотки включены паралельно

Ошибаетесь, сорри...
Когда-то я уже выкладывал РедСтару этот рисунок:

Рис.1. - Каскад с КО.
Рис.2. - Точки +Ea и "земля", как известно(!) соединены ПО ПЕРЕМЕНКЕ между собой через нулевое сопротивление БП, т.е. схемка - по переменному току.
Рис.3. - та же схема, показывающая, что Wa и Wк соединены между собой ПОСЛЕДОВАТЕЛЬНО-синфазно.



Ну, а доказательство, какая там действует ООС - по напряжению или по току, параллельная или последовательная - это предмет отдельного разговора. Про это всё доходчиво написано в книге "Красный Цыкин", стр. 278-279 (мелким шрифтом), но это не для Редстара.

Спасибо: 0 
Профиль
Aleph





Пост N: 623
Зарегистрирован: 03.06.15
Откуда: Макеевка
Рейтинг: 7
ссылка на сообщение  Отправлено: 04.11.19 17:49. Заголовок: Очень помогает собст..


Очень помогает собственноручно сделать таблицу комбинаций ОС. Способ ввода,вывода, по току, или напряжению. И что происходит с входным и выходным сопротивлениями, с искажениями, как выглядит ОС на схеме (резистивный делитель скажем, или КО, УЛ отвод...)
Когда своими руками распишешь больше в голове остаётся. Держишь табличку при себе и благодаря ей не пишешь явных глупостей

С ув. Сергей Спасибо: 0 
Профиль
XMR



Пост N: 26
Зарегистрирован: 02.11.19
Рейтинг: 0
ссылка на сообщение  Отправлено: 04.11.19 19:00. Заголовок: Пермяк пишет: Ошиба..


Пермяк пишет:
 цитата:
Ошибаетесь, сорри..

Пермяк, ну я не боюсь ошибаться, изучу вопрос, вникну, спасибо за разъяснение...
Ну да, последовательно, но я имел в виду, что они на одном железе и между ними индуктивная связь имеется, или я не правильно думаю, и просто нагрузка делится так как со стороны катода внутренее лампы меньше, но получается и уменьшается как паралельное соединение двух резисторов. поправьте если гоню.

Спасибо: 0 
Профиль
Пермяк
moderator




Пост N: 6411
Зарегистрирован: 15.01.10
Откуда: Россия
Рейтинг: 25
ссылка на сообщение  Отправлено: 04.11.19 21:37. Заголовок: Aleph пишет:Очень по..


XMR, схемка №3 показывает, что нагрузка получает сумму напряжений с анодной части обмотки и с катодной. Это вообще единый трансформатор. Неделимый.
А вот в сеточную цепь отводится, конечно, только часть анодного напряжения.

Aleph пишет:
 цитата:
Очень помогает собственноручно сделать таблицу комбинаций ОС. Способ ввода,вывода, по току, или напряжению. И что происходит с входным и выходным сопротивлениями...

.Такую?



Ку и Кг во всех случаях снижаются, т.к. ОС - таки отрицательная.

Последовательная и параллельная - это по способу подачи на вход.
По напряжению и по току - по способу снятия с выхода (сигнал ОС пропорционален вых. напряженнию или пропорционален вых. току).

Спасибо: 0 
Профиль
RedStar
постоянный участник




Пост N: 2622
Зарегистрирован: 02.08.16
Откуда: Смоленск
Рейтинг: 0
ссылка на сообщение  Отправлено: 11.11.19 02:25. Заголовок: Однажды было сказано..


Однажды было сказано в теме про "Ул-резистивный драйвер":
Бокарёв Александр пишет:
 цитата:
Редстарыч, вы делаете устройство , задача которого - звучать и радовать вас звучанием. и если его параметры случайно не укладываются в общепризнанные нормы, то у вас два выхода: или довести ваше устройство до этих общепризнанных норм и слушать, убеждая себя, что это и есть настоящий звук. Либо тихо послать эти общепризнанные нормы подальше и слушать любимую музыку. Примерно так. ваш АБ.

Вот смотрю, считаю, примеряю с практикой, а вывод остается прост: "тихо послать эти общепризнанные нормы подальше".
Как то так.
Может скоро поделюсь новыми Практическими работами.

Не надо иметь много колес, лучше иметь много спиц, удерживающих одно колесо. © Спасибо: 0 
Профиль
Пермяк
moderator




Пост N: 6426
Зарегистрирован: 15.01.10
Откуда: Россия
Рейтинг: 25
ссылка на сообщение  Отправлено: 11.11.19 05:36. Заголовок: RedStar , Вы цитируе..


RedStar , Вы цитируете А.Бокарёва? Очень правильно. При этом хорошо бы Вам учесть, что Александр никогда не лез в Теорию, не пытался опровергать КНИГИ, и тем более - не выдумывал собственных теорий. Берите пример!.

Спасибо: 1 
Профиль
RedStar
постоянный участник




Пост N: 2624
Зарегистрирован: 02.08.16
Откуда: Смоленск
Рейтинг: 0
ссылка на сообщение  Отправлено: 11.11.19 17:45. Заголовок: Пермяк пишет: Алекс..


Пермяк пишет:
 цитата:
Александр никогда не лез в Теорию ... не выдумывал собственных теорий

Как же без Теории, методом подбора конструировать? Да кто же признается, что есть у каждого свои методы?

Не надо иметь много колес, лучше иметь много спиц, удерживающих одно колесо. © Спасибо: 0 
Профиль
Фурман



Пост N: 101
Зарегистрирован: 09.01.18
Рейтинг: -3
ссылка на сообщение  Отправлено: 02.12.19 04:28. Заголовок: RedStar пишет: Сколь..


RedStar пишет:
 цитата:
Сколько сталкиваюсь, столько прихожу к мнению, что знать Ri нет необходимости.

и здесь же RedStar пишет:
 цитата:
Как же без Теории

В связи с этим, хочется задать такой вопрос:

Имеем Усилитель SE на 6п14п-Е, с переключателем "пентод/триод" подключением экрана к аноду.
Питание 250в, фикс-смещение -6,4в, ток покоя 48мА, ТВЗ 5кОм/8ом.

Вопрос Редстару: почему в пентоде выходное сопротивление усилителя равно 48 ом, а в триоде 2,4 ома?

Спасибо: 0 
Профиль
RedStar
постоянный участник




Пост N: 2789
Зарегистрирован: 02.08.16
Откуда: Смоленск
Рейтинг: 1
ссылка на сообщение  Отправлено: 03.12.19 15:01. Заголовок: Начну со встречного ..


Начну со встречного вопроса. Как проводили измерение? Опишите подробнее.
На макетке могу повторить с этой же лампой.

Не надо иметь много колес, лучше иметь много спиц, удерживающих одно колесо. © Спасибо: 0 
Профиль
Фурман



Пост N: 106
Зарегистрирован: 09.01.18
Рейтинг: -3
ссылка на сообщение  Отправлено: 03.12.19 16:05. Заголовок: Измерение проводилос..


Измерение проводилось методом двух нагрузок, 6 и 9 Ом - в триодном включении, 6 и 12 Ом - в пентодном, на частоте 600 Герц.
ТВЗ имел Ктр= 25, 5к/8ом с учётом активных сопротивлений обмоток.
https://postimg.cc/Vd4JD8MR

Если захотите повторить - можете применить любой ТВЗ. Питание и смещение тоже большого значения не имеют, главное - чтобы они были одинаковыми при триоде и при пентоде.
Главное здесь - сравнить результаты полученных Rвых, а не их точность.

Добавлю: и разумеется без всяких ООС !!!

Спасибо: 0 
Профиль
RedStar
постоянный участник




Пост N: 2790
Зарегистрирован: 02.08.16
Откуда: Смоленск
Рейтинг: 1
ссылка на сообщение  Отправлено: 03.12.19 16:17. Заголовок: Эту методику хорошо ..


Эту методику хорошо знаю. Вообще то нужно дополнение.
1. Вторичная обмотка была одна для этих типов нагрузки, без отводов?
2. Какое напряжение раскачки в обоих включениях?
3. Анодное амплитудное напряжения в этих же случаях.
4. Почему в пентоде 12 Ом, а не 9, как в триодном? От этого тоже зависит.
5. Выходные напряжения под нагрузками во всех подключениях.
Всё это очень важно.
Иначе разговор окажется тупиковым.

Не надо иметь много колес, лучше иметь много спиц, удерживающих одно колесо. © Спасибо: 0 
Профиль
Фурман



Пост N: 108
Зарегистрирован: 09.01.18
Рейтинг: -3
ссылка на сообщение  Отправлено: 03.12.19 16:52. Заголовок: RedStar пишет: 1. Вт..


RedStar пишет:
 цитата:
1. Вторичная обмотка была одна для этих типов нагрузки, без отводов?

Разумеется. ТВЗ - 5к/8ом, сколько же можно повторять?
 цитата:
2. Какое напряжение раскачки в обоих включениях?

Раскачка была разной для пентода и триода, но выходное напряжение было 2 вольта, в обоих случаях. Но можно любое, важно, чтобы было одинаковое. Разумеется - не на полную раскачку.
 цитата:
3. Анодное амплитудное напряжения в этих же случаях.

Анодное не измерял, только выходное. Мы же выходное сопротивление измеряем..
 цитата:
4. Почему в пентоде 12 Ом, а не 9, как в триодном? От этого тоже зависит.

Не 12, а 6 и 12, т.е по обе стороны от номинальных 8-ми. Но и это тоже не обязательно.
 цитата:
5. Выходные напряжения под нагрузками во всех подключениях.

А как же без нагрузки-то, милый мой?

Выходное сопротивление - это величина, характеризующая реакцию выходной цепи на изменение нагрузки.

Спасибо: 0 
Профиль
RedStar
постоянный участник




Пост N: 2791
Зарегистрирован: 02.08.16
Откуда: Смоленск
Рейтинг: 1
ссылка на сообщение  Отправлено: 03.12.19 17:28. Заголовок: Фурман пишет: Разум..


Фурман пишет:
 цитата:
Разумеется. ТВЗ - 5к/8ом,

Значит Ктр изменяется. Далее,...
При 6 Ом нагрузки должно получиться 3750. При 9 Ом - 5625...
Ток лампы то больше, то меньше. Амплитуда Iвторички/Ктр=Iпервички. Само собой выходное напряжение будет изменяться.
 цитата:
Раскачка была разной для пентода и триода,

- разной!? Подробнее напишите, это интересно ведь, не правда ли почему?
 цитата:
но выходное напряжение было 2 вольта, в обоих случаях.

Как это? Вы же пишете, что вых. сопрот. значительно отличается, но сейчас 2 вольта в обоих случаях?
 цитата:
Но можно любое, важно, чтобы было одинаковое.

Конечно важно. Как же получили тогда разное сопротивление?
 цитата:
Анодное не измерял,

Вот это плохо. Так как оно будет отличаться, которое зависит от типа включения лампы. Тогда анодная нагрузка будет совсем другой.
Но у вас, в трафе, неизменный Ктр, т.е. количество витков.
 цитата:
Мы же выходное сопротивление измеряем..

Да. Только не учли мною выше приведенное.
 цитата:
Не 12, а 6 и 12, т.е по обе стороны от номинальных 8-ми. Но и это тоже не обязательно.

Как это не обязательно? А Ктр
 цитата:
Выходное сопротивление - это величина, характеризующая реакцию выходной цепи на изменение нагрузки.

В таком случае, лучше подсоединить динамик.
Не учли то, что нагрузка должна быть ориентирована под свое Ктр.

Прошу прощения, но это не верные и не полные измерения, вами предоставленные.
П.С. Простите за мой каламбур. Надеюсь, смысл понятен, хоть немного.

Не надо иметь много колес, лучше иметь много спиц, удерживающих одно колесо. © Спасибо: 0 
Профиль
Фурман



Пост N: 109
Зарегистрирован: 09.01.18
Рейтинг: -3
ссылка на сообщение  Отправлено: 03.12.19 18:00. Заголовок: На,п рочитай ещё раз..


На, прочитай ещё раз, и делай, как написано !!!



Спасибо: 0 
Профиль
RedStar
постоянный участник




Пост N: 2792
Зарегистрирован: 02.08.16
Откуда: Смоленск
Рейтинг: 1
ссылка на сообщение  Отправлено: 03.12.19 18:09. Заголовок: Не надо НАкать. На з..


Не надо НАкать.
На заборе тоже написано, но не означает, что это там есть.
Почему же вы не принимаете во внимание другие факторы, о которых вас спрашиваю?
Скажете зачем?
Еще раз прочтите, что от вас требуется для измерения, что бы состоялся правильный и адекватный разговор.

П.С. Я могу извиниться, если будут доказательства правильности измерения.
Но пока не вижу этого.

Не надо иметь много колес, лучше иметь много спиц, удерживающих одно колесо. © Спасибо: 0 
Профиль
r9o-11





Пост N: 672
Зарегистрирован: 28.01.15
Откуда: Искитим, НСО
Рейтинг: 4
ссылка на сообщение  Отправлено: 03.12.19 18:13. Заголовок: А ничего, что акусти..


А ничего, что акустика обычно не только имеет реактивное сопротивление (т.е., разное на разных частотах), но и указанные на ней значения 2,4,6,8 или 16 Ом - это примерные значения.
Тестером когда измеряете, что показывает?

Андрей Спасибо: 0 
Профиль
Пермяк
moderator




Пост N: 6476
Зарегистрирован: 15.01.10
Откуда: Россия
Рейтинг: 25
ссылка на сообщение  Отправлено: 03.12.19 18:20. Заголовок: Фурман , я тебя пред..


Фурман , я тебя предупреждал: пациент безнадёжен...

RedStar пишет:
 цитата:
На заборе тоже написано, но не означает, что это там есть.

Замечательный аргумент! Приравнивать надписи на заборе к статьям в технической литературе - Ваш конёк.
Зачем вообще пишете на форуме, и зачем здесь задаёте вопросы?

r9o-11 пишет:
 цитата:
акустика обычно не только имеет реактивное сопротивление (т.е., разное на разных частотах), но и указанные на ней значения 2,4,6,8 или 16 Ом - это примерные значения.

Мы этого не понимаем! Выдумки всё это. Написано 8 Ом - значит так оно и есть!

Спасибо: 0 
Профиль
RedStar
постоянный участник




Пост N: 2793
Зарегистрирован: 02.08.16
Откуда: Смоленск
Рейтинг: 1
ссылка на сообщение  Отправлено: 03.12.19 19:06. Заголовок: r9o-11 пишет: А нич..


r9o-11 пишет:
 цитата:
А ничего, что акустика обычно не только имеет реактивное сопротивление

Так выше указал, подсоедините дин и посмотрите. С учетом фильтра колонок, где выравнивается импеданс, нагрузка должна незначительно меняться

Пермяк пишет:
 цитата:
пациент безнадёжен...

С чего это?
Продолжу. Если траф рассчитан на одну нагрузку, то зачем ему "пихать" другую.
И, прежде, чем отвечу, хотелось бы увидеть измерения.
Вы, и практически все на форуме, толкуете, что надо придерживаться одного. Соответственно, под другую нагрузку, либо делать отводы вторички, либо перематывать траф.
Сложно сделать отводы и измерить с ними под каждую им предназначенную нагрузку?
Конечно, результаты не особо обрадуют, так как вых. сопр. не особо то изменится.
 цитата:
Мы этого не понимаем!

Как сказать..., а додумать сложно видимо.
Не торопитесь с выводами.

П.С. Подожду, что ответит Фурман. Интересно все же узнать, какие у него выходные данные при неизменных входных!!!

Не надо иметь много колес, лучше иметь много спиц, удерживающих одно колесо. © Спасибо: 0 
Профиль
r9o-11





Пост N: 675
Зарегистрирован: 28.01.15
Откуда: Искитим, НСО
Рейтинг: 4
ссылка на сообщение  Отправлено: 03.12.19 19:25. Заголовок: Так а какие доказате..


Так а какие доказательства правильности измерения требуются? И что значит эта правильность?
Вот, подсоединил и посмотрел акустику на шириках - какие значения будем считать правильными, те, что на 1 кГц?:

Здесь показаны сопротивления напрямую включенных динамиков 4А28, через LRC цепь (для коррекции на СЧ) и с подключенными RC цепями (для коррекции сопротивления на ВЧ).

Андрей Спасибо: 0 
Профиль
RedStar
постоянный участник




Пост N: 2794
Зарегистрирован: 02.08.16
Откуда: Смоленск
Рейтинг: 1
ссылка на сообщение  Отправлено: 03.12.19 19:54. Заголовок: r9o-11 пишет: какие..


r9o-11 пишет:

 цитата:
какие доказательства правильности измерения требуются?...


Почему все читают вскользь? Выше постами написано же.
У Фурмана сказано, что он добивался одного напряжения в 2 Вольта при разных подключениях:


 цитата:
Раскачка была разной для пентода и триода, но выходное напряжение было 2 вольта, в обоих случаях.


То есть, у него, происходит намеренное изменение во входной части, что бы достичь желаемого на выходе?
Получается, что (U2-U1)=(2-2)=0!? Какие 48 Ом и 2,4 Ом?

П.С. Вот потому прошу от него ответа на перечисленное мной выше.
Либо он ответил с ошибкой в тексте.

Не надо иметь много колес, лучше иметь много спиц, удерживающих одно колесо. © Спасибо: 0 
Профиль
r9o-11





Пост N: 676
Зарегистрирован: 28.01.15
Откуда: Искитим, НСО
Рейтинг: 4
ссылка на сообщение  Отправлено: 03.12.19 20:21. Заголовок: :sm54: Почему Вы сч..


Почему Вы считаете, что все читают вскользь? Выше же написано, что выходное напряжение было 2 вольта в обоих случаях.
Какая разница, выставить на выходе 1,9 вольта или 2,1 вольта?

В данном измерении важна разница выходных напряжений при изменении сопротивления нагрузки.
И если выходное сопротивление изменилось, то о чём это говорит? Какие изменения произошли в лампе?

Андрей Спасибо: 0 
Профиль
aleks8845



Пост N: 660
Зарегистрирован: 07.03.16
Рейтинг: 1
ссылка на сообщение  Отправлено: 03.12.19 23:33. Заголовок: r9o-11 пишет: если ..


r9o-11 пишет:

 цитата:
если выходное сопротивление изменилось, то о чём это говорит?


Видимо RedStar понимает изменение R вых УМ, как изменение R самой нагрузки УМ (активное, реактивное) , отсюда и недоумение ,что такое R вых источника сигнала, усилителя....

Спасибо: 1 
Профиль
r9o-11





Пост N: 677
Зарегистрирован: 28.01.15
Откуда: Искитим, НСО
Рейтинг: 4
ссылка на сообщение  Отправлено: 03.12.19 23:38. Заголовок: :sm12: ...Так, навер..


...Так, наверное, не бывает... Слишком альтернативно...

Андрей Спасибо: 0 
Профиль
RedStar
постоянный участник




Пост N: 2795
Зарегистрирован: 02.08.16
Откуда: Смоленск
Рейтинг: 1
ссылка на сообщение  Отправлено: 04.12.19 01:19. Заголовок: r9o-11 пишет: В дан..


r9o-11 пишет:
 цитата:
В данном измерении важна разница выходных напряжений при изменении сопротивления нагрузки.

Только эти измерения должны сопровождаться неизменными входными!
Амплитудные значения тока и напряжения меняются... Все написал выше, зачем повторяться.

aleks8845 пишет:
 цитата:
изменение R вых УМ, как изменение R самой нагрузки УМ

Здесь не понял.
Я вот писал:
 цитата:
Получается, что (U2-U1)=(2-2)=0!? Какие 48 Ом и 2,4 Ом?

Видимо поторопился немного, извиняюсь конечно. Автор вопроса предполагал установленное выходное напряжение под Одной расчетной нагрузкой, которое изменяется под другой?
Хотя не важно. Смысла отвечать нет, даже подумать над этим прав-неправ, так как нет вводных данных об усилителе.

П.С. Простите, но метод двух нагрузок не учитывает большинства параметров. Только начинаю это понимать, читая Книги и форумы с обсуждениями темы.
Вопрос. С чего решено получать вых. сопр. таким методом. Ищу ответ на его решение и связанные с ними мотивации.

Не надо иметь много колес, лучше иметь много спиц, удерживающих одно колесо. © Спасибо: 0 
Профиль
r9o-11





Пост N: 678
Зарегистрирован: 28.01.15
Откуда: Искитим, НСО
Рейтинг: 4
ссылка на сообщение  Отправлено: 04.12.19 08:45. Заголовок: Может, потому что на..


Может, потому что на самом деле без разницы как определять?
Не нравится метод двух нагрузок, возьмите метод одной нагрузки:


Андрей Спасибо: 0 
Профиль
Сергеев Сергей
администратор




Пост N: 8254
Зарегистрирован: 21.05.07
Откуда: Иркутск
Рейтинг: 26
ссылка на сообщение  Отправлено: 04.12.19 09:43. Заголовок: Может я не прав, но ..


Может я не прав, но мне почему-то кажется что не все понимают что такое вых сопротивление усилителя.
https://habr.com/ru/company/soundpal/blog/378219/
https://www.lcard.ru/lexicon/res_outp
Отсюда и непонимание Ri.

Для измерения вых сопротивления (сравнения) важно равенство вых напряжения.
При этом у пентода и триода будет разное выходное.
Из-за разного усиления.

Новый сайт http://tubeaudio.ucoz.ru/
Тел. в Иркутске: рабочий (3952) 23-52-54. моб 661-681. 8-90-25-661-681.
WhatsApp Viber 89647335850
sergeev158(собака)mail.ru Заменить (собака) на @
С уважением, Сергеев Сергей Евгеньевич.
Спасибо: 0 
Профиль
Пермяк
moderator




Пост N: 6477
Зарегистрирован: 15.01.10
Откуда: Россия
Рейтинг: 25
ссылка на сообщение  Отправлено: 04.12.19 11:54. Заголовок: Формула №3: http://h..

Спасибо: 0 
Профиль
Плюмбум





Пост N: 290
Зарегистрирован: 14.12.16
Рейтинг: 3
ссылка на сообщение  Отправлено: 04.12.19 12:23. Заголовок: Леонид, такие таблич..


Леонид, такие картинки - не для наших средних умов

Спасибо: 0 
Профиль
volli





Пост N: 796
Зарегистрирован: 16.05.10
Откуда: Псков
Рейтинг: 2
ссылка на сообщение  Отправлено: 04.12.19 16:43. Заголовок: RedStar пишет: Здес..


RedStar пишет:
 цитата:
Здесь не понял.

Если бы только здесь!
 цитата:
Только эти измерения должны сопровождаться неизменными входными!

RedStar пишет:
 цитата:
Простите, но метод двух нагрузок не учитывает большинства параметров. Только начинаю это понимать,

Судя по этому:
RedStar пишет:
 цитата:
С чего решено получать вых. сопр. таким методом. Ищу ответ на его решение и связанные с ними мотивации.

- так ничего и не понял. Ну не укладывается это "старьё" в новую "теорию"!

Говори что думаешь. Думай, что говоришь! Спасибо: 0 
Профиль
RedStar
постоянный участник




Пост N: 2796
Зарегистрирован: 02.08.16
Откуда: Смоленск
Рейтинг: 1
ссылка на сообщение  Отправлено: 04.12.19 17:55. Заголовок: volli пишет: Если б..


volli пишет:
 цитата:
Если бы только здесь!

А вы поняли сказанное?

 цитата:
Ну не укладывается это "старьё" в новую "теорию"!

Кто бы говорил. Плановая экономика рулит во всей красе до сих пор, запирая подальше огромное количество нововведений. Потому и наша страна в числе отсталых напрочь.

Все здесь больше говорят теорией, чем делают практически. На форуме вообще перестали показывать результаты работ. Больше словоблудие ни о чем. Я не прав?

П.С. Выходное больше зависит от типа лампы. Пентод или триод. Там скорее другие обстоятельства, чем Ri. Вот до них и докопаюсь со временем.
Мне это ИНТЕРЕСНО! Просто, для себя. Вы же не хотите просто послушать?
Только не надо оскорблений. На себя, все, посмотрите. Хотя бы немного со стороны.
Много ошибаюсь в вопросах лампостроения, теоретически. Но практически многое доказывается обратное.
Если не верите, так неХ... переходить на личности, т.к. у многих отсутствует практика с пересчетом теории.
Что не так?

Не надо иметь много колес, лучше иметь много спиц, удерживающих одно колесо. © Спасибо: 0 
Профиль
Пермяк
moderator




Пост N: 6478
Зарегистрирован: 15.01.10
Откуда: Россия
Рейтинг: 25
ссылка на сообщение  Отправлено: 04.12.19 18:33. Заголовок: RedStar пишет: Прост..


RedStar пишет:
 цитата:
Простите, но метод двух нагрузок не учитывает большинства параметров.

Этот метод как раз учитывает ВСЕ параметры каскада, даже те, которые не входят в формулу. Потому, что он ПРАКТИЧЕСКИЙ, т.е. его-то Вы и должны бы уважать.:)
Метод двух нагрузок - не единственный.
подобный ему - метод холостой ход - нагрузка. Даёт менее точный результат, но в некоторых случаях вполне может применяться.
Есть и третий метод, при котором сигнал (переменный ток) подают на выход. Хорош тогда, когда ожидается очень малое значение выходного сопротивления, напр. - в транзисторных усилителях с их многочисленными глубокими ООС.
Ну, и наконец, есть метод х.х.-к.з. , который так напугал Вас недавно, но Вам забыли сказать, что применяется он преимущественно "мысленно", при выводе формул, хотя в редких случаях может применять и практически.

Спасибо: 0 
Профиль
RedStar
постоянный участник




Пост N: 2797
Зарегистрирован: 02.08.16
Откуда: Смоленск
Рейтинг: 1
ссылка на сообщение  Отправлено: 04.12.19 18:45. Заголовок: Пермяк пишет: Этот ..


Пермяк пишет:
 цитата:
Этот метод как раз учитывает ВСЕ параметры каскада, даже те, которые не входят в формулу.

Допустим. Траф намотан под нагрузку 8 Ом. Ему на выход нагружаем 6 или 4 Ом.
Соответственно меняются входные параметры. Верно? И смысл в методе двух нагрузок?
Так почему нельзя намотать траф с отводом, и при этом измерять под свою нагрузку?
Напряжение-сопротивление при 8 Ом с одним отводом и напряжение-сопротивление при 6-4 Ом со своими отводами.
Тогда формула должна немного измениться.

Не надо иметь много колес, лучше иметь много спиц, удерживающих одно колесо. © Спасибо: 0 
Профиль
Пермяк
moderator




Пост N: 6479
Зарегистрирован: 15.01.10
Откуда: Россия
Рейтинг: 25
ссылка на сообщение  Отправлено: 04.12.19 19:06. Заголовок: Добавлю к моему пред..


Добавлю к моему предыдущему посту:
https://www.hi-fi.ru/audioportal/topic/6114-vyhodnoe-soprotivlenie-lampovogo-usilitelya/page/5/

ЗЫ. Анатолий, убедительная просьба: не употребляйте при мне слово "траф.

Спасибо: 0 
Профиль
r9o-11





Пост N: 681
Зарегистрирован: 28.01.15
Откуда: Искитим, НСО
Рейтинг: 4
ссылка на сообщение  Отправлено: 04.12.19 19:13. Заголовок: RedStar, Вы же, наве..


RedStar, Вы же, наверное, знаете что такое пропорции и как рассчитывать резистивные делители из двух резисторов?
И, наверное, когда известны U1, U2 и R1, то не составит труда вычислить неизвестное Rx?
А в случае, если Rх находится внутри источника питания и Вы знаете только R1 и U2?
Разве не логично будет посчитать методом двух разных нагрузок?


Андрей Спасибо: 0 
Профиль
Пермяк
moderator




Пост N: 6480
Зарегистрирован: 15.01.10
Откуда: Россия
Рейтинг: 25
ссылка на сообщение  Отправлено: 04.12.19 19:27. Заголовок: RedStar пишет: смысл..


RedStar пишет:
 цитата:
смысл в методе двух нагрузок?

Что значит - "смысл"? Усилитель своим выходным сопром ДЕМПФИРУЕТ подвижную систему динамика, не позволяет ему резонировать на частоте собственного резонанса, сглаживает АЧХ на НЧ.
Популярные схемы без ООС могут иметь повышенное Rвых, чтобы убедиться в том, что Кдемпф достаточен, надо уметь это Rвых измерять по факту, особенно на НЧ.

Спасибо: 0 
Профиль
RedStar
постоянный участник




Пост N: 2798
Зарегистрирован: 02.08.16
Откуда: Смоленск
Рейтинг: 1
ссылка на сообщение  Отправлено: 04.12.19 19:56. Заголовок: r9o-11 пишет: Разве..


r9o-11 пишет:
 цитата:
Разве не логично будет посчитать методом двух разных нагрузок?

Не логично. Если R1 не соответствует нагрузке, значит можно определить наиболее подходящую нагрузку. Но никак не выходное сопротивление.
Лампа там не единственный элемент, верно?
Rх покажет не выходное, а изменение внутренних параметров нагружаемого источника, на котором происходит то падение напряжения, которое является комплексным.
Токи, напряжения, трансформатор, лампа... Комплекс должен соответствовать нагрузке.
Как можно привязывать целую систему под одну только лампу?
Остальные методы пока не рассматриваю, как наименее состоятельные.

Пермяк пишет:
 цитата:
Усилитель своим выходным сопром ДЕМПФИРУЕТ подвижную систему динамика,...

Да не об этом. Смысл в том, что рассчитанный под определенное сопротивление усилитель, может не удовлетворять требованиям других нагрузок.
Вот потому и спорю, что это не выход из положения определять методом двух нагрузок при одной рассчитанной.

Не надо иметь много колес, лучше иметь много спиц, удерживающих одно колесо. © Спасибо: 0 
Профиль
r9o-11





Пост N: 682
Зарегистрирован: 28.01.15
Откуда: Искитим, НСО
Рейтинг: 4
ссылка на сообщение  Отправлено: 04.12.19 20:23. Заголовок: RedStar пишет: Если..


RedStar пишет:
 цитата:
Если R1 не соответствует нагрузке, значит можно определить наиболее подходящую нагрузку. Но никак не выходное сопротивление.

Что означает словосочетание "можно определить". И что такое "подходящая нагрузка"? Для чего подходящая?

Андрей Спасибо: 0 
Профиль
Кузьмич



Пост N: 1483
Зарегистрирован: 21.11.15
Рейтинг: 8
ссылка на сообщение  Отправлено: 04.12.19 21:06. Заголовок: r9o-11 пишет: что т..


r9o-11 пишет:
 цитата:
что такое "подходящая нагрузка"? Для чего подходящая?

Понимаю так: Когда делается бездумно, не проводя каких-либо расчётов, в том числе и ожидаемого выходного сопротивления, как говорится, я его слепила из того, что было... Вот тогда и встаёт вопрос, для чего подходит.

А вообще, в который раз говорю, что этот тупица похоже ещё и умышленно устраивает этот базар. Он ему видимо нравится.

Спасибо: 0 
Профиль
Плюмбум





Пост N: 291
Зарегистрирован: 14.12.16
Рейтинг: 3
ссылка на сообщение  Отправлено: 04.12.19 21:31. Заголовок: Да он вообще задаёт ..


Да он вообще задаёт вопросы только для того, чтобы тут же, не задумываясь, опровергнуть ответы.
Люди для него стараются, пытаются ответить, ссылки дают, сканы выкладывают... а он даже не утруждается смысл написанного понять.

Спасибо: 0 
Профиль
RedStar
постоянный участник




Пост N: 2799
Зарегистрирован: 02.08.16
Откуда: Смоленск
Рейтинг: 1
ссылка на сообщение  Отправлено: 04.12.19 21:33. Заголовок: Кузьмич пишет: этот..


Кузьмич пишет:
 цитата:
этот тупица похоже ещё и умышленно устраивает этот базар.

Тупица здесь вы. Не раз говорил, на личности не переходить.

 цитата:
Когда делается бездумно,

Бездумно делаете вы. Хотя... вы ничего не делаете.

Не надо иметь много колес, лучше иметь много спиц, удерживающих одно колесо. © Спасибо: 0 
Профиль
r9o-11





Пост N: 684
Зарегистрирован: 28.01.15
Откуда: Искитим, НСО
Рейтинг: 4
ссылка на сообщение  Отправлено: 04.12.19 21:49. Заголовок: Ну так что там с воп..


Ну так что там с вопросами?
Вы хоть скажите "можно определить" - это подразумевается "посчитать по формуле" или это в смысле "задать нужное"? Просто, в одном предложении вместе с "Если R1 не соответствует нагрузке...", вообще не понятно о чём идёт речь. Что за условие возникло "если..."?

Андрей Спасибо: 0 
Профиль
RedStar
постоянный участник




Пост N: 2800
Зарегистрирован: 02.08.16
Откуда: Смоленск
Рейтинг: 1
ссылка на сообщение  Отправлено: 04.12.19 22:19. Заголовок: Плюмбум пишет: а он..


Плюмбум пишет:

 цитата:
а он даже не утруждается смысл написанного понять.


Не надо переворачивать... Сами попробуйте провести эксперимент с двумя нагрузками, но при отдельных отводах...
...кому говорю, все равно впустую.

r9o-11 пишет:
 цитата:
Ну так что там с вопросами?

Как бы сказать? Подумаю над формулировкой предложений.

П.С. Скажите, разве не прав, что:
 цитата:
Вот потому и спорю, что это не выход из положения определять методом двух нагрузок при одной рассчитанной.



Не надо иметь много колес, лучше иметь много спиц, удерживающих одно колесо. © Спасибо: 0 
Профиль
r9o-11





Пост N: 685
Зарегистрирован: 28.01.15
Откуда: Искитим, НСО
Рейтинг: 4
ссылка на сообщение  Отправлено: 04.12.19 22:36. Заголовок: Так, может, ну её на..


Так, может, ну её нафиг... Чего там думать-то? Давно бы уже взяли усь с набором резисторов через два ома и проверили расчётное Rвыходное... А то, может, мы тут сообщений настрочили больше, чем там проценты точности...
А, да, ещё и резисторы, наверное, надо проверить на соответствие маркировке. А то мало ли чего на них написано...

Андрей Спасибо: 0 
Профиль
Кузьмич



Пост N: 1485
Зарегистрирован: 21.11.15
Рейтинг: 8
ссылка на сообщение  Отправлено: 04.12.19 22:38. Заголовок: RedStar пишет: попр..


RedStar пишет:
 цитата:
попробуйте провести эксперимент с двумя нагрузками, но при отдельных отводах...
...кому говорю, все равно впустую.

Так тупице и не понятно, что от отводов меняется коэффициент трансформации. Меняется и реультат.
Это коню понятно, но не тупице.

Спасибо: 0 
Профиль
RedStar
постоянный участник




Пост N: 2801
Зарегистрирован: 02.08.16
Откуда: Смоленск
Рейтинг: 1
ссылка на сообщение  Отправлено: 04.12.19 23:32. Заголовок: Кузьмич, да уймись, ..


Кузьмич пишет:
 цитата:
...от отводов меняется коэффициент трансформации.

А от разной нагрузки, что меняется? Ra. И...:
Тоже -
 цитата:
Меняется и реультат.


r9o-11 пишет:
 цитата:
Так, может, ну её нафиг...

Не. Буду доводить один проект, попробую отводы сделать и проверить.

Не надо иметь много колес, лучше иметь много спиц, удерживающих одно колесо. © Спасибо: 0 
Профиль
Кузьмич



Пост N: 1486
Зарегистрирован: 21.11.15
Рейтинг: 8
ссылка на сообщение  Отправлено: 04.12.19 23:49. Заголовок: RedStar пишет: По т..


RedStar пишет:
 цитата:
А от разной нагрузки, что меняется?

Меняется напряжение на нагрузке.
С помощью которого и вычисляется Вых. Сопр.
Ra безусловно меняется. Но в Ra входят ещё компоненты, которые лишние при вычислении Вых. Сопр.
А нужны только Ктр.

Спасибо: 0 
Профиль
RedStar
постоянный участник




Пост N: 2803
Зарегистрирован: 02.08.16
Откуда: Смоленск
Рейтинг: 1
ссылка на сообщение  Отправлено: 04.12.19 23:53. Заголовок: Кузьмич пишет: кото..


Кузьмич пишет:
 цитата:
которые лишние при вычислении Вых. Сопр.


Говорите лишние? Выходное напряжение не зависит от тока первички?

Не надо иметь много колес, лучше иметь много спиц, удерживающих одно колесо. © Спасибо: 0 
Профиль
Кузьмич



Пост N: 1487
Зарегистрирован: 21.11.15
Рейтинг: 8
ссылка на сообщение  Отправлено: 05.12.19 00:22. Заголовок: RedStar пишет: Гово..


RedStar пишет:
 цитата:
Говорите лишние?

Говорю.
И в формуле Ra нет никаких токов, ни напряжений.



Не надо усложнять себе и другим жизнь поиском блох, там где их нет.
15-страница исписана... ни о чём. Как об стенку горох.

Спасибо: 0 
Профиль
RedStar
постоянный участник




Пост N: 2804
Зарегистрирован: 02.08.16
Откуда: Смоленск
Рейтинг: 1
ссылка на сообщение  Отправлено: 05.12.19 00:34. Заголовок: омельян, ток первичк..


Кузьмич пишет:
 цитата:
И в формуле Ra нет никаких токов, ни напряжений.

Как это не зная Ктр вычислить Ra? Если только транс-тор уже готовый...

Не надо иметь много колес, лучше иметь много спиц, удерживающих одно колесо. © Спасибо: 0 
Профиль
Кузьмич



Пост N: 1488
Зарегистрирован: 21.11.15
Рейтинг: 8
ссылка на сообщение  Отправлено: 05.12.19 01:01. Заголовок: RedStar пишет: Как ..


RedStar пишет:
 цитата:
Как это не зная Ктр вычислить Ra?

Формулу-то смотри!!!
Или ты вообще ничего не хочешь знать, видеть и замечать???

Спасибо: 0 
Профиль
RedStar
постоянный участник




Пост N: 2807
Зарегистрирован: 02.08.16
Откуда: Смоленск
Рейтинг: 1
ссылка на сообщение  Отправлено: 05.12.19 01:39. Заголовок: Ктр=√(Ra/Rn) R..


Кузьмич пишет:
 цитата:
Формулу-то смотри!!!

Из этой формулы:
Ктр=√(Ra/Rn)
А как же Ra=U/I ? Там нет напряжения и тока?
Вот картинка в помощь http://hiend.borda.ru/?1-2-1575480778632-00000583-000-0-0#000.001.001.001

Не надо иметь много колес, лучше иметь много спиц, удерживающих одно колесо. © Спасибо: 0 
Профиль
Кузьмич



Пост N: 1491
Зарегистрирован: 21.11.15
Рейтинг: 8
ссылка на сообщение  Отправлено: 05.12.19 04:50. Заголовок: RedStar Не сваливай..


RedStar, не сваливай всё в одну кучу. Не плоди сущностей сверх необходимых.
 цитата:
Тогда говорить не о чем.

Похоже, что уже очень многие ждут, когда ты наконец замолчишь.

Спасибо: 0 
Профиль
volli





Пост N: 798
Зарегистрирован: 16.05.10
Откуда: Псков
Рейтинг: 2
ссылка на сообщение  Отправлено: 05.12.19 04:59. Заголовок: RedStar пишет: Если..


RedStar пишет:
 цитата:
Если R1 не соответствует нагрузке, значит можно определить наиболее подходящую нагрузку.

Как? И главное - зачем?
 цитата:
Rх покажет не выходное, а изменение внутренних параметров нагружаемого источника, на котором происходит то падение напряжения, которое является комплексным.

Почему? докажите!
 цитата:
Комплекс должен соответствовать нагрузке.

"С ног на голову?" А может наоборот - сначала выбрать нагрузку, а потом рассчитать соответствующий "комплекс"? А то ведь может получиться так, что к Вашему "комплексу" не окажется подходящей нагрузки?
 цитата:
Может считаете так: 2-5Ri=Ra

Ктр=√(Ra/Rn)
Ra=U/I
Что Вы имеете в виду под Ra? Не тут ли "собака" зарыта? Подсказка: Что такое "приведённое" сопротивление? и какое отношение оно имеет к Ri?

Говори что думаешь. Думай, что говоришь! Спасибо: 0 
Профиль
Кузьмич



Пост N: 1493
Зарегистрирован: 21.11.15
Рейтинг: 8
ссылка на сообщение  Отправлено: 05.12.19 05:10. Заголовок: volli пишет: к Ваше..


volli пишет:
 цитата:
к Вашему "комплексу" не окажется подходящей нагрузки?

Вот у него как раз и оказывается. А потом при этом заявляет, что теория - фуфло.

Спасибо: 0 
Профиль
volli





Пост N: 800
Зарегистрирован: 16.05.10
Откуда: Псков
Рейтинг: 2
ссылка на сообщение  Отправлено: 05.12.19 05:54. Заголовок: Короче, случай "..


Короче, случай "клинический"!
Плохо владея теорией, методикой расчёта, и техникой измерений, не соответствие расчёта и полученных результатов "измерений" своего "комплекса" решил упразднить "устаревшую классическую теорию", и создать свою новую "теорию", подогнав её под свои "результаты измерений"... . Причём чужими "мозгами"!

Говори что думаешь. Думай, что говоришь! Спасибо: 0 
Профиль
RedStar
постоянный участник




Пост N: 2808
Зарегистрирован: 02.08.16
Откуда: Смоленск
Рейтинг: 1
ссылка на сообщение  Отправлено: 05.12.19 14:17. Заголовок: volli пишет: Как? И..


volli пишет:
 цитата:
Как? И главное - зачем?

Затем, что большинство, мотая тран-торы, не всегда "попадают" в расчетную нагрузку.
Иногда и я не исключение. Приходится перематывать. Но мне это не сложно.
Меняя нагрузку можно определить наиболее подходящую, и потом, отматывая-доматывая вторичку выбрать оптимальный вариант.
 цитата:
Почему? докажите!

Попробуйте со вторички снять бОльший ток, если первичка не сможет ее "передать". Будет значительное понижение напряжения. Известную формулу описал выше.
 цитата:
"С ног на голову?"

Вообще-то все правильно. Комплекс должен соответствовать нагрузке, а нагрузка комплексу.
Да и не такой уж большой выбор нагрузок. 4, 6, 8 и редко выше.
В последнем вопросе, - все в куче. Поконкретнее.
 цитата:
Короче, случай

Теория, методика расчета. Да все мне известно. И так, и так - экспериментирую.

Упразднять? Это вы решили за меня. Я не предлагаю подобного, а лишь пробую дополнять и показываю Практические результаты!
Создаю? Вполне. Подгонять под результаты? Как бы - да. Они точнее. А техника измерений едина, смотря как считать потом.

Кузьмич пишет:
 цитата:
теория - фуфло.

За всю ни слова не сказал. Опять передергиваете.

Не надо иметь много колес, лучше иметь много спиц, удерживающих одно колесо. © Спасибо: 0 
Профиль
Кузьмич



Пост N: 1498
Зарегистрирован: 21.11.15
Рейтинг: 8
ссылка на сообщение  Отправлено: 05.12.19 15:27. Заголовок: RedStar пишет: что ..


RedStar пишет:
 цитата:
что большинство, мотая тран-торы, не всегда "попадают" в расчетную нагрузку.

Не надо про всех обобщать. Расчёт и производится для того, чтобы "попасть" в расчётную нагрузку.
А если ты не умеешь рассчитывать, это твоё горе.
 цитата:
Меняя нагрузку можно определить наиболее подходящую, и потом, отматывая-доматывая вторичку выбрать оптимальный вариант.

Это из серии: Дурная голова рукам и ногам покоя не даёт.
 цитата:
если первичка не сможет ее "передать"

Чушь!
 цитата:
За всю ни слова не сказал

Сказал. И много.

Спасибо: 0 
Профиль
volli





Пост N: 803
Зарегистрирован: 16.05.10
Откуда: Псков
Рейтинг: 2
ссылка на сообщение  Отправлено: 05.12.19 15:48. Заголовок: RedStar пишет: Попр..


RedStar пишет:
 цитата:
Попробуйте со вторички снять бОльший ток, если первичка не сможет ее "передать".

Объясните пожалуйста, как первичка передаёт "ток" вторичке, если они гальванически не связаны?
Ещё раз:
 цитата:
Из этой формулы:
Ктр=√(Ra/Rn)
А как же Ra=U/I ? Там нет напряжения и тока?

Ra - что это такое, откуда Вы его взяли (получили)?
Подсказки: (по системе ЕГ - выбери правильный ответ )
1. сопротивление по постоянному току,
2. сопротивление по переменному току.
3. приведенное сопротивление.

RedStar пишет:
 цитата:
А техника измерений едина,

Но Вы ею не владеете в должной степени, так как пишете:
 цитата:
смотря как считать потом.

при "подсчёте" постоянно "мешаете в кучу" (заменяя ( подгоняя) результат под нужный Вам ответ путём подмены действующего напряжения амплитудным, сопротивление по пстоянному току с сопротивлением по переменному и т.д. в том же духе
 цитата:
что большинство, мотая тран-торы, не всегда "попадают" в расчетную нагрузку.

С какой точностью ( в процентах)? Объясните, для чего такая точность с точки зрения "комплексной нагрузки" и частотной зависимости.
 цитата:
Теория, методика расчета. Да все мне известно.

Есть большие сомнения.

Говори что думаешь. Думай, что говоришь! Спасибо: 0 
Профиль
volli





Пост N: 804
Зарегистрирован: 16.05.10
Откуда: Псков
Рейтинг: 2
ссылка на сообщение  Отправлено: 05.12.19 15:48. Заголовок: RedStar пишет: Попр..


RedStar пишет:
 цитата:
Попробуйте со вторички снять бОльший ток, если первичка не сможет ее "передать".

Объясните пожалуйста, как первичка передаёт "ток" вторичке, если они гальванически не связаны?
Ещё раз:
RedStar пишет:
 цитата:
Из этой формулы:
Ктр=√(Ra/Rn)
А как же Ra=U/I ? Там нет напряжения и тока?

Ra - что это такое, откуда Вы его взяли (получили)?
Подсказки: (по системе ЕГ - выбери правильный ответ )
1. сопротивление по постоянному току,
2. сопротивление по переменному току.
3. приведенное сопротивление.
 цитата:
А техника измерений едина,

Но Вы ею не владеете в должной степени, так как
RedStar пишет:
 цитата:
смотря как считать потом.

при "подсчёте" постоянно "мешаете в кучу" (заменяя ( подгоняя) результат под нужный Вам ответ путём подмены действующего напряжения амплитудным, сопротивление по пстоанному току с сопротивлением по переменному и т.д. в том же духе
 цитата:
что большинство, мотая тран-торы, не всегда "попадают" в расчетную нагрузку.

С какой точностью ( в процентах)? Объясните, для чего такая точность с точки зрения "комплексной нагрузки" и частотной зависимости.
 цитата:
Теория, методика расчета. Да все мне известно.

Есть большие сомнения.

Говори что думаешь. Думай, что говоришь! Спасибо: 0 
Профиль
r9o-11





Пост N: 688
Зарегистрирован: 28.01.15
Откуда: Искитим, НСО
Рейтинг: 4
ссылка на сообщение  Отправлено: 05.12.19 16:12. Заголовок: RedStar, а Вы читали..


RedStar, а Вы читали опусы Владимира Пронина про его "путь в болоте флейма"?

Андрей Спасибо: 0 
Профиль
RedStar
постоянный участник




Пост N: 2809
Зарегистрирован: 02.08.16
Откуда: Смоленск
Рейтинг: 1
ссылка на сообщение  Отправлено: 05.12.19 16:47. Заголовок: Кузьмич, какой же вы..


Кузьмич, какой же вы упертый.
 цитата:
Не надо про всех обобщать...

Так докажите делом, приведите пример. Предварительно нагрузив на 4, 6, 8 Ом и составьте таблицу. По ней видно будет, попали в яблочко или в небо. Только желательно пару-тройку тран-ров, а то выберете самый-самый.
 цитата:
Чушь!

Ну, если так считаете, значит не знаете:

https://ru.wikipedia.org/wiki/%D0%9A%D0%BE%D1%8D%D1%84%D1%84%D0%B8%D1%86%D0%B8%D0%B5%D0%BD%D1%82_%D1%82%D1%80%D0%B0%D0%BD%D1%81%D1%84%D0%BE%D1%80%D0%BC%D0%B0%D1%86%D0%B8%D0%B8


Вы уверены, что под определенную нагрузку не стоит учитывать токи?

volli пишет:
 цитата:
Объясните пожалуйста, как первичка передаёт "ток" вторичке, если они гальванически не связаны?

И для вас ссылка выше.
Ответ.
 цитата:
3. приведенное сопротивление.

Ничего не мешаю в кучу. Считаю только амплитудными.
А с точностью все прекрасно получается.

r9o-11 пишет:
 цитата:
"путь в болоте флейма"

Скиньте, почитаю.

П.С. Уже не надо, нашел.
Цитата: "Пройти же в топях, не зная троп, не сможет никто."
Так вот, не знал бы основ, не лез бы напролом.

Не надо иметь много колес, лучше иметь много спиц, удерживающих одно колесо. © Спасибо: 0 
Профиль
Кузьмич



Пост N: 1499
Зарегистрирован: 21.11.15
Рейтинг: 8
ссылка на сообщение  Отправлено: 05.12.19 17:22. Заголовок: RedStar пишет: Ну, ..


RedStar пишет:
 цитата:
Ну, если так считаете, значит не знаете:

Всё больше и больше удивляюсь, какой ты тупой и ещё глупый...
Так вот попытаюсь донести в очередной раз. Если твоя первичка не способна что-то передать...
то КПД твоего транса ниже плинтуса. Т.е. этот ТВЗ по КПД не рассчитывался. И актсопры этого очень высокие.
Это результат твоего стремления сэкономить на нём и сделать его как можно меньше.
Выручит только досканальное изучение классиков Цыкина, Войшвилло. И неукоснительное соблюдение их требований.
 цитата:
Так докажите делом, приведите пример

Может тебе ещё и ... вареньем намазать???
В теме "ТВЗ для 6С4С" привёл пример ТВЗ. Не до конца посчитал, но это мелочи. Мотай, будет отличный ТВЗ!
Сомневаешься? Это от незнания.

Спасибо: 0 
Профиль
RedStar
постоянный участник




Пост N: 2810
Зарегистрирован: 02.08.16
Откуда: Смоленск
Рейтинг: 1
ссылка на сообщение  Отправлено: 05.12.19 17:35. Заголовок: Кузьмич пишет: И не..


Кузьмич пишет:
 цитата:
И неукоснительное соблюдение их требований.

Так соблюдайте себе на здоровье, а ко мне не лезьте. Все равно не понимаете о чем я.
 цитата:
Мотай, будет отличный ТВЗ!

Нет уж, спасибо. Получить максимум 3,3 Вт?
Я бы на 4,5 сделал.

Не надо иметь много колес, лучше иметь много спиц, удерживающих одно колесо. © Спасибо: 0 
Профиль
Кузьмич



Пост N: 1500
Зарегистрирован: 21.11.15
Рейтинг: 8
ссылка на сообщение  Отправлено: 05.12.19 17:46. Заголовок: RedStar пишет: Так ..


RedStar пишет:
 цитата:
Так соблюдайте себе на здоровье, а ко мне не лезьте.

Нефиг тогда на техническом форуме свою бредятину нести!
 цитата:
Нет уж, спасибо. Получить максимум 3,3 Вт?
Я бы на 4,5 сделал.

Ещё раз тупишь. Даже прочитать нормально не можешь. В теме все выкладки.
5,42 ватта. При КПД 95% всё больше 5 Ватт на выходе. Только чуток вторичку "подправить".
Видишь какой ты тупой??? Прочитать даже не можешь.

Спасибо: 1 
Профиль
RedStar
постоянный участник




Пост N: 2811
Зарегистрирован: 02.08.16
Откуда: Смоленск
Рейтинг: 1
ссылка на сообщение  Отправлено: 05.12.19 18:36. Заголовок: Кузьмич, остынь. Вед..


Кузьмич, лучше бы там расписал подробнее, как получил 5 Вт. Какие напряжения....
Скажешь, что нет необходимости, опять откажешься?

r9o-11 пишет:
 цитата:
не про те топи и тропы Вы прочитали.

"А ещё надо уметь это всё воплотить".
Чего не видно от окружающих.

Не надо иметь много колес, лучше иметь много спиц, удерживающих одно колесо. © Спасибо: 0 
Профиль
Кузьмич



Пост N: 1501
Зарегистрирован: 21.11.15
Рейтинг: 8
ссылка на сообщение  Отправлено: 05.12.19 18:44. Заголовок: RedStar пишет: Лучш..


RedStar пишет:
 цитата:
Лучше бы там расписал подробнее, как получил 5 Вт

Ну ты и пенёк, чессслово... Удивлению нет предела. Там всё расписано и указано.




RedStar пишет:
 цитата:
Скажешь, что нет необходимости, опять откажешься?

Тебе очень много здесь было сказано. И ещё сказали бы, если бы ты хоть немного слушал и вникал. И не придумывал свою ахинею, типа этой
RedStar пишет:
 цитата:
а ко мне не лезьте.


 цитата:
Чего не видно от окружающих.

А тебе надо чтобы к тебе на квартиру привезли, включили и заставили слушать???
А ты всё обсерешь, и скажешь, что это всё фигня, вот у тебя 6Ж2П это вещь. Да?

Спасибо: 0 
Профиль
r9o-11





Пост N: 691
Зарегистрирован: 28.01.15
Откуда: Искитим, НСО
Рейтинг: 4
ссылка на сообщение  Отправлено: 05.12.19 18:47. Заголовок: Так вопрос в том, чт..


RedStar, так вопрос в том, что Вы собрались воплощать... Надо ли оно кому-то ещё кроме Вас? И это "надо" - оно точно "надо", а то может, это временная прихоть? А надо совсем другое?..
Вот примерно об этом можно почитать у Влада Пронина...

Андрей Спасибо: 0 
Профиль
RedStar
постоянный участник




Пост N: 2812
Зарегистрирован: 02.08.16
Откуда: Смоленск
Рейтинг: 1
ссылка на сообщение  Отправлено: 05.12.19 19:10. Заголовок: Кузьмич, вот привели..


Кузьмич, вот привели найденную по формуле мощность. А в реальности, при Ктр 29?

Не нужно ко мне привозить. Покажите здесь ваши измерения на действующей модели. Тогда разговор примет другие обороты.
Я же привожу измерения.

Не надо иметь много колес, лучше иметь много спиц, удерживающих одно колесо. © Спасибо: 0 
Профиль
volli





Пост N: 805
Зарегистрирован: 16.05.10
Откуда: Псков
Рейтинг: 2
ссылка на сообщение  Отправлено: 05.12.19 19:24. Заголовок: RedStar пишет: Вы у..


RedStar пишет:
 цитата:
Вы уверены, что под определенную нагрузку не стоит учитывать токи?

Уверен. Потому как нагрузка у Вас НЕопределённая, она "гуляет" довольно в больших интервалах по разным причинам.
Ваша ссылка не совсем корректна - силовые трансформаторы довольно сильно отличаются от трансформаторов "звукового" диапазона, и "тупо" применять методику расчёта силового к "звуковому", как Вы делаете, по крайней мере - невежество.
 цитата:
Попробуйте со вторички снять бОльший ток, если первичка не сможет ее "передать". Будет значительное понижение напряжения. Известную формулу описал выше.

А как насчёт соответствия ЭДС?
Сама формула коэф. трансвормации через U или I имеет погрешность (Если пренебречь потерями в обмотках), достаточно приличную, а Вы пытаетесь "ловить блох" в расчёте трансформатора.
RedStar пишет:
 цитата:
Ответ.
цитата:
3. приведенное сопротивление.

Поясните пожалуйста: Какого и к чему? С каким коэффициентом?
пишет:
Из этой формулы:
1. Ктр=√(Ra/Rn)
А как же
2.Ra=U/I ? Там нет напряжения и тока?
А так же:
Какое Ra в первой формуле, а какое во второй? Из трёх ответов по ЕГ.
Можно ли их отождествлять?

Говори что думаешь. Думай, что говоришь! Спасибо: 0 
Профиль
Кузьмич



Пост N: 1502
Зарегистрирован: 21.11.15
Рейтинг: 8
ссылка на сообщение  Отправлено: 05.12.19 20:26. Заголовок: RedStar пишет: вот ..


RedStar пишет:
 цитата:
вот привели найденную по формуле мощность. А в реальности, при Ктр 29?

Ты вообще читать умеешь? А запоминать что прочитал?
И сортировать, что к чему???
Ктр = 29 для Ra=7 кОм.
Мощность на скрине из учебника 5,42 Ватта при Ra = 6,24 кОм. Вторичку для этого случая надо подкорректировать, т.е.
количество витков вторички несколько увеличить (до 99 витков). А проводок выбрать потоньше, что бы уместить на каркасе.
При КПД ТВЗ 95% реально на выходе будет мах. мощность 5,1 ватта.
Сомневаешься??? Значит опровергаешь теорию.
А твоя практика, если она не соответствует теории - туфта.
RedStar пишет:

 цитата:
Тогда разговор примет другие обороты.


С кем, с тобой?
До тех пор, пока ты не изучишь теорию...
До тех пор, пока у тебя практика не будет сходится с теорией...
До тех пор, пока ты не научишься выражаться технически грамотно и без дебильного диалекта...
До тех пор, пока ты не перестанешь выкладывать сюда всякую ересь....
Разговора не жди. А вот бить тебя будут. Я не исключение.

Спасибо: 0 
Профиль
RedStar
постоянный участник




Пост N: 2813
Зарегистрирован: 02.08.16
Откуда: Смоленск
Рейтинг: 1
ссылка на сообщение  Отправлено: 05.12.19 20:28. Заголовок: volli пишет: силовы..


volli пишет:
 цитата:
силовые трансформаторы довольно сильно отличаются от трансформаторов "звукового" диапазона,

Отличаются, верно подметили. Только никто не отменял Закон Фарадея и уравнения идеального трансформатора.
Считаете это невежеством? Остальное тоже верно, но погрешности считаемы. Но кто их толком рассматривает?
Остальное оставлю без ответа, сами знаете, что да как.

Не надо иметь много колес, лучше иметь много спиц, удерживающих одно колесо. © Спасибо: 0 
Профиль
Кузьмич



Пост N: 1504
Зарегистрирован: 21.11.15
Рейтинг: 8
ссылка на сообщение  Отправлено: 05.12.19 20:36. Заголовок: RedStar пишет: Толь..


RedStar пишет:
 цитата:
Только никто не отменял Закон Фарадея

Это что??? Попытка сделать умное лицо при отвратительной игре???
Ты простое, азы освой, изучи! Фарадей, блин!

Спасибо: 0 
Профиль
RedStar
постоянный участник




Пост N: 2814
Зарегистрирован: 02.08.16
Откуда: Смоленск
Рейтинг: 1
ссылка на сообщение  Отправлено: 05.12.19 20:50. Заголовок: Кузьмич пишет: сдел..


Кузьмич пишет:
 цитата:
сделать умное лицо при отвратительной игре

Играешь ты, словесно, без доказательств.
 цитата:
При КПД ТВЗ 95% реально на выходе будет мах. мощность 5,1 ватта.

Будет ли? Собирайте, проверим. По расчету выходит красиво, а на деле?

Не надо иметь много колес, лучше иметь много спиц, удерживающих одно колесо. © Спасибо: 0 
Профиль
Кузьмич



Пост N: 1505
Зарегистрирован: 21.11.15
Рейтинг: 8
ссылка на сообщение  Отправлено: 05.12.19 21:34. Заголовок: RedStar пишет: без ..


RedStar пишет:
 цитата:
без доказательств.

Я тебе учебник привёл. Это основа! Самое главное доказательство!
 цитата:
По расчету выходит красиво, а на деле?

На деле только твои кривые руки и бестолковая голова могут всё изменить.

Это ещё раз подтверждает, что ты отвергаешь теорию.
Значит - не иначе, как тупица. Обратное можешь доказать?

Спасибо: 0 
Профиль
RedStar
постоянный участник




Пост N: 2815
Зарегистрирован: 02.08.16
Откуда: Смоленск
Рейтинг: 1
ссылка на сообщение  Отправлено: 05.12.19 22:41. Заголовок: Кузьмич пишет: ...у..


Кузьмич пишет:
 цитата:
...учебник привёл. Это основа!

Хорошо, допустим ты произвел расчет выходного тран-ра по теории.
Есть уверенность в том, что твой расчет совпадет с реальным изделием?
Ты привел формулы, в: http://hiend.borda.ru/?1-22-1575554626759-00000544-000-280-0#131.001.001.001.001.001.001
Практических доказательств правильности не приводишь. Это минус.
По теории, как бы, плюс. Зачетно! Сам грешу таким, во Имя Справедливости!
Только объясни, каким образом, по формуле 8.23, выходит мой Гуттаперчевый на мощность от 20 Вт? Хотя там только 10 Вт?
Промолчим, или будем ссылаться на оскорбления, теорию, и отмазки
 цитата:
твои кривые руки и бестолковая голова могут всё изменить.

Кривые руки у того, кто не задумывается над процессами внутри. Дальше не буду говорить. Сам поймешь.
Бестолковая голова у тех, кто беЗтолково твердит заученное. Не пытаясь думать своими проветренными мозгами.
"Бес - приставка. В Русском языке означает "беса" (с маленькой буквы). А Слово Без - Отрицание".
Раньше так и было, пока не внес Луначарский, и ему подобные товарищи, эту хрень.

 цитата:
Обратное можешь доказать?

Что мне доказывать? Вам мало может практики, в которых не понимаете, почему очень многое идет вразрез?
В который раз сообщаю, я НЕ доказываю! Только Показываю на возможность изменения некоторых "ваших" постулатов.
Не понятно в очередной раз?

Форумчане! Извините меня.
За настойчивость и корявый ломанный язык в терминологии. За предъявление своего мировоззрения. За пробу внедрения некоих формулировок, отличных от теории.
Ошибаюсь часто. Но эти ошибки явно показывают отличия. Кто не верит или упертый, - ваше право. Ничего против не имею. Будьте благоразумны в выражениях. Спасибо.
Мне все Интересно, хоть и устарело это для нашего времени. Учителя срываются. Только форум не средство обучения. И это очень жаль.

Не надо иметь много колес, лучше иметь много спиц, удерживающих одно колесо. © Спасибо: 0 
Профиль
r9o-11





Пост N: 693
Зарегистрирован: 28.01.15
Откуда: Искитим, НСО
Рейтинг: 4
ссылка на сообщение  Отправлено: 05.12.19 22:49. Заголовок: А, может, некоторая ..


А, может, некоторая проблема в том, что у RedStar-а осциллограф неправильно THD показывает и он думает, что «ух ты, как у меня всё здорово с точными формулами и с катодными обмотками получается, а эти «лопухи» собирают усилители с процентными искажениями и до сих пор не слышали настоящего звука»?

Попробуем перепроверить «через вольты» - переведём значения dBV в условные напряжения и посчитаем их процентное отношение. Тогда пусть напряжение основного тона будет 1 В. Уровень его 2-й гармоники будет 1/63=0,01587 В, что опять же составляет 1,587% от 100% сигнала в 1 В.

А что же тогда за значение 0,2819% показывает осциллограф?
Смотрим синюю строку и видим, что THD там указан ещё раз, но уже в децибелах -50,9984 dB. Что это такое, к чему привязано? А если посчитать этот уровень относительно 1 В, то что получится? Округляем 50,9984 до 51 (можно же? ) и посчитав, получаем 0,28% - то самое последнее показание в строке - THD 0,2819%.

Теперь вопрос – а почему осциллограф проводит расчёт THD от нулевой отметки шкалы децибел? Может, у него установки такие – привязка расчёта не к уровню основного сигнала, а к отметке «0» шкалы dBV?
Ну, и ещё один вопрос - а что это за значение уровня -50,9984 dB, к чему оно относится? Где "это" на спектре? Куда смотреть?
А вот это фиг его знает - наверное, надо читать инструкцию, но как-то лень…

Андрей Спасибо: 0 
Профиль
RedStar
постоянный участник




Пост N: 2816
Зарегистрирован: 02.08.16
Откуда: Смоленск
Рейтинг: 1
ссылка на сообщение  Отправлено: 05.12.19 23:00. Заголовок: r9o-11 пишет: осцил..


r9o-11 пишет:
 цитата:
осциллограф неправильно THD показывает?

А вы спросите у других людей, которые пользуются таким же прибором. Что они вам скажут?
 цитата:
почему осциллограф проводит расчёт THD от нулевой отметки шкалы децибел?

От чего же еще? Есть тема про этот осцилл., там почитайте и присоединяйтесь для обсуждения
 цитата:
надо читать инструкцию, но как-то лень…

Вот и в инструкции сказано, что все измерения производятся на основе Общепринятых. Даже сказано, что он соответствует стандартам.
Приобретите его, он не дорогой. Сами проверите.
Я буду только рад, если вы измените это положение.
У меня, и других, нет оснований ему не доверять.

Не надо иметь много колес, лучше иметь много спиц, удерживающих одно колесо. © Спасибо: 0 
Профиль
volli





Пост N: 806
Зарегистрирован: 16.05.10
Откуда: Псков
Рейтинг: 2
ссылка на сообщение  Отправлено: 05.12.19 23:00. Заголовок: R-S пишет: Только ни..


R-S пишет:
 цитата:
Только никто не отменял Закон Фарадея и уравнения идеального трансформатора.

Интересно, кто кроме Вас применяет "идеальные силовые трансформаторы" в аудио?
И чего это люди "изгаляются" с аудиотрансформаторами? Кстати, Вы тоже.
RS пишет:
 цитата:
Траф намотан под нагрузку 8 Ом. Ему на выход нагружаем 6 или 4 Ом.
Соответственно меняются входные параметры. Верно?

Верно так же и то, что на частоте 50Гц динамик может иметь сопротивление 5 Ом, на частоте 500Гц - 10 Ом, на частоте 1000Гц - 18 Ом и т.д. Хотя на постоянном токе - 7.2 Ом-а! Вы на какой частоте предпочитаете слушать свой "комплекс"? Вот под такое частотное сопротивление и рассчитывайте трfнсформатор.
Ах да, я забыл - Вы слушеете " идеальный силовой" трансформатор - тогда да, частота сети довольно стабильна, так что считайте на 50Гц и "будет Вам счастье"!

Говори что думаешь. Думай, что говоришь! Спасибо: 0 
Профиль
r9o-11





Пост N: 694
Зарегистрирован: 28.01.15
Откуда: Искитим, НСО
Рейтинг: 4
ссылка на сообщение  Отправлено: 05.12.19 23:14. Заголовок: RedStar, так он у ме..


RedStar, так он у меня есть, но в аудиоизмерениях всё равно предпочитаю пользоваться Спектралабом. Вот смотрите, как раз Спектралабовский скрин, сделанный при проверке какого-то макета:



Что мы здесь видим? Разница между основным тоном и 2-й гармоникой чуть менее 40 dB и показания THD более 1%. Вроде как, здесь теория с практикой сходятся...

Андрей Спасибо: 0 
Профиль
RedStar
постоянный участник




Пост N: 2817
Зарегистрирован: 02.08.16
Откуда: Смоленск
Рейтинг: 1
ссылка на сообщение  Отправлено: 05.12.19 23:15. Заголовок: volli пишет: кто кр..


volli пишет:
 цитата:
кто кроме Вас применяет "идеальные силовые трансформатора" в аудио?

В который раз. Кто вам сказал, что применяю расчеты от силовиков?
Закон трансформации НИКТО НЕ ОТМЕНЯЛ! Как малые дети...
 цитата:
Верно так же и то, что на частоте 50Гц динамик может иметь сопротивление 5 Ом, на частоте 500Гц - 10 Ом, на частоте 1000Гц - 18 Ом и т.д

Верно. Но измерения проводите на Постоянном резисторе? Комплекс почему то многие не учитывают?
Увеличение нагрузки, более расчетной, к вашему примеру, только благотворнее сказывается на ток первички не принуждая лампу уменьшать ЭДС тран-ра.
Есть попытка определения минимальной нагрузки, что бы обеспечить подъем индуктивного сопротивления динамика. Но промолчу, и вам не советую пока это затрагивать.
 цитата:
Вы слушеете " идеальный силовой" трансформатор - тогда да, частота сети довольно стабильна, так что считайте на 50Гц и "будет Вам счастье"!

Ошибаетесь. Рассчитываю на 30 Гц - получаю 30. Рассчитываю на 20 - получаю 20.

r9o-11 пишет:
 цитата:
мотрите, как раз Спектралабовский скрин, сделанный при проверке какого-то макет

Сделайте сравнение обоих программ, без изменения входных параметров.
Покажите скрины. Готов обсудить с вами (не здесь, а в теме про осцилл.)
Возможно у вас не настроен прибор или проблемки в звуковой карте. Всякое бывает.
 цитата:
разница между основным тоном и 2-й гармоникой чуть менее 40 dB и показания THD более 1%.

Какой метод измерения предустановлен? Их там более 20.

Не надо иметь много колес, лучше иметь много спиц, удерживающих одно колесо. © Спасибо: 0 
Профиль
r9o-11





Пост N: 695
Зарегистрирован: 28.01.15
Откуда: Искитим, НСО
Рейтинг: 4
ссылка на сообщение  Отправлено: 05.12.19 23:27. Заголовок: А, кажется, я понял,..


А, кажется, я понял, почему китайские проценты такие - они вычисляют именно коэффициент нелинейных искажений сигнала, но не в процентном отношении, т.е. не умножают полученный результат на 100% ( [здесь можно почитать - там есть строка:

"Коэффициент безразмерный, но обычно умножается на 100% для получения значения в %."[/url] ).

А почему к этому значению они подставляют значок "%" - да фиг его знает, им, наверное, наши арабские цифры ничего не говорят - у них же своё написание чисел...

Переизмерять и перепроверять я ничего не буду - пользуюсь спектралабом более 10 лет, всё проверено неоднократно на куче карт, компьютеров и измерений.

Андрей Спасибо: 0 
Профиль
RedStar
постоянный участник




Пост N: 2819
Зарегистрирован: 02.08.16
Откуда: Смоленск
Рейтинг: 1
ссылка на сообщение  Отправлено: 05.12.19 23:31. Заголовок: r9o-11, Не верно, со..


r9o-11, Не верно, совсем.
В чистом пентоде получаю реальные значения искажений до 10%. Скаждете, что там не 10%, а 100%. Даже 2% мне коробит уши так, что словно мимо меня едет Белаз!
а также во многих других случаях при неправильно выбранных режимах. В триоде добивался 0,8% чистых.
Предлагаю вам перенести это в тему об осцилле.
 цитата:
Переизмерять и перепроверять я ничего не буду - пользуюсь спектралабом более 10 лет, всё проверено неоднократно.

Зря. Спектралаб зависит от карты и настройки, более тщательной, чем готовый прибор.

Не надо иметь много колес, лучше иметь много спиц, удерживающих одно колесо. © Спасибо: 0 
Профиль
r9o-11





Пост N: 696
Зарегистрирован: 28.01.15
Откуда: Искитим, НСО
Рейтинг: 4
ссылка на сообщение  Отправлено: 05.12.19 23:36. Заголовок: Не надо ничего перен..


Не надо ничего переносить - всё здесь написанное относится именно к тому, как все мы бываем уверенны в правильности своих измерений.

Андрей Спасибо: 0 
Профиль
RedStar
постоянный участник




Пост N: 2820
Зарегистрирован: 02.08.16
Откуда: Смоленск
Рейтинг: 1
ссылка на сообщение  Отправлено: 05.12.19 23:39. Заголовок: r9o-11 пишет: все м..


r9o-11 пишет:
 цитата:
все мы бываем уверенны в правильности своих измерений.

Относительно. И заблуждение. А проверять чем то другим? Привыкли видимо.

Не надо иметь много колес, лучше иметь много спиц, удерживающих одно колесо. © Спасибо: 0 
Профиль
r9o-11





Пост N: 697
Зарегистрирован: 28.01.15
Откуда: Искитим, НСО
Рейтинг: 4
ссылка на сообщение  Отправлено: 06.12.19 00:00. Заголовок: Какой Вы нудный... Н..


Какой Вы нудный... Ну, ладно, минутное дело - запустил запись 1кГц сигнала с THD 1% с тестового диска и посмотрел

Андрей Спасибо: 0 
Профиль
RedStar
постоянный участник




Пост N: 2821
Зарегистрирован: 02.08.16
Откуда: Смоленск
Рейтинг: 1
ссылка на сообщение  Отправлено: 06.12.19 00:12. Заголовок: r9o-11 пишет: Какой..


r9o-11 пишет:
 цитата:
Какой Вы нудный...

Не нудный, а дотошный. Все нравится, а многое не нравится.
Нравится экспериментировать и получать удовольствие. Просто для себя. Для получения адекватных значений при применении формул. Кому как, а мне ответственно подходить надо.
Не нравится - это тупое следование основам пятилеток.
 цитата:
...и посмотрел

Смешного там нет ничего. Это так, о смайлах.
Судя по графику, очень большой шум от БП. Хотя очень хорошо давит основную гармонику питающей сети. Вам респект!
П.С. Еще раз повторю. Переносим это обсуждение в тему про осцилл. Хорошо?

Не надо иметь много колес, лучше иметь много спиц, удерживающих одно колесо. © Спасибо: 0 
Профиль
r9o-11





Пост N: 698
Зарегистрирован: 28.01.15
Откуда: Искитим, НСО
Рейтинг: 4
ссылка на сообщение  Отправлено: 06.12.19 00:15. Заголовок: Зачем переносить? Ес..


Зачем переносить?
Если не по теме, то, наверное, удалить надо.

То же измерение на другой карте

Андрей Спасибо: 0 
Профиль
Кузьмич



Пост N: 1507
Зарегистрирован: 21.11.15
Рейтинг: 8
ссылка на сообщение  Отправлено: 06.12.19 00:40. Заголовок: RedStar пишет: Хоро..


RedStar пишет:
 цитата:
Хорошо, допустим ты произвел расчет выходного тран-ра по теории. Есть уверенность в том, что твой расчет совпадет с реальным изделием?

Абсолютно. У меня и получается всегда с погрешностью плюс-минус 5% в домашних условиях. И это отличный результат.
Здесь на форуме есть мои посты по ТВЗ, даже индуктивность рассеяния совпала с расчётной менее чем в 5%.
Со 100% -ой вероятностью невозможно изготовить даже на производстве. Этому существует ряд причин. Да это и не нужно.
 цитата:
объясни, каким образом, по формуле 8.23, выходит мой Гуттаперчевый на мощность от 20 Вт? Хотя там только 10 Вт?

А я что, экстрасенс? Или ты хочешь, что бы всю твою тему перепроверил?
Ищи свои ошибки. Кто ищет, тот найдёт. И вообще, не прыгай с пята на десята, разберись с одним сначала.

Спасибо: 0 
Профиль
RedStar
постоянный участник




Пост N: 2822
Зарегистрирован: 02.08.16
Откуда: Смоленск
Рейтинг: 1
ссылка на сообщение  Отправлено: 06.12.19 02:20. Заголовок: r9o-11 пишет: Если ..


r9o-11 пишет:
 цитата:
Если не по теме, то, наверное, удалить надо.

Перенести. Так лучше было бы.
В вашем графике теперь больше ВЧ шума стало. Есть общий подъем в области высших гармоник.
Больше леса получили. Мне не нравится. Ужасный звук будет.
Я прекращу говорить на эту тему здесь. Прошу перейти на другую.

Кузьмич пишет:
 цитата:
У меня и получается всегда с погрешностью плюс-минус 5% в домашних условиях. И это отличный результат.

Может быть да, но этого не отражается в твоих разработках, коих нет на форуме. Бабка надвое сказала...
Сказать? что-то и я могу, апеллируя образностью, а может своеобразным мышлением?
 цитата:
Со 100% -ой вероятностью невозможно изготовить даже на производстве.

Абсолютно согласен с тобой. Этого вряд ли кто добьется. И мне не нужны заоблачные дали.
 цитата:
А я что, экстрасенс?

Дело не в экстрасенсорике, а предоставленных, мною в теме, Измеренных значениях. Таким расчетом, о котором толкую, не владеете, что бы реально подсчитать
 цитата:
Или ты хочешь, что бы всю твою тему перепроверил?

Зачем? Есть схема, есть данные. Сложно прочитать? Подумав хорошо, там очень просто решается.
 цитата:
Ищи свои ошибки.

Да, есть там, немного, ошибок. Признаю то, что допустил отклонение в расчетах.
Не заметишь их со своим эгоцентризмом. Но это не помешает понять идею применения дополнительных обмоток, если не много уклонитесь от основ.
Все просто, до банальности.

Не надо иметь много колес, лучше иметь много спиц, удерживающих одно колесо. © Спасибо: 0 
Профиль
Плюмбум





Пост N: 292
Зарегистрирован: 14.12.16
Рейтинг: 3
ссылка на сообщение  Отправлено: 06.12.19 03:29. Заголовок: RedStar пишет: Тольк..


RedStar пишет:
 цитата:
Только объясни, каким образом, по формуле 8.23, выходит мой Гуттаперчевый на мощность от 20 Вт? Хотя там только 10 Вт?

Считать не умеешь, вот и вся причина.

Спасибо: 0 
Профиль
RedStar
постоянный участник




Пост N: 2823
Зарегистрирован: 02.08.16
Откуда: Смоленск
Рейтинг: 1
ссылка на сообщение  Отправлено: 06.12.19 03:51. Заголовок: Плюмбум пишет: Счит..


Плюмбум пишет:
 цитата:
Считать не умеешь, вот и вся причина.

Сказали раз, скажите два.
Подсчитайте, если не согласны.. Или только слова одни есть, без подкрепления?

Не надо иметь много колес, лучше иметь много спиц, удерживающих одно колесо. © Спасибо: 0 
Профиль
Плюмбум





Пост N: 293
Зарегистрирован: 14.12.16
Рейтинг: 3
ссылка на сообщение  Отправлено: 06.12.19 04:37. Заголовок: RedStar пишет: Подсч..


RedStar пишет:
 цитата:
Подсчитайте, если не согласны.. Или только слова одни есть, без подкрепления?

Нет, дорогОй! Ты сделал заявление на ошибочность формулы Войшвилло, ты и должен был бы показать, как считал. Но ты сделал голословное утверждение, а с меня требуешь расчёт...
Далее:
 цитата:
Из этой формулы:
Ктр=√(Ra/Rn)
А как же Ra=U/I ? Там нет напряжения и тока?

Это формула Коэффициента трансформации. А он может быть вычислен как:

- отношение напряжений Ктр=U1/U2,
- отношение токов Ктр= I2/I1
- как кв. корень из отношения сопротивлений Ктр=√(Ra/Rn)

В разных конкретных случаях можно пользовать любое из этих соотношений.
При расчёте трансформатора удобно применить третью формулу.
Тебе, правда, это сложно понять, раз такой вопрос задал...

Спасибо: 0 
Профиль
Кузьмич



Пост N: 1508
Зарегистрирован: 21.11.15
Рейтинг: 8
ссылка на сообщение  Отправлено: 06.12.19 05:00. Заголовок: RedStar пишет: Сказ..


RedStar пишет:
 цитата:
Сказать? что-то и я могу, апеллируя образностью, а может своеобразным мышлением?

Чудак ты Толя, на букву МЭ. Мелешь языком всякую чушь. Кичишься китайской дешёвкой, которой правильно пользоваться не научился.
 цитата:
Таким расчетом, о котором толкую, не владеете, что бы реально подсчитать

Этим расчётом "владеешь" только ты. Только гордиться им не получится, сплошная лажа.
 цитата:
Есть схема, есть данные. Сложно прочитать? Подумав хорошо, там очень просто решается.

Я для тебя должен посчитать? Не много чести? Просто? А что же у тебя там ошибки???
 цитата:
если не много уклонитесь от основ.

Какая чушь???!!! Для сравнения, ты предлагаешь умножать по какой-то другой таблице умножения. Самому - не смешно?
А, да... Закон Ома у тебя другой, свой...
 цитата:
Все просто, до банальности.

Ага, а то тут никто не видит.

Плюмбум пишет:
 цитата:
Это формула Коэффициента трансформации. А он может быть вычислен как:

Есть ещё вариант: Ктр = W1 / W2, или n = W2 / W1
Есть ещё варианты.

Спасибо: 0 
Профиль
Ответов - 299 , стр: 1 2 3 4 5 6 7 8 9 10 11 12 13 14 15 All [только новые]
Тему читают:
- участник сейчас на форуме
- участник вне форума
Все даты в формате GMT  8 час. Хитов сегодня: 281
Права: смайлы да, картинки да, шрифты да, голосования нет
аватары да, автозамена ссылок вкл, премодерация откл, правка нет